Download git

Document related concepts

Prenatal testing wikipedia , lookup

Medical ethics wikipedia , lookup

Dental emergency wikipedia , lookup

Patient safety wikipedia , lookup

Electronic prescribing wikipedia , lookup

Adherence (medicine) wikipedia , lookup

Dysprosody wikipedia , lookup

List of medical mnemonics wikipedia , lookup

Transcript
A 51-year-old man is admitted to the hospital because of alcoholic pancreatitis.
The patient has a long history of alcohol abuse and has been admitted multiple
times with alcohol withdrawal seizures. The patient called 911 after
experiencing 3 days of worsening abdominal pain. On arrival to the emergency
room, he reported that he had been drinking approximately 1 quart of vodka
per day for the past week. About 4 days ago, he developed midepigastric
abdominal pain that has worsened to 10/10 pain. He has not had any food or
liquids in over 24 hours. Admission laboratory tests revealed profoundly
elevated amylase and lipase. An abdominal CT demonstrated pancreatic
stranding consistent with pancreatitis. The patient now reports being mildly
short of breath. He has decreased breath sounds at the left base and there is
some dullness to percussion in the same area. The most appropriate diagnostic
test at this time is
A. chest radiographs in decubitus views
B. a CT scan of the chest
C. a thoracocentesis
D. an ultrasound of the abdomen
E. a ventilation perfusion scan
Explanation:
The correct answer is A. This patient has pancreatitis as evidenced by his
symptoms, laboratory values, and CT scan. There are many complications
associated with pancreatitis and many of them relate to the pulmonary system.
In the case of this patient, he has unilateral dullness to percussion with absent
breath sounds suggesting pleural effusion. Diagnosis of pleural effusions is
best accomplished with chest radiographs in both the right and left decubitus
positions. In general, only moderate sized effusions are visible since the height
of the effusions will appear approximately 1cm in decubitus views.
There is no indication that the patient has a pulmonary process that requires
advanced imaging such as a CT scan of the chest (choice B). If the patient had
a chest radiograph that revealed some parenchymal or pleural abnormality not
adequately resolved on plain films, then a CT would be appropriate.
A thoracocentesis (choice C) would be the appropriate therapeutic procedure
once a diagnosis of effusion is made. Once an effusion is confirmed,
thoracocentesis can also be used for more detailed diagnosis in terms of
classifying the effusion as either exudative or transudative.
There is no indication for ultrasound of the abdomen (choice D) at this time.
The patient has already had an abdominal CT scan and has diagnosed
pancreatitis. Since there is no suggestion of biliary ductal dilation or
gallbladder involvement, this test adds no useful information to the
management options for the patient.
Unless the suspicion for pulmonary embolism (PE) is high, there is no need for
a ventilation perfusion scan (choice E). The most frequent signs and symptoms
of PE are sinus tachycardia, tachypnea, elevated jugular venous pulse, right
axis deviation on EKG, hypoxia, and pleuritic chest pain. The patient
complains only of mild shortness of breath which conveys a low clinical
suspicion of PE.
You are called to see a patient with end-stage liver disease secondary to
hepatitis C obtained from injection drug abuse. He reports that he has
experienced increasing abdominal girth for the last 2 weeks. He also notes that
his urine output has been minimal for the last 3 days, producing approximately
30 cc of urine each day. His temperature is 37 C (98.6 F), blood pressure is
95/60 mm Hg, pulse is 70/min, and respirations are 19/min. Physical
examination reveals scleral icterus, huge abdominal distention with bulging
flanks, and a fluid wave. His lower extremities have 2+ edema. Laboratory
studies show:
Sodium
128 mEq/dL
Potassium
4.8 mEq/dL
Chloride
98 mEq/dL
Bicarbonate
21 mEq/dL
BUN
28 mg/dL
Creatinine
3.2 mg/dL
Urinalysis
Color
Clear
Specific gravity
1.020
Osmolality
55 mOsmol/kg
Leukocyte esterase
Negative
Nitrite
Negative
Protein
Negative
Blood
Negative
Microscopic
Few hyaline casts
Urine Sodium
4 mEq/L
In an effort to increase urine output, you perform a therapeutic paracentesis
and provide a fluid challenge with 500 ml normal saline. Urine output does not
improve. He is "so sick of all of this" and wants to know what is the most
effective treatment. He should be told that his condition can be most
effectively managed with
A. continued fluid resuscitation with normal saline
B. intravenous albumin therapy
C. liver transplantation
D. renal dose dopamine therapy
E. treatment with furosemide
Explanation:
The correct answer is C. This patient has hepatorenal syndrome (HRS). The
treatment is a liver transplant. It is seen in end-stage liver disease in patients
with chronic liver disease more often than in acute fulminant hepatic failure. It
is sometimes difficult to differentiate from prerenal azotemia. Typically, the
urine sodium in prerenal azotemia is less than 20 mEq/L, while the urine
sodium with hepatorenal syndrome is less than 10 mEq/L. Urine output will
not increase with a fluid challenge in HRS while it will in prerenal azotemia.
The management of HRS should include removal of ascites, and an attempt to
increase urine output and a small fluid challenge with normal saline or
albumin. Furosemide can be tried but it likely will not work. The prognosis is
very poor unless the patient receives a liver transplant. The kidneys of these
patients are normal and will begin to work with a new liver.
Fluid resuscitation (choice A) has been tried without success already in this
case. Continued aggressive hydration of this patient will likely increase his
extravascular volume (e.g., ascites and lower extremity edema) without
increasing his urine output.
Resuscitation with albumin (choice B) is thought to be helpful in expanding
intravascular volume in cirrhotic patients since their underlying problem is low
oncotic pressure in the intravascular space. Albumin might help to temporarily
bring fluid into the vessels but will likely also eventually become
extravascular. However, this patient is likely not prerenal, so volume
expansion is unlikely to be effective.
Renal dose dopamine (choice D) is controversial. It is thought to work by
dilating renal vasculature and increasing blood flow to the kidneys. At low
doses, dopamine acts selectively in renal vasculature, while at higher doses it
acts non-selectively like norepinephrine. Studies have not shown renal dose
dopamine to be effective but it is still fairly widely used despite inconclusive
evidence.
Furosemide (choice E) is part of the management of cirrhotic patients as a way
to decrease ascites and decrease peripheral edema. In HRS, furosemide can be
attempted but it will likely not work. The most effective management of this
patient with HRS is clearly liver transplant.
A 55-year-old woman comes to the emergency department because of
abdominal pain. She had just finished eating a steak dinner with her family
when she suddenly experienced sharp, crampy pain in the upper right and
middle of her abdomen. The pain has lasted for the past 3 hours and she is
starting to feel nauseous. On physical examination, she is obese and in obvious
discomfort. Her temperature is 38.8 C (101.8 F), blood pressure is 140/87 mm
Hg, pulse is 90/min, and respirations are 16/min. Abdominal examination is
significant for focal tenderness and guarding in her right upper quadrant. She is
particularly tender when you palpate her right upper quadrant as she takes in a
deep breath. The most appropriate next step in the evaluation of her abdominal
pain is
A. an abdominal x-ray
B. a CT of the abdomen
C. an endoscopic retrograde cholangiopancreatography (ERCP)
D. serum liver function tests including bilirubin
E. an ultrasonography of the abdomen
Explanation:
The correct answer is E. Crampy pain in the right upper quadrant or
epigastrium is classic for biliary colic. The pain is thought to be due to
obstruction of the cystic duct by gallstones and often follows a fatty meal.
Acute cholecystitis is a concern when biliary colic is accompanied by a fever,
leukocytosis, nausea, and vomiting. Right upper quadrant pain precipitated by
deep inspiration during palpation is known as Murphy's sign and is highly
suggestive of acute cholecystitis. The presence of gallstones in the clinical
setting of right upper quadrant pain and fever is sufficient to make the
diagnosis of acute cholecystitis. Ultrasonography of the abdomen is the
diagnostic test of choice to evaluate for gallstones. Ultrasonography is highly
sensitive and specific for gallstones, and can also demonstrate inflammation of
the gallbladder. During ultrasonography, a sonographic Murphy's sign can also
be tested for by using the transducer to press over the region of the gallbladder.
The presence of gallstones and a sonographic Murphy's sign are highly
suspicious for acute cholecystitis.
An abdominal x-ray (choice A) is not a good study to evaluate for gallstones.
Most gallstones are composed of cholesterol and are radiolucent on abdominal
x-rays. Only 10-15% of gallstones contain enough calcium to appear radio
opaque on x-rays.
A CT of the abdomen (choice B) is not the first or the best test in the
evaluation of gallstones or acute cholecystitis. Most gallstones are composed
of cholesterol and may not be clearly distinguishable from adjacent bile in the
gallbladder on a CT scan. A CT scan provides information about inflammation
around the gallbladder and biliary ductal dilatation, but it is not a sensitive or
specific diagnostic study for gallstones.
An endoscopic retrograde cholangiopancreatography or ERCP (choice C) is
performed by gastroenterologists to evaluate the biliary ductal system. It is the
procedure of choice when a common bile duct stone or choledocholithiasis is
suspected. When a patient has gallstones and common bile duct dilatation on
ultrasonography of the abdomen, ERCP is useful in the evaluation of the
biliary system as well as for clearing the obstructing stone. ERCP is not
indicated in cases of simple biliary colic or acute cholecystitis without signs of
common bile duct obstruction.
Serum liver function tests including bilirubin (choice D) can provide useful
information about biliary obstruction. Elevated bilirubin and frank jaundice are
clinical indicators of biliary obstruction, most commonly in the common bile
duct. These serum laboratory tests do not help in the diagnosis of gallstones or
acute cholecystitis.
A 47- year-old man comes to the emergency department because he is "not
feeling well and his abdomen is bloated and painful." He denies any previous
medical history. He reports that he has had similar episodes in the past, which
resolved spontaneously. This episode started 12 hours ago, when he started
feeling discomfort and pain in the abdomen. He has not passed flatus since
then. He is feeling nauseous. His temperature is 38.1 C (100.6 F), blood
pressure is 146/80 mm Hg, pulse is 94/min, respirations are 16/min, and
oxygen saturation is 98% on room air. His abdomen is distended with fullness
in the right upper quadrant and empty in the left lower quadrant. He has
marked tenderness in the left lower quadrant. Rectal examination is positive
for occult blood. His leukocyte count is 16,000/mm3. A chest x-ray is
unremarkable. An abdominal x-ray shows a distended colonic loop pointing
towards the left lower quadrant. The most appropriate next step in management
is to
A. admit the patient for a colonoscopy on the following day
B. obtain an emergency surgical consult and prepare the patient for the
operating room
C. order a barium enema
D. order a CT scan of the abdomen
E. order tap water enemas
Explanation:
The correct answer is B. This patient presents with the classic symptoms and
signs of sigmoid volvulus. An abdominal x-ray is usually diagnostic. The next
step in the management depends upon the clinical examination. In the absence
of peritoneal signs, flexible sigmoidoscopy helps in the detorsion of the colon.
When the volvulus is not treated for a long time, part of the sigmoid colon can
become ischemic. In the presence of a tender abdomen associated with
tachycardia, fever, and a raised total leukocyte count, like in this patient,
ischemia should be suspected. This patient should be taken to the operating
room for immediate detorsion of the volvulus and to resect the ischemic bowel.
Patients with partial intestinal obstruction should be admitted for non-operative
management to decompress the bowel and a colonoscopic examination to rule
out an obstructing lesion, only in the absence of peritoneal signs. Nonoperative
management (choice A) is contraindicated if the patient develops peritoneal
signs or evidence of complete bowel obstruction either clinically or
radiologically.
A barium enema (choice C) may be diagnostic in the management of colonic
obstruction as an elective investigation. In children with intussusception, it is
both diagnostic and therapeutic. In a patient with peritoneal signs, either an
adult or a child, a barium enema is harmful.
A CT scan of the abdomen (choice D) is helpful to rule out diverticulitis in a
patient with a similar history and physical exam, but without evidence of
volvulus on the abdominal x-ray. In patients with diverticulitis, the abdomen is
usually uniformly and mildly distended. In the presence of classic signs of
volvulus, with a distended colonic loop in the right upper quadrant and tip
pointing towards left lower quadrant, a CT scan of the abdomen is not
indicated.
Tap water enemas are harmful in an acute abdomen without a definitive
diagnosis (choice E).
A 55-year-old woman is brought to the emergency department by her daughter
because of left lower quadrant abdominal pain, anorexia, fever, and chills for
the past 24 hours. Her temperature is 38.7 C (101.6 F), pulse is 110/min, and
respirations are 18/min. She is awake and alert, although she appears
uncomfortable. Examination shows hypoactive bowel sounds and a soft
abdomen with mild voluntary guarding especially in the left lower quadrant.
Digital rectal examination is significant for heme-positive stool. An
electrocardiogram shows a sinus tachycardia at 110 beats per minute. There are
no ST segment changes when compared with old electrocardiograms. A chest
x-ray shows no acute disease. Abdominal x-ray demonstrates no air under the
diaphragm and no ileus. A complete blood count, biochemical profile, cardiac
enzymes, amylase, and lipase are drawn, but results are not yet available. The
most appropriate initial management of this patient is to
A. admit for intravenous antibiotics, nil per os diet, and abdominal CT scan
B. discharge to home on a clear liquid diet and PO antibiotics
C. immediately refer to a gastroenterologist for colonoscopy
D. prepare the patient for immediate surgical exploration
E. send patient for urgent cardiac catheterization
Explanation:
The correct answer is A. This patient most likely has diverticulitis, an
inflamed herniation of the mucosa of the colon through the muscular layer of
the bowel wall. The best way to diagnose diverticulitis is with an abdominal
CT scan. The appropriate management of this patient is admission to the
hospital. She should be kept NPO and given IV antibiotics.
Mild cases of diverticulitis may be treated as an outpatient with a clear liquid
diet and PO antibiotics (choice B). However, this patient is too sick to be
discharged and ought to be treated with IV antibiotics.
A colonoscopy (choice C) is useful to diagnose diverticulosis, but would not
be used during an acute attack of diverticulitis because of risk of perforation.
Indications for urgent surgical intervention include abscess formation, severe
disease, or confirmed perforation. This patient does not yet show signs of
needing surgical intervention (choice D).
Abdominal pain is often the presenting sign of cardiac ischemia. However, this
patient has few risk factors, no EKG changes, and does have abdominal exam
findings. Therefore, cardiac catheterization (choice E) would not be indicated.
You should, however, still continue to monitor her cardiac function and obtain
cardiac enzymes.
A 58-year-old woman comes to the emergency department complaining of
crampy left upper quadrant pain that is exacerbated by fatty foods. She has a
history of diabetes, hyperlipidemia, and gallstones and her medications include
glyburide, simvastatin, and aspirin. She denies any alcohol or drug use. She is
morbidly obese and her temperature is 37.9 C (100.2 F), blood pressure is
102/87 mm Hg, pulse is 105/min, and respirations are 23/min. On examination,
her lungs are clear to auscultation bilaterally. Her cardiac sounds are muffled,
although her cardiac rhythm is regular. No murmurs are audible. She has
definite left upper quadrant tenderness to palpation, without rebound or
guarding. Rectal examination shows guaiac-negative brown stool. Her amylase
and lipase levels are elevated. The most appropriate next step is to order a
A. an abdominal ultrasound
B. a chest radiograph
C. an electrocardiogram
D. a HIDA scan
E. an upper endoscopy
Explanation:
The correct answer is A. Given that this patient has pancreatitis, it is
reasonable to suspect gallstones as its etiology given the available
epidemiologic factors (sex, age, obesity). If gallstones are present, an ERCP
and gallstone removal can be considered.
A chest radiograph (choice B) is not indicated at this juncture given that her
lung exam is benign and there is no evidence of a pulmonary pathology.
An electrocardiogram (choice C) is not indicated at this juncture since cardiac
pathology is not evident on history or exam and the patient appears to be in
sinus rhythm.
A HIDA scan (choice D), sometimes used for the diagnosis of cholecystitis, is
not used in the diagnosis of pancreatitis. An upper endoscopy (choice E) is not
indicated in the setting of pancreatitis without confounding pathologies such as
variceal bleeding or gastric ulceration.
An asymptomatic previously healthy 60-year-old man comes to the office
because he is found to have a liver mass. Recently, he had epigastric and right
upper quadrant pain, which was investigated by means of ultrasound.
Sonography demonstrated a lesion in the right lobe of the liver, but no
gallstones or evidence of cholecystitis. Further investigations by means of
endoscopy revealed gastritis from Helicobacter pylori, for which he was
treated. He is concerned about this liver mass and hence, comes to the office.
The liver mass is described as an 8-cm solitary lesion within the right lobe of
the liver. No enterohepatic biliary ductal dilatation was noticed. A CT scan of
the abdomen performed with contrast demonstrated a progressive peripheral to
central prominent enhancement and a central hypodense region. An MRI
shows a dense T2 weighted image. The most appropriate next step in the
management of this patient's liver lesion is
A. celiac arteriography
B. observation
C. percutaneous needle biopsy
D. radiation therapy
E. resection
Explanation:
The correct answer is B. The characteristic lesion described in this patient fits
in well with a diagnosis of a cavernous hemangioma. A cavernous
hemangioma is a hamartomatous transformation blood vessel. This can be
classically diagnosed with a contrast CT or a T2 weighted image on MRI. A
radiolabeled red blood cell scan can diagnose a hemangioma, but is not usually
necessary. Observation is typically indicated, as rupture is rare.
Celiac arteriography (choice A) would be able to diagnose a hemangioma, but
is not necessary in the workup of this patient.
A percutaneous needle biopsy (choice C) is contraindicated because of the risk
of bleeding.
Radiation therapy (choice D) has no established role in the treatment of a
hemangioma.
Resection (choice E) is indicated for a rapidly expanding hemangioma, for a
symptomatic lesion, or if the diagnosis is uncertain. Enucleation is usually
carried out in these cases.
A 37-year-old woman comes to the office because of a "burning sensation" in
the chest for the past 3 months. The "burning" typically begins in the "upper
stomach and travels up to the neck." The symptoms worsen when she lies
down to go to sleep. She is a chef at a local American restaurant, has 3
children, and has been married for 12 years. She "tries" to eat a healthy diet,
but it is difficult because she is around food all day and night. She has no
chronic medical conditions, takes no medications, and does not drink alcohol
or caffeine-containing beverages. She recently quit smoking. Her temperature
is 37.0 C (98.6 F), blood pressure is 120/80 mm Hg, pulse is 65/min, and
respirations are 14/min. Physical examination is unremarkable. An
electrocardiogram is unremarkable. A complete blood count and metabolic
profile are normal. Serologic testing for H. pylori is negative. The most
appropriate next step is to
A. order ambulatory esophageal pH testing
B. order an upper gastrointestinal barium radiograph
C. recommend elevation of the head of bed and avoidance of food before
bedtime
D. schedule an upper endoscopy
E. schedule esophageal manometry
Explanation:
The correct answer is C. This patient complains of the classic symptoms of
gastroesophageal reflux disease (GERD). Reflux disease is usually worse at
night because the recumbent position allows gastric acid contents to go up into
the esophagus. Since all of the tests ordered in the case were normal, you
should first recommend non-pharmacologic therapy before continuing with
further diagnostic studies. Elevation of the head of bed, avoiding eating before
bed, and avoiding alcohol, tobacco, chocolate, and caffeine should all be
recommended. Alcohol, tobacco, chocolate, and caffeine all lower the lower
esophageal sphincter pressure leading to gastric reflux. If these measures are
ineffective, pharmacologic therapy with a H2 blocker such as cimetidine,
famotidine, or ranitidine is indicated. For more severe symptoms, a proton
pump inhibitor, such as omeprazole or lansoprazole, is indicated.
Ambulatory esophageal pH testing (choice A) is usually reserved for patients
who fail nonpharmacologic and pharmacologic management.
An upper gastrointestinal barium radiograph (choice B) is useful in detecting
esophageal rings or strictures, which typically present with dysphagia. This
patient complains of heartburn, not dysphagia.
An upper endoscopy (choice D) is usually indicated only after the failure of
nonpharmacologic and pharmacologic management for GERD and when a
patient has GERD for >5 years, and upper endocsopy is recommended to
screen for Barrett's metaplasia. However, it is not indicated at this time.
Esophageal manometry (choice E) is typically reserved for cases of GERD
when surgical therapy is being considered.
A 50-year-old man with Crohn's disease comes to the clinic for a routine
follow-up appointment. He was diagnosed with Crohn's disease approximately
15 years ago. He is currently taking prednisone and sulfasalazine, and reports
feeling well. He says he still occasionally has watery diarrhea, but denies fever,
abdominal pain, or weight loss. He had a colonoscopy 1 year ago which
demonstrated a few transmural inflammatory lesions in his descending colon.
The most important management of this patient is
A. increase prednisone dose
B. increase sulfasalazine dose
C. prophylactic colectomy
D. surveillance barium enema every year
E. surveillance colonoscopy every year
Explanation:
The correct answer is E. The most important recommendation for this patient
is a surveillance colonoscopy every year in an effort to detect colon cancer
early. Although the risk of colon cancer in Crohn's disease is much less than in
ulcerative colitis, the risk increases significantly with involvement of the
colon, and if the disease has been present for more than 10 years. This patient
is at increased risk for colon cancer because he has had Crohn's disease for at
least 15 years and has evidence of colon involvement. A colonoscopy is
recommended because it is superior at detecting small lesions and biopsy of
suspicious lesions can also be performed simultaneously.
An increase in prednisone dosage (choice A) is not indicated in this patient at
this time. He reports feeling well and has only rare episodes of diarrhea.
Medication adjustments should be made based on the patient's symptoms.
An increase in sulfasalazine dosage (choice B) is not indicated in this patient at
this time. He reports feeling well and has only rare episodes of diarrhea.
Medication adjustments should be made based on the patient's symptoms.
Prophylactic colectomy (choice C) is not indicated or recommended for this
patient. Many patients with Crohn's disease who have extensive colitis undergo
colectomy early in the course of disease to relieve persistent symptoms. This
patient does not have severe symptoms nor does he have prior colonoscopy
findings of dysplasia to warrant a colectomy. Prophylactic colectomy is often
recommended for patients with ulcerative colitis with long standing colitis due
to the increased risk of colon cancer.
Surveillance barium enema (choice D) is not the best recommendation for this
patient. He has had Crohn's disease for over 10 years and is at increased risk
for developing colon cancer. A barium enema is not as sensitive or specific for
the detection of early colon cancer. Colonoscopy is recommended because it is
superior at detecting small lesions and biopsy of suspicious lesions can also be
performed simultaneously.
A 47-year-old man with a long history of alcohol abuse comes to the
emergency department after "vomiting blood." He denies any abdominal pain,
chest pain, shortness of breath, weakness, or numbness but does feel
lightheaded when he stands and noticed some dark stool yesterday. He drank
"some whiskey" last night but denies any smoking or injection drug use. He
has no prior history of gastrointestinal bleeding or known liver disease. He
occasionally takes aspirin for his hangovers. His temperature is 37.0 C (98.6
F), blood pressure is 128/72 mm Hg, and pulse is 98/min lying down.
Standing, his blood pressure is 98/61 mm Hg, pulse is 112/min, and
respirations are 16/min. Physical examination shows 1+ pitting edema in the
lower extremities and black, guaiac-positive stool. A chest and abdominal xray are normal. An electrocardiogram shows sinus tachycardia at 98/min.
Laboratory studies show:
The most appropriate next step in the management of this patient is to
A. administer intravenous cimetidine
B. begin intravenous fluid resuscitation
C. order a CT scan of the abdomen
D. perform nasogastric lavage
E. perform an upper endoscopy
Explanation:
The correct answer is B. In evaluating and managing a patient with a GI
bleed the most important first step is to stabilize their vital signs. By history
and exam this patient has evidence for a significant upper GI bleed and he is
orthostatic. It is therefore imperative to obtain intravenous access and to start
fluid resuscitation preferably with normal saline immediately.
In the management of patients with acute upper GI bleeding intravenous H2
blockers (choice A) are usually prescribed even though the evidence for this
common practice is weak. Nevertheless, in our patient the most pressing issue
is to correct his intravascular volume depletion as soon as possible.
In evaluating patients with upper GI bleeds it is usually not necessary to obtain
an abdominal CT scan (choice C) . In this patient with possible cirrhosis
secondary to his alcohol abuse it might be useful at some point to image his
liver with a CT scan or ultrasound when he stabilizes, but it is certainly not the
first step.
Likewise, nasogastric lavage (choice D) is commonly performed in the
emergency department on patients presenting with upper GI bleeding, but as
mentioned previously the first priority should be to hydrate the patient.
Upper endoscopy (choice E) should be performed on all patients who present
with a significant upper GI bleed. Endoscopy is the gold standard for the
diagnosis of the particular lesion that caused the bleeding and can also provide
important prognostic information on the chance for rebleeding. Therapeutic
endoscopy can also help stop the bleeding with various techniques. The timing
of the endoscopy is also somewhat controversial but all authorities agree that
the first step should be to fluid resuscitate the patient.
A 63-year-old retired airline pilot comes to the office for a first time visit. He
is switching primary care doctors because he is very unhappy with the care that
he had been getting elsewhere. He brings his medical records with him and you
see that he has long-standing cirrhosis due to previous alcohol abuse and he
has documented portal hypertension. He has no history of gastrointestinal
bleeding. He has a very long medication list. He tells you that his pharmacist
told him that his medications "were wrong" and that his doctor was prescribing
drugs improperly. He is understandably very concerned. You review his
medication list with him. You find a number of medication incompatibilities.
To prevent disability from this situation, he should be advised to discontinue
A. lactulose and oral potassium
B. propranolol and inhaled albuterol
C. propranolol and isosorbide dinitrate
D. spironolactone and hydrochlorothiazide
E. spironolactone and oral potassium
Explanation:
The correct answer is E. Pharmacology is one of the most important applied
basic science topics in clinical medicine. The possibility of drug interactions is
so important to recognize, that entire organizations (pharmacies) and computer
cross checking (MicroMedex) have been created to address this issue. These
conveniences, however, in no way absolve a physician from knowing the
consequences and potential interactions of each and every medicine that they
prescribe for their patients. Spironolactone is a potassium sparing diuretic. The
mistake of calling this drug a "diuretic" and cueing your brain to think "replete
electrolytes" could cause this patient to die from hyperkalemic arrest.
Lactulose and oral potassium (choice A) is a useful combination for the
treatment of hepatic encephalopathy. Lactulose is a non-metabolizable sugar
that acidifies the stool and thus traps ammonia in the GI lumen for excretion.
Propranolol and inhaled albuterol MDI (choice B) is the combination of a
nonselective beta blocker and an inhaled beta agonist for asthma. By some
literature, the combination has no interactions that are dangerous.
Propranolol and isosorbide dinitrate (choice C) is a combination of a
nonspecific venodilator and a nonspecific beta blocker. This combination is
useful in the treatment of angina since the beta blocker prevents a reflex
tachycardia secondary to the Isordil-induced drop in preload.
Spironolactone and hydrochlorothiazide (choice D) are a combination of a
potassium sparing diuretic and a thiazide diuretic, a combination very effective
in the treatment of cirrhotic edema and ascites.
A 63-year-old retired airline pilot comes to the office for a first time visit. He
is switching primary care doctors because he is very unhappy with the care that
he had been getting elsewhere. He brings his medical records with him and you
see that he has long-standing cirrhosis due to previous alcohol abuse and he
has documented portal hypertension. He has no history of gastrointestinal
bleeding. He has a very long medication list. He tells you that his pharmacist
told him that his medications "were wrong" and that his doctor was prescribing
drugs improperly. He is understandably very concerned. You review his
medication list with him. You find a number of medication incompatibilities.
To prevent disability from this situation, he should be advised to discontinue
A. lactulose and oral potassium
B. propranolol and inhaled albuterol
C. propranolol and isosorbide dinitrate
D. spironolactone and hydrochlorothiazide
E. spironolactone and oral potassium
Explanation:
The correct answer is E. Pharmacology is one of the most important applied
basic science topics in clinical medicine. The possibility of drug interactions is
so important to recognize, that entire organizations (pharmacies) and computer
cross checking (MicroMedex) have been created to address this issue. These
conveniences, however, in no way absolve a physician from knowing the
consequences and potential interactions of each and every medicine that they
prescribe for their patients. Spironolactone is a potassium sparing diuretic. The
mistake of calling this drug a "diuretic" and cueing your brain to think "replete
electrolytes" could cause this patient to die from hyperkalemic arrest.
Lactulose and oral potassium (choice A) is a useful combination for the
treatment of hepatic encephalopathy. Lactulose is a non-metabolizable sugar
that acidifies the stool and thus traps ammonia in the GI lumen for excretion.
Propranolol and inhaled albuterol MDI (choice B) is the combination of a
nonselective beta blocker and an inhaled beta agonist for asthma. By some
literature, the combination has no interactions that are dangerous.
Propranolol and isosorbide dinitrate (choice C) is a combination of a
nonspecific venodilator and a nonspecific beta blocker. This combination is
useful in the treatment of angina since the beta blocker prevents a reflex
tachycardia secondary to the Isordil-induced drop in preload.
Spironolactone and hydrochlorothiazide (choice D) are a combination of a
potassium sparing diuretic and a thiazide diuretic, a combination very effective
in the treatment of cirrhotic edema and ascites.
A 75-year-old man who lives in an elder care facility is brought to the
emergency department because of a sudden onset of severe, colicky abdominal
pain. He is generally in good health and does not take any regular medications.
Alcohol and cigarettes are not permitted in his living facility. His temperature
is 37 C (98.6 F), blood pressure is 110/70 mm Hg, pulse is 65/min, and
respirations are 16/min. Physical examination shows abdominal distention and
high-pitched tinkling sounds. An abdominal x-ray shows a dilated colon,
forming an "omega loop," with the narrowed colonic segment pointing to the
right lower quadrant. Intravenous fluids are started and a nasogastric tube is
placed. The most appropriate management at this time is to
A. admit him to the hospital and begin sulfasalazine and corticosteroids,
intravenously
B. admit him to the hospital and begin vancomycin and metronidazole,
intravenously
C. admit him to the hospital and observe
D. arrange for immediate laparotomy
E. perform a sigmoidoscopy with rectal tube placement
Explanation:
The correct answer is E. This patient most likely has a sigmoid volvulus,
which is caused by the bowel twisting on its mesentery, and leads to
obstruction and possibly vascular compromise. It typically presents with the
acute onset of abdominal pain and obstipation. Patients will have abdominal
distention and bowel sounds that are characterized by high-pitched "rushes and
gurgles." The abdominal x-ray shows a dilated colon forming a loop, with the
narrowed segment pointing to the obstruction site. Nonoperative reduction
with a sigmoidoscopy with rectal tube placement is generally effective in
treating the condition. Individuals in nursing homes and mental institutions
have an increased risk for developing a sigmoid obstruction, however, the
reason is not yet established.
Sulfasalazine and corticosteroids, intravenously (choice A) are used in acute
management of ulcerative colitis (UC), which is a disease that typically affects
younger individuals. It is characterized by fever, bloody diarrhea, and
abdominal pain. A barium enema typically shows a loss of haustral markings,
which is generally referred to as a "lead pipe" pattern. Complications include
toxic megacolon, perforation, colon cancer, and hemorrhage. This patient is
not the typical UC patient, and the clinical findings are more consistent with a
sigmoid volvulus than UC.
Vancomycin and metronidazole (choice B) are used to treat
pseudomembranous colitis, which is due to Clostridium difficile. It typically
occurs during or after the use of antibiotics and presents with severe watery
diarrhea. Treatment of pseudomembranous colitis is vancomycin or
metronidazole, given orally, to reach high levels in the stool. Treatment is
usually with one or the other medications, not both at the same time. This
patient is not taking any medications and the clinical findings are more
consistent with a sigmoid volvulus than pseudomembranous colitis.
It is inappropriate to admit him and observe (choice C), because he most likely
has a sigmoid volvulus, which is a colonic obstruction that requires immediate
intervention to prevent strangulation, vascular compromise, and infarction.
If a patient with a sigmoid volvulus presents with generalized abdominal pain,
fever, and hypovolemia, immediate laparotomy (choice D), may be necessary
because strangulation has probably already occurred. This patient has stable
vital signs, which makes this unlikely.
A 61-year-old woman comes to the emergency department because she is
"lightheaded and dizzy" after having 2 bowel movements over the past hour
that consisted of bright red blood and no stool. She denies any abdominal pain
or nausea, but does recall having crampy abdominal discomfort after eating
over the last several days. She tells you that she has a history of "benign
polyps" that are resected endoscopically every other year in her
gastroenterologist's office. Her last colonoscopy was 6 months ago and 3
hyperplastic polyps were removed. Her mother and father both passed away
from complications due to colon cancer. Her temperature is 37.0 C (98.6 F),
blood pressure is 100/70 mm Hg, and her pulse is 110/min. Her abdomen is
non-tender and soft. There is no guarding or rebound tenderness present. There
is fresh red blood in the rectum, but there are no palpable masses. Intravenous
fluids are started. The most appropriate next step in management is to
A. order a barium enema
B. order a CT scan of the abdomen
C. order a nuclear bleeding scan of the colon
D. perform flexible sigmoidoscopy
E. perform upper gastrointestinal endoscopy
Explanation:
The correct answer is D. This patient has acute lower gastrointestinal
bleeding and after stabilization with intravenous fluids, requires visualization
of the colon. This should first be done with a flexible sigmoidoscope, which
will allow you to see the ano-rectum and determine if a lesion in this area is the
bleeding source. It will also allow you to see the sigmoid colon, which is the
most common site for a diverticular bleed, (one of the most common causes of
a lower GI bleed).
A barium enema (choice A) should not be done in this actively bleeding
patient at this time. It is often useful in patients with diverticulitis, after the
acute attack subsides.
A CT scan of the abdomen (choice B) is the diagnostic study to use if this
patient presented with left-sided abdominal pain, nausea, vomiting, fever, and
diarrhea, the symptoms of diverticulitis, not diverticulosis.
A nuclear bleeding scan of the colon (choice C) may be helpful in identifying
the exact site (but not the etiology) of the bleed; however it is often done after
a flexible sigmoidoscopy or colonoscopy.
An upper endoscopy (choice E) is unnecessary at this time because it seems as
if this patient has a lower GI bleed, not an upper GI bleed. A nasogastric tube
can be placed and if blood or coffee-ground material is found, then the source
is likely to be in the upper GI tract. However, in this case, bright red blood is
seen in the rectum (and yes there is a possibility that it comes from the upper
GI tract), however it seems like this "painless" bleed is lower GI in nature and
requires a flexible sigmoidoscopy at this time.
A 67-year-old woman with peripheral vascular disease, bilateral leg
claudication, and hypertension comes to the clinic because of nausea and
severe, diffuse abdominal pain that she rates as 7/10 in intensity for the past 2
days. The pain is related to meals, particularly lunch. She has smoked a pack of
cigarettes per day for the past 30 years. The patient has a temperature of 36.1
C/(97 F) with a pulse of 80/min and a blood pressure of 120/80 mm Hg.
Abdominal examination demonstrates normal bowel sounds, no tenderness,
and no hepatosplenomegaly. Laboratory studies reveal a leukocyte count of
4,000/mm3 and a hematocrit of 47%. You should be immediately suspicious of
A. acute appendicitis
B. acute cholecystitis
C. malingering
D. mesenteric ischemia
E. ulcerative colitis
Explanation:
The correct answer is D. Mesenteric ischemia , although uncommon, must
remain on the differential diagnosis of abdominal pain. The hallmark of
mesenteric ischemia is pain out of proportion to physical exam findings.
Mesenteric ischemia is especially likely in a patient with known vascular
disease and a history of cigarette smoking. The next diagnostic step is a
mesenteric angiogram. The superior mesenteric artery is the most often
compromised vessel.
Acute appendicitis (choice A) may present with atypical symptoms in the
elderly, but is usually present with a fever or elevated white blood cell count.
Appendicitis is uncommon in the elderly.
Acute cholecystitis (choice B) should present with right upper quadrant pain
and a positive Murphy's sign.
Malingering (choice C) should be considered on the differential diagnosis for
any patient complaint. It is, however, diagnoses of exclusion that must be
entertained only when an extensive diagnostic work up is completed and is not
suggestive of a disease process.
Ulcerative colitis (choice E) should present with diarrhea, constipation, heme
positive stools, and abdominal pain.
A 61-year-old man undergoes a right radical nephrectomy for renal-cell
carcinoma. The procedure is performed via a flank incision over the 12th rib.
The operation was contained within the retroperitoneum, and there was no
evidence that the peritoneal cavity was entered during the procedure.
Postoperatively, the patient's pain is managed with intravenous morphine
sulfate that is infused through a patient controlled analgesia (PCA) pump. The
patient's initial postoperative course is uneventful. He is out of bed and into a
chair on the first day, and within 48 hours of surgery he was ambulating with
assistance. Due to the painful nature of his flank incision, the patient utilizes a
large quantity of morphine after surgery. On postoperative day #5, you are
called to evaluate the patient after he vomited 250 cc of bilious material. Upon
questioning, the patient states that he has not passed any flatus, nor has he had
a bowel movement. He denies any chest pain or shortness of breath. His
temperature is 37.7 C (99.9 F), blood pressure is 137/62 mm Hg, pulse 87
/min, and respirations are 18/min. There are fine crackles over the right lung
base. Abdominal exam reveals a moderately distended abdomen, and an
incision that is clean and dry. Bowel sounds are hypoactive diffusely and
percussion displays tympanitic sounds. He has no calf tenderness. His
electrolytes are all within normal limits, however his hemoglobin and
hematocrit are 10.8 g/dL and 32%, respectively (preop 13.1g/dL and 39%). An
obstructive series shows dilated small and large bowel loops with air-fluid
levels within the small bowel. There is no free air appreciated. You should
advise this patient to
A. accept a blood transfusion
B. begin eating solid food
C. continue excessive use of his PCA pump
D. force himself to vomit every 6 hours
E. not eat anything
F. undergo a barium enema
Explanation:
The correct answer is E. This patient is suffering from a prolonged
postoperative ileus. An ileus is defined as the interval from the time of
operation, until the return of flatus and bowel movements. The expected time
frame for return of bowel function varies with the operation performed. In a
surgery such as nephrectomy, which is confined to the retroperitoneum, bowel
function should return within 36-48 hours. The absence of bowel sounds,
flatus, or bowel movements beyond the expected period indicates delayed
resolution or an adynamic ileus. This type of ileus usually responds to
nonoperative intervention. It frequently requires intestinal decompression with
the placement of a nasogastric tube. Diagnosis is made with a combination of
history, physical and radiographic studies if necessary. The etiology of
adynamic ileus is varied. Causes include neurogenic, metabolic,
pharmacologic, and infectious sources. It is important to "rule out" any of these
which might be suspected. This patient's ileus is most likely from his increased
use of narcotics. Conservative management by making the patient "NPO" and
stopping his PCA should improve his condition.
While the patient's blood level is decreased from preoperative levels, he does
not display any signs or symptoms of anemia (hypotension, tachycardia,
fatigue, light headedness). Giving him a blood transfusion (choice A) will not
alleviate the ileus and will expose him to the risks associated with blood
transfusion.
As previously stated, it is necessary to await the return of bowel function prior
to giving the patient food. Allowing him to eat now (choice B) will not help
his bowels. Instead it will only cause him more nausea and vomiting and
prolong his recovery.
Narcotic analgesics are a primary factor in the generation of postoperative
ileus. While narcotics are necessary in the initial postoperative course, their use
should be limited as soon as possible. Rather than encourage this patient to
continue with his PCA pump (choice C) you should stop his narcotic
analgesics completely.
You should not encourage the patient to vomit (choice D). Rather, if his ileus
is causing him to vomit continuously, then a nasogastric tube should be placed
for gastric decompression.
A barium enema (choice F) will not provide any additional information at this
time. It does not play a role in the management of a postoperative ileus.
A 28-year-old man comes to the emergency department with a 2-day history of
worsening abdominal pain and a lack of desire to eat. He was diagnosed as
having leukemia 4 months ago, for which he received chemotherapy
successfully. For the past 2 months he has been asymptomatic and able to
tolerate a regular diet. In the emergency department, he is noted to pass bloody
mucoid stools and hematuria. His temperature is 37.8 C (100 F), blood
pressure is 120/70 mm Hg, and pulse is 92/min. Abdominal examination shows
a diffusely tender abdomen with voluntary guarding and no peritoneal signs.
Laboratory studies show a total leukocyte count of 500/mm3 and a platelet
count of 15,000/mm3. A subsequent CT scan of the abdomen and pelvis
reveals thickening of the sigmoid colon with minimal infiltration of the
surrounding fat. The most appropriate management of this patient is
A. antibiotics
B. antibiotics, bowel rest, platelet transfusion, and granulocyte stimulating
factor
C. antibiotics, colonoscopy
D. colonoscopy
E. granulocyte stimulating factor, antibiotics, and colonoscopy
Explanation:
The correct answer is B. This patient was recently treated with chemotherapy
for leukemia and has developed a neutropenic enterocolitis. Neutropenic
enterocolitis is usually segmental, often localized to the sigmoid colon. This
can be diagnosed on a CT scan of the abdomen and pelvis by means of a
thickened sigmoid colon and infiltration of the surrounding fat. Without being
treated with antibiotics and bowel rest, this neutropenic enterocolitis can
progress to worsening of the symptoms and signs leading to complications of
systemic septicemia and hypotension. Hence, this patient should be treated
with bowel rest, appropriate antibiotics, and platelet transfusion to increase his
platelet counts and decrease the chance of hematuria. Granulocyte stimulating
factor may help to increase his neutrophil count.
Antibiotics (choice A) alone may not resolve colitis completely without the
correction of neutropenia.
Colonoscopy (choices C and D) is contraindicated as the patient may develop
complications from colonoscopy of an inflamed colon.
Granulocyte stimulating factor (choice E) is indicated to increase the
neutrophil count but colonoscopy is contraindicated.
A 60-year-old man with diabetes and hypertension comes to the clinic because
his wife is worried that his skin is turning yellow. The patient's wife reports
that she first noticed the skin changes about 1 month ago and now she says
"even his eyes look bright yellow!" He drinks a case of beer a week and
smokes 2-3 packs of cigarettes a week. He says he has been feeling well and
denies abdominal pain, nausea, or vomiting. Vital signs are normal. He is a
thin male and the abdominal examination is normal. Laboratory studies show:
The most appropriate test at this time is
A. CT of abdomen and pelvis
B. endoscopic retrograde cholangiopancreatography (ERCP)
C. mesenteric angiography
D. serum CA19-9
E. upper gastrointestinal barium study
Explanation:
The correct answer is A. The presentation of painless jaundice is highly
suspicious for a pancreatic head mass and in particular adenocarcinoma of the
pancreas. Adenocarcinoma of the pancreas accounts for more than 90% of
pancreatic malignancies and jaundice is present in about 65% of patients. Risk
factors for pancreatic adenocarcinoma include smoking and diabetes. The best
initial evaluation for pancreatic masses is by CT of the abdomen and pelvis.
Endoscopic retrograde cholangiopancreatography (ERCP) (choice B) is not the
best initial test to evaluate for pancreatic masses because it primarily evaluates
the biliary duct system in the liver and pancreas. It will not define a mass that
does not involve the biliary ducts. An ERCP may show a discrete stricture in
the main pancreatic duct with proximal dilatation.
A mesenteric angiography (choice C) is not the best initial test to evaluate for
pancreatic masses because it primarily evaluates the vascular structures that
supply and drain the pancreas and abdominal organs. Angiography can be
useful preoperatively because it may show displacement or encasement of the
pancreatic or duodenal arteries by a mass. The venous phase is also useful if
the superior mesenteric vein or splenic vein is occluded due to tumor
extension.
Serum CA19-9 (choice D) is not the best initial test to evaluate for pancreatic
adenocarcinoma. It is a tumor marker that has been associated with
adenocarcinoma of the pancreas. It is not useful as a screening test, but it has a
high sensitivity and specificity as a marker for recurrent disease or metastases
after the primary pancreatic tumor is resected.
An upper gastrointestinal barium study (choice E) is not the best initial test to
evaluate for pancreatic masses. It is a useful study to evaluate for mucosal
lesions within the esophagus, stomach, and small bowel. In the setting of
pancreatic malignancy or other masses, the upper gastrointestinal barium study
may show a widened loop or an "inverted 3 sign" caused by abnormal
indentation of the pancreas along the medial aspect of the duodenum.
A 61-year old patient with a history of a duodenal ulcer, is admitted to the
hospital for a scheduled right knee arthroplasty because of end-stage
osteoarthritis. After an uneventful surgery and immediate postoperative course,
the patient has an episode of 100 cc of bloody emesis. Vital signs are:
temperature 37C (98.6 F), blood pressure 140/75 mm Hg, pulse 70/min, and
respirations 10/min. Oxygen saturation measurement obtained while the patient
is receiving supplemental oxygen of 2 L/min via nasal cannula is 99%.
Physical examination is unremarkable. The hematocrit profile is:
After sending blood for type and cross match, an upper endoscopy is
performed and fails to identify the source of bleeding. The most appropriate
next step in the management of this patient is
A. an angiography
B. an endoscopy of lower gastrointestinal tract
C. an enteroclysis
D. a tagged red blood cell bleeding scan
E. a transfusion of packed red blood cells
Explanation:
The correct answer is D. A tagged red blood cell scan is advantageous over
angiography in a stable patient. It offers improved detection of bleeding and
localization, as the bleeding is likely to be intermittent and slow given the
patient's normal range in hematocrit. A bleeding scan can detect bleeding rates
as low as 0.05 mL/minute versus 0.5 mL/minute for angiography.
An angiography (choice A) is only useful in patients with rapid bleeding.
These patients may be hemodynamically unstable and are generally
tachycardic and hypotensive. An angiography, like an endoscopy, can be
diagnostic as well as used to perform therapeutic measures.
Endoscopy of the lower gastrointestinal tract (choice B) would look at the
colon. The symptoms of bloody emesis are consistent with an upper
gastrointestinal source of bleeding.
Enteroclysis (choice C) is also known as a small bowel enema and is used to
evaluate the small bowel for causes of bleeding such as polyps. Although a
small bowel lesion is possible in this patient, peptic ulcer disease is much more
common and must be the first consideration.
Blood transfusion (choice E) should not proceed until there is a significant
drop in hematocrit below the normal limits. In the meantime, the patient should
be typed and crossed for possible transfusion, should massive bleeding begin.
You are evaluating a 72-year-old man brought to the emergency department in
an altered mental status. He lives alone, but his neighbors informed paramedics
that the patient was complaining of abdominal discomfort earlier in the day.
Paramedics noted that there is evidence of vomiting at the bedside at the
patient's home. The patient is wearing a wristband identifying him as an
insulin-dependent diabetic. His temperature is 38.3 C (101 F), blood pressure
is 106/60 mm Hg, pulse is 86/min, and oxygen saturation is 94% on room air.
He is moaning in pain and is flexing his hips. His mucous membranes appear
dry, and chest auscultation reveals coarse crepitations. Abdominal examination
shows a distended, tender abdomen with guarding and hyperactive bowel
sounds. His extremities are warm with diminished distal pulses. During the
physical examination, particular attention should be given to his
A. chest
B. groin
C. hips
D. neurological status
E. peripheral vasculature
Explanation:
The correct answer is B. Hernial sites must be examined to complete any
acute abdomen evaluation. Incarcerated or strangulated hernias in an elderly
patient can cause acute abdomen, dehydration, and altered mental status.
Failure to diagnose an incarcerated hernia at an early stage can lead to
strangulation.
An incarcerated hernia causing bowel obstruction leads to nausea and vomiting
and may result in aspiration to lungs. Coarse crepitations on auscultation
(choice A) may be the result of aspiration and to prevent further aspiration, it is
important to diagnose and treat the incarcerated hernia.
Hip fractures are common in the elderly. This patient's lower extremity is in an
abducted position and shortened. Flexed hips might be due to an incarcerated
hernia in this patient (choice C).
Altered mental status in an elderly patient might be due to widely spread
reasons including metabolic (diabetes, dehydration), respiratory (pneumonia),
neurological (intracranial bleeding), abdominal, or due to ischemia from lower
extremities in advanced stages. In this patient with abdominal signs, while his
neurologic status (choice D) and peripheral vasculature (choice E) may be
abnormal, with his symptoms and physical findings, an incarcerated hernia
needs to be ruled out, and therefore particular attention should be given to his
groin.
A 52-year-old man, who recently transferred to the city, comes to the office for
a first visit complaining of weight loss and frequent malodorous stools. He
admits to abusing alcohol and reports that he has developed chronic
pancreatitis from his alcoholism. For the past year, he has given up alcohol
completely and has transferred his job to your city. He reports that he has lost
20 pounds over the past 3 months. In addition, he reports frequent, greasy,
malodorous stools. He denies any recent alcohol consumption and any
abdominal pain. Laboratory studies and a CT scan of the abdomen confirm the
diagnosis of chronic pancreatitis. A 72-hour fecal fat collection confirms
steatorrhea. The most appropriate treatment for this patient is
A. endoscopic placement of pancreatic duct stent
B. an enteric-coated pancreatic enzyme replacement tablet with meals and
calcium-containing antacids
C. a low-fat diet
D. a non-enteric coated pancreatic enzyme replacement with H2 blockers
E. octreotide
Explanation:
The correct answer is D. Frequent, greasy, malodorous stools are a result of
steatorrhea from chronic pancreatitis. This happens from the lack of pancreatic
enzymes. Non-enteric coated pancreatic enzyme supplements with concurrent
H2 blockers will deliver active enzymes to the proximal small bowel and help
reduce malabsorption and steatorrhea.
Enteric-coated enzyme preparations are effective in the treatment of
steatorrhea but are more expensive (choice B). When combined with calcium
containing antacids, this can worsen steatorrhea by complexing with fatty acids
and thus making absorption more difficult.
Endoscopic placement of a pancreatic duct stent (choice A) may relieve pain in
patients with a dilated pancreatic duct, but would not help in treating
steatorrhea.
A low-fat diet can worsen malnutrition (choice C). A normal allowance of fat
intake will result in greater patient compliance.
Octreotide (choice E) is an effective agent for the treatment of pain in
pancreatitis, but would not help malabsorption from steatorrhea.
A 56-year-old man is admitted to the hospital because of a 1-day history of
acute, severe, cramping abdominal pain that radiated to his back. The pain was
constant and exacerbated when he tried to eat some food. The patient
attempted to self medicate with acetaminophen, but with no relief. The pain
has slowly worsened and he has not had anything to eat or drink in over a day.
On admission to the hospital, his serum amylase and lipase levels are elevated.
The appropriate therapy is initiated and the patient has improvement in his
pain. He is also started on a morphine patient-controlled anesthetic (PCA) with
excellent results. Over the next 24 hours, he remains stable. A follow-up set of
blood chemistries shows a BUN of 26 mg/dL and a creatinine of 1.0 mg/dL
with an unchanged amylase and lipase. A right upper quadrant ultrasound
shows gallstones with no ductal dilation. The patient's other medications,
besides the PCA, are diazepam for sleep and diphenhydramine. The most
appropriate next step is to
A. arrange for endoscopic retrograde cholangiopancreatography
B. arrange for hepatobiliary iminodiacetic acid scan
C. arrange for laparoscopic cholecystectomy
D. arrange for an open cholecystectomy
E. continue intravenous hydration and nil per os status
Explanation:
The correct answer is E. The correct therapy for pancreatitis is hydration,
avoiding oral intake, and pain control. Given the success of this therapy for this
patient over the previous 24 hours, it should continue.
Some centers will arrange for an ERCP (choice A) within 2 days of a
pancreatitis episode to determine if papillotomy may be beneficial in
abrogating the course of the disease. There is some clinical data for this but it
is not yet common practice.
A HIDA scan (choice B) is used to diagnose cholecystitis, it is not indicated
given that the abdominal ultrasound did not show evidence of this entity
(gallbladder wall thickening, sludge).
Arranging for a laparoscopic cholecystectomy (choice C) or open
cholecystectomy (choice D) will eventually be necessary given the pancreatitis
and the presence of gallstones. However, this is not to be done presently and is
generally done after the pancreatitis episode has passed (6-8 weeks).
A 48-year-old man comes to the office complaining of progressively
worsening heartburn and dysphagia over the past 3 months. The heartburn is
intermittent and is progressively getting worse and, unfortunately, no particular
position while lying bed is giving him any relief. He has no other past medical
history or surgical history. Physical examination is unremarkable. You
recommend behavior and lifestyle modification and he convinces you to
prescribe medications. He returns a few months later because the symptoms
have not improved. At this time you should order a(n)
A. acid infusion test
B. endoscopy and biopsy
C. esophageal manometry
D. 24-hour pH monitoring
E. upper gastrointestinal series
Explanation:
The correct answer is D. The single best test for establishing and quantifying
the amount of reflux is a 24-hour pH monitoring test. The pH probe is placed
5-cm proximal to the lower esophageal sphincter as determined by manometry.
The probe is connected to a computer and the data is recorded continuously at
1-second intervals throughout the 24-hour period. The number of reflux
episodes with a pH below 4, the total time the pH is below 4, and the longest
duration of the pH below 4 in the lower esophagus, is analyzed from this data.
Also, the number of reflux episodes longer than 5 minutes is analyzed from
this data. This data is used to interpret the presence of gastroesophageal reflux
disease and to quantify its severity. Patients with a pH of less than 4 more than
9% of the time are highly likely to benefit from an antireflux procedure. Under
normal circumstances, no reflux episode should last longer than 5 minutes.
An acid infusion test, (Bernstein test), is an older test (choice A), where saline
or one-tenth normal hydrochloric acid is infused into the lower esophagus to
see whether the symptoms were reproduced. This test is no longer performed.
An endoscopy and biopsy (choice B) are important to detect Barrett's
esophagus, but not essential in the detection of gastroesophageal reflux.
Esophageal manometry (choice C) is helpful prior to the correction of
esophageal reflux because it quantifies peristalsis within the body of the
esophagus and quantifies the strength, length, and location of the lower
esophageal sphincters. This is not essential as a first step in the diagnosis of
gastroesophageal reflux, although, it is essential before surgical correction.
Upper gastrointestinal series (choice E) is useful to demonstrate a hiatus
hernia, paraesophageal hernia, or an esophageal stricture associated with reflux
disease. A substantial number of patients who don't actually have clinically
significant reflux will have reflux on the upper gastrointestinal series and
hence, this test is not very specific.
A 37-year-old woman comes to the emergency department because of a 30minute history of vomiting reddish-brown material. She informs you that she
suffers from fibromyalgia syndrome and uses a number of "pain killers" to
control her pain. Her blood pressure is 120/70 mm Hg and pulse is 110/min, no
orthostasis. Physical examination is unremarkable. Her extremities are cool
and her capillary refill is less than 2 seconds. A nasogastric tube is passed and
returns 200 cc of coffee ground material that eventually clears with normal
saline lavage. The patient is then sent for endoscopy. The most likely cause of
this patient's gastrointestinal bleeding is
A. esophageal varices
B. esophagitis
C. gastric neoplasm
D. gastric ulcers
E. Mallory Weiss tears
Explanation:
The correct answer is D. The most common causes of upper gastrointestinal
(UGI) bleeds are peptic ulcer disease (PUD) (45-50%), gastritis (30%), varices
(10%), and then the remainder of causes are due to other disorders such as
Mallory-Weiss tears, esophagitis, and neoplasms. In any patient with a history
of "pain killer" use, especially females with rheumatological conditions, the
diagnosis of gastritis or gastric ulcers secondary to nonsteroidal antiinflammatory drug (NSAID) use must be suspected. The most common cause
of these two conditions is NSAID use.
Esophageal varices (choice A) are a very common cause of UGI bleeds in
patients with cirrhosis. In the United States, the most common cause of
cirrhosis is alcohol and hepatitis virus infection. Worldwide, Schistosomiasis is
the most common cause. Since this patient has none of the above diseases, the
likelihood of her having varices is almost zero.
Esophagitis (choice B) is usually due to acid reflux diseases and is not a
significant cause of GI bleeding. It may lead to a premalignant condition
(Barrett esophagus).
Gastric neoplasm (choice C), although accounting for a small percentage of
UGI bleed patients, requires other associated findings of cancer in order to be
suspected. Gastric cancer in particular is associated with early satiety,
epigastric pain, palpable abdominal mass, and certain nitrate containing foods.
Mallory-Weiss tears (choice E) are small esophageal tears induced by
vomiting. It should be suspected in patients who have the triad of hematemesis,
alcohol abuse, and vomiting. It is not a cause of severe or prolonged or
recurrent UGI bleeding.
A 48 year-old man with hypertension and cirrhosis is brought to the emergency
department by his wife because of hematemesis. This morning he woke up
feeling nauseated and vomited "coffee ground" looking material. He then ate
his breakfast and afterwards, vomited bright red blood. His medications
include atenolol, ranitidine, and folate. The most appropriate next step in
evaluating this patient is to
A. assess airway and respiratory status
B. give him an emergent blood transfusion
C. insert a nasogastric tube
D. perform upper endoscopy and sclerotherapy
E. prepare him for immediate laparotomy
Explanation:
The correct answer is A. For all patients that present in an emergency
situation, the same basic approach should be applied. This approach, the ABCs
(airway, breathing, and circulation) are a reminder that these basics must
always be assessed first; the so-called primary survey. Once this is
accomplished, more detailed interventions can be taken based upon additional
findings. A patient that has vomited blood must have their airway and
breathing assessed to ensure that there is no compromise that is lifethreatening.
Emergent blood transfusion (choice B) may be appropriate after laboratory
data are obtained that show a low hematocrit or if there are clinical signs
suggestive of profound hypovolemia or shock.
Insertion of a nasogastric tube (choice C) may be appropriate, and is in fact
likely to be appropriate, after the initial primary survey is completed. This
allows for decompression of any remaining blood in the stomach and serves to
evacuate any ongoing losses to prevent additional episodes of emesis.
An upper endoscopy and sclerotherapy (choice D) will likely be the treatment
option of choice once the patient is stable and fully evaluated. Most common
causes of upper GI bleeding are amenable to diagnosis and treatment by upper
endoscopy.
Immediate laparotomy (choice E) may be indicated if the source of the
bleeding can only be corrected surgically. These procedures are however NOT
diagnostic.
A 4-week-old boy is brought to your office by his mother because he has had
increasing amounts of vomiting over the past week. Initially it started as
"spitting up" after a few meals a day, but now the baby is having projectile
vomiting after every meal. She says the vomitus is non-bloody and non-bilious
and the baby appears hungry after he vomits. This is her first child and she is
not sure if this is normal. Physical examination is unremarkable. Laboratory
studies show:
At this time the most appropriate next step is to
A. do nothing
B. obtain an abdominal radiograph
C. obtain an abdominal ultrasound
D. order a barium enema
E. order a CT scan of the abdomen
F. order a upper gastrointestinal series with small bowel follow through
Explanation:
The correct answer is C. This baby has presented with the typical picture of
pyloric stenosis. Projectile non-bilious vomiting is seen in virtually all patients.
Patients also often develop a hypokalemic, hypochloremic metabolic alkalosis
from the persistent vomiting. On exam, the upper abdomen may be distended
after feeding and prominent peristaltic waves from left to right may be seen.
The classic physical exam finding of an "olive-shaped" mass in the epigastric
area is not always felt. An ultrasound is the least invasive, safest diagnostic
test. The typical appearance on ultrasound is a hypoechoic ring with a
hyperdense center. The pyloric muscle thickness measures greater than 4mm.
Since this patient has the symptoms of pyloric stenosis, it is inappropriate to do
nothing (choice A).
A plain film of the abdomen (choice B) will most likely appear normal unless
there is a complete obstruction of the pyloric outlet. Again, this study would
expose the baby to unnecessary radiation. And, in this case, it would not help
you to establish the diagnosis.
A barium enema (choice D) will not help to make the diagnosis in this patient.
The fact that the vomitus is non-bilious puts the point of obstruction prior to
the entrance of the sphincter of Oddi. A study performed from below will not
help to see what is happening so high in the intestinal tract. If you suspected
intussusception (paroxysmal abdominal pain, vomiting, diarrhea, hemepositive stool, and sausage-shaped mass in upper mid abdomen on palpation),
then a barium enema (or air enema) would be both diagnostic and therapeutic.
An abdominal CT scan (choice E) is also not helpful in this case. A good study
may show the thickening of the pylorus muscle (depending on the cut), but an
ultrasound is a much quicker and cheaper study with no radiation exposure.
In a baby with pyloric stenosis, an upper gastrointestinal series (UGI) with
small bowel follow through (choice F) will show a delay in gastric emptying
and an elongated narrowed pyloric channel. Although a diagnosis of pyloric
stenosis can be made with an UGI series, this study exposes the baby to
unnecessary radiation. If you suspected a malrotation ("cyclic vomiting" with
bilious emesis), an UGI with small bowel follow-through is the study of
choice.
A 3-year-old boy is brought to the emergency department by his parents
because of a 24-hour history of intermittent, generalized abdominal pain. The
parents tell you that he complains of the pain for 10-minute episodes and
during these times he refuses to walk, but then he spontaneously returns to his
normal activities. This occurred 8-9 times yesterday. Today the symptoms
occurred more frequently and were associated with 3 episodes of non-bloody,
non-billous emesis so the parents brought him into the hospital. There is no
history of fever, constipation, or soiling. On examination the patient appears
tired and has mild diffuse abdominal pain. He has guaiac-positive stool. His
pulse is 125/min. The study most likely to provide a diagnosis is
A. an abdominal x-ray
B. a barium enema
C. a CBC with differential
D. a CT scan of the abdomen
E. a lumbar puncture
Explanation:
The correct answer is B. This patient presents with a very common complaint
in the pediatric population, abdominal pain. The key to this case is the quality
and frequency of this abdominal pain. The pain was described as being diffuse
and intermittent with periods of resolution of the symptoms. This type of pain
pattern, along with emesis, the lethargy seen in the emergency department, and
the guaiac-positive stools should raise red flags for the diagnosis of
intussusception. In intussusception a segment of bowel (most commonly the
distal ileum into the cecum) telescopes into an adjacent segment causing
obstruction. This obstruction tends to resolve and recur causing the intermittent
abdominal pain. The barium enema is diagnostic and in many cases a curative
procedure as well and is therefore the study of choice in this case.
In a case of diffuse abdominal pain an abdominal x-ray (choice A) could show
a colon full of stool and aid in a diagnosis of constipation causing the pain.
That history was not seen in this case. In intussusception an abdominal x-ray
might show a paucity of air in the area of the intussusception but it will most
likely be inconclusive.
In intussusception, a CBC (choice C) might show us a leukocytosis, which
could also be present in an infectious cause of the abdominal pain or in
appendicitis, and therefore would not assist in providing a definitive diagnosis.
An abdominal CT scan (choice D) would be indicated if there was a higher
index of suspicion for appendicitis, but it is not indicated as a study for
suspected intussusception.
If this patient were febrile along with the lethargy, a lumbar puncture (choice
E) might be warranted to rule out meningitis as a cause for the lethargy. In this
case with the history as given, the spinal tap is not indicated.
A 50-year-old woman comes to the emergency department complaining of
abdominal pain that is constant and radiating to the left upper quadrant. She
has a history of hypertension, depression, and pancreatitis. Her current
medications include furosemide and paroxetine. She denies alcohol, tobacco,
or drug use. Her temperature is 38.1 C (101.6 F), blood pressure is 105/78 mm
Hg, pulse is 102/min, and respirations are 23/min. Her weight is 137kg (302
lb) and she is 136cm (5ft 2in) tall. She has pain to palpation in the epigastrium,
no rebound tenderness, and her rectal examination is guaiac negative. Her
breath sounds are clear and her cardiac rhythm is regular. The most appropriate
laboratory study at this time is
A. serum albumin
B. serum 5' nucleotidase
C. serum ionized calcium
D. serum lipase
E. serum total bilirubin
Explanation:
The correct answer is D. Both the serum amylase and lipase become elevated
acutely with pancreatitis. Since amylase can also be elevated with other
pathologies, a serum lipase should always be evaluated in a patient with
suspected pancreatitis.
A serum albumin (choice A), while providing information on the patient's
nutritional status and hepatic synthetic ability, does not help in the diagnosis of
pancreatitis.
A serum 5' nucleotidase (choice B), which is used to evaluate whether an
elevated alkaline phosphatase is of hepatic origin, does not assist in the
diagnosis of pancreatitis.
A serum ionized calcium (choice C), which is often abnormally low in the
setting of pancreatitis, does not help in the diagnosis of the condition.
A serum total bilirubin (choice E) does not assist in the diagnosis of
pancreatitis.
A 39-year-old man comes to the emergency department because of severe
abdominal pain for the past 12 hours. He says that the pain came on suddenly,
and woke him from sleep. He says that over the past few months, he has had a
few episodes of burning abdominal pain that radiates to his back. The previous
episodes of pain have been relieved by food, but he vomited when he tried to
eat before coming to the hospital. The pain was "excruciating" on the cab ride
over. His blood pressure is 100/70 mm Hg, pulse is 90/min, and respirations
are 28/min. His abdomen appears flat, rigid, and "board-like". The most
appropriate next step in evaluating his abdominal pain is
A. auscultation of the abdomen
B. deep, slow palpation with quick release
C. light palpation at point of maximal pain
D. percussion at point of maximal pain
E. shifting dullness of the abdomen
Explanation:
The correct answer is A. This patient has the signs and symptoms suggestive
of a perforated duodenal ulcer, and after inspection of the abdomen comes
auscultation of the abdomen. Soon after perforation, the bowel sounds are
typically hypoactive and later on they are most likely absent. Many physicians
feel that auscultation should be performed before percussion or palpation so as
not to disrupt the intestinal motility. This way, the physician may be able to
obtain a more accurate assessment of bowel sounds.
Deep, slow palpation with quick release (choice B), is the technique to test for
rebound tenderness, which is most likely present in a patient with a perforated
duodenal ulcer, and an acute abdomen. However, this part of the examination
will cause severe pain in a patient with peritoneal inflammation, and it is
therefore best to do this last, after inspection, auscultation, percussion, and
light palpation.
Light palpation at point of maximal pain (choice C), or percussion at point of
maximal pain (choice D), should not be the next step in evaluating his
abdominal pain. The order of the abdominal examination is typically
inspection, auscultation, percussion, light palpation, deep palpation, and lastly,
to rule out rebound tenderness. Some people prefer to palpate before
percussing. However inspection and auscultation should still be before these 2
techniques. Also, percussion and palpation should always begin in the area
furthest from the location of the pain. This is to allow the patient to feel
comfortable for the most amount of time during the exam and for the examiner
to be able to obtain the most information, (they may be in too much pain for
you to complete the examination if you begin with palpation at the point of
maximal pain).
Testing for shifting dullness (choice E), is usually done in patients with
suspected ascites, and is part of the percussion examination. It is used to
determine if ascites is present, and it is performed by determining the borders
of dullness and tympany first with the patient in the supine position. Then
patients are asked to turn on their side, and borders are determined again by
percussion. If ascites is present, there will be dullness in the dependant
location, (due to fluid) and tympany, (due to gas in the bowel) in the area of
the abdomen furthest from the table. These dull and tympanic areas will "shift"
or change location as the patient lies in different positions. The patient in this
case has a flat, board-like abdomen, and does not seem to have ascites. Even if
he did have ascites, this test should be performed as part of the percussion
examination, after inspection, auscultation, and maybe palpation.
A 62-year-old man comes to the emergency department after passing
approximately 500 milliliters of bright red blood per rectum in the toilet 2
hours earlier. His past medical history is significant for very severe
emphysema requiring two partial lobectomies. The bleeding episode was
painless and stopped spontaneously. His blood pressure is 110/80 mm Hg,
pulse is 70/min, and respirations are 16/min. Laboratory studies show:
An abdominal CT scan reveals multiple diverticula in the descending colon.
He is admitted for observation overnight and 2 units of packed red blood cells
are transfused. Around midnight, the patient has another episode of bleeding.
At this time his heat rate is 120/min and his blood pressure is 88/54 mm Hg . A
99
Tc-labeled red blood cell scan reveals active bleeding in the sigmoid colon
and the bleeding source is subsequently determined by angiography. Given the
patient's presentation, the best intervention option at this time is
A. colonoscopy and sclerosis of the bleeding site
B. intraarterial embolization
C. intraarterial vasopressin infusion
D. medical management and supportive care
E. partial colectomy and temporary colostomy
Explanation:
The correct answer is C. This patient is suffering from a significant
diverticular bleed. His initial bleed was self-limited and his vital signs were
stable. An abdominal CT scan confirmed the presence of diverticular disease.
At that time, supportive therapy with blood transfusions was initiated. During
his second bleeding episode, the patient had become tachycardic with a lower
blood pressure suggesting worsening hemodynamics and significant blood
loss. Imaging with a tagged red blood cell scan confirmed active bleeding and
the general site. Angiography of the mesenteric blood supply located the site of
bleeding. The appropriate intervention at this time would be a directed
vasopressin infusion into the arterial mesenteric circulation to constrict the
bleeding vessel. Because the patient has significant pulmonary disease and
multiple lung resections, he is not a good candidate for surgical intervention.
Vasopressin infusion is less invasive, and if it is administered at the time of
angiography, systemic side effects can be limited. This intervention is
relatively contraindicated in patients with significant coronary artery disease
due to risk of ischemia or infarction.
Colonoscopy and sclerotherapy (choice A) of a diverticular bleed are difficult
due to typically poor visualization of the bleeding source from an incomplete
bowel prep. In addition, the patient must generally be hemodynamically stable.
Like vasopressin therapy, arterial embolization (choice B) of the bleeding
source can be done at the time of angiography. However, embolization carries
a higher risk of bowel infarction or perforation than vasopressin infusion in
colonic bleeds, so it is mostly used in upper gastrointestinal bleeds.
Embolization can be considered an alternative intervention if a vasopressin
infusion fails.
Supportive care and observation (choice D) is typical management for a patient
with a first-time lower gastrointestinal bleed, since these bleeds are usually
self-limited. However, this patient continued to experience bleeding episodes
and became hemodynamically unstable. Therefore he required further
measures to prevent continued bleeding and volume loss.
This patient suffers from severe respiratory disease and would not be a good
surgical candidate. Unless other interventions have failed, a partial colectomy
and temporary colostomy(choice E) should not be done in this patient.
A 91-year-old woman with hypertension comes to the clinic complaining of
constipation for the past 2 months. She had a hysterectomy 10 years ago and
surgery for pelvic floor prolapse 6 months ago. There is hard stool in the vault
on rectal examination. The stool is not grossly bloody but is heme positive.
Laboratory tests reveal a hematocrit of 29% with a reticulocyte distribution
width (RDW) of 33% and a carcinoembryonic antigen (CEA) of 18 ng/ml. The
first test necessary to further evaluate this patient is a(n)
A. colonoscopy
B. colposcopy
C. reticulocyte count
D. ultrasound of the right upper quadrant
E. Westergren sedimentation rate (ESR)
Explanation:
The correct answer is A. Heme positive stools and signs of colonic
obstruction are common in left sided colon cancer. The elevated CEA supports
the diagnosis of colon cancer. The next step is a colonoscopy to biopsy the
lesion. A barium enema may also demonstrate the mass but will not allow for
tissue diagnosis.
Colposcopy (choice B) utilizes a microscope to better visualize the cervix and
take biopsies in women with a positive Pap smear. There is no indication for
this procedure in this patient.
This patient most likely has anemia secondary to gastrointestinal blood loss as
demonstrated by the elevated RDW. If a colon mass is found, further work up
for this mild anemia (choice C) is not warranted.
Right upper quadrant ultrasound (choice D) is not used to stage colon cancer.
A CT scan of the abdomen can be used to evaluate the local stage of the
disease as well as assess for liver metastasis. Liver function tests should
precede CT. Statistically, colon cancer is much more likely than a primary
hepatocellular carcinoma or cholangiocarcinoma given the symptoms of
constipation with heme positive stools and an elevated CEA. Colonoscopy
should be performed first given the symptoms of constipation and the high
likelihood of colon cancer.
ESR (choice E) is a nonspecific marker of inflammation and may be elevated
in many situations, including cancer. It is of little clinical value in this patient.
A 28-year-old white woman comes to the office with an 8-month history of
weight loss, fatigue, and diarrhea. She states that she has stools approximately
6 times per day and there is blood present in the majority of them. She denies
any personal or family history of previous gastrointestinal problems. Her
temperature is 37.2 C (99.1 F), blood pressure is 120/80 mm Hg, and pulse is
110/min. Physical examination reveals present normoactive bowel sounds. Her
abdomen is soft with diffuse tenderness without rebound or guarding. An
office sigmoidoscopy reveals friable mucosa with multiple bleeding points and
no areas of normal mucosa. A colonic mucosal biopsy is likely to show
A. cobble stoning
B. skip lesions
C. superficial mucosal inflammation
D. transmural inflammation
E. villous atrophy
Explanation:
The correct answer is C. The clinical case describes ulcerative colitis, which
is an inflammatory bowel disease that is characterized by weight loss, bloody
stools, abdominal pain, and tenesmus. Sigmoidoscopy reveals mucosal
inflammation that is continuous from a rectum proximal colon. A biopsy
would most likely show superficial mucosal inflammation and ulceration.
Crohns disease is another type of inflammatory bowel disease that affects the
entire thickness and will show transmural inflammation(choice D) of the wall
of the colon. Cobble stoning (choice A) describes the gross appearance of the
intestine in CD. CD need may occur anywhere from mouth to anus and usually
has skip lesions (choice B) between areas of the normal intestine.
Villous atrophy (choice E) may be found in celiac sprue and malabsorption. It
is not a typical feature of Crohns disease.
A 79-year-old man is admitted to the hospital for a gangrenous right foot. He
has a long history of peripheral vascular disease, hypertension,
hypercholesterolemia, coronary artery disease, and has suffered 2 strokes. The
patient's daughter visited him at home today and noticed his foot was black.
The patient is admitted to the hospital for a right, below-the-knee amputation.
Over the next 48 hours the patient complains of increasing abdominal pain. His
temperature is 39.8 C (103.6 F), blood pressure is 100/50 mm Hg, pulse is
120/min, and respirations are 22/min. Physical examination shows a diffusely
tender and distended abdomen and his right foot is unchanged. Stat blood work
is drawn and shows:
The diagnostic procedure most likely to establish the diagnosis is
A. abdominal radiographs flat and upright
B. chest films flat and upright
C. CT scan of the abdomen
D. exploratory laparotomy
E. ultrasound of the abdomen
Explanation:
The correct answer is D. This patient has an ongoing abdominal catastrophe,
likely an ischemic bowel or bowel perforation. This is a typical course for such
a patient, an indolent, smoldering entity to an alarmingly overt one in just a
few short hours. Surgical exploration of the abdomen with possible total
colectomy or small bowel resection is life-saving in such cases.
Abdominal radiographs flat and upright (choice A) or chest films flat and
upright (choice B) are useful for the diagnosis of free air and perhaps
obstruction, but are not very useful in the diagnosis of early ischemic bowel.
Once the bowel becomes necrotic with wall thickening, then radiographs may
have more utility. Once this finding is present however, the patient will have
deteriorated so severely that death would most certainly have occurred or will
imminently occur.
A CT scan of the abdomen (choice C) is very helpful in these cases and is
perfectly acceptable in cases where surgical exploration is not an option. The
gold standard for diagnosis is direct visualization. The CT can offer
radiographic suggestion of the disease but cannot, to the same degree as
laparotomy, confirm the diagnosis. In fact, once a CT scan suggests it, patients
are then scheduled for laparotomy for exploration.
An ultrasound of the abdomen (choice E) has no utility in these situations.
This modality is very useful for biliary and pelvic diagnoses, but not colonic
on bowel ones.
A 39-year-old man comes to the office because of "gnawing" abdominal pain
and diarrhea for the past 2 months. He states that the pain is worst about 3
hours after a meal and it often wakes him at night. He says, "surprisingly, the
pain is relieved by food." He takes a nonsteroidal antiinflammatory drug every
couple of weeks for a headache or back ache, does not smoke cigarettes, and
has a couple of glasses of wine on the weekends. He vaguely recalls that his
father and brother have had similar symptoms in the past. Physical
examination shows epigastric tenderness, midway between the xiphoid process
and the umbilicus. There is no rebound tenderness. You prescribe amoxicillin,
bismuth, and metronidazole, and tell him to return in 2 months. He returns for
his follow-up appointment and says that his diarrhea is still present and that the
abdominal pain has not decreased in intensity or quality. Physical examination
is unchanged. Laboratory studies show:
At this time the most appropriate management is to
A. measure serum gastrin levels
B. measure serum secretin levels
C. order a CT scan of the abdomen
D. order an MRI of the abdomen
E. order an ultrasound of the abdomen
Explanation:
The correct answer is A. This patient most likely has Zollinger-Ellison
syndrome (ZES), which is typically characterized by peptic ulcers, increased
gastrin secretion, and a tumor in the pancreas (gastrinoma). Many patients with
ZES have multiple endocrine neoplasia I (MEN I), which is an autosomal
dominant disorder consisting of tumors in the parathyroid gland, pancreas, and
pituitary gland. Individuals with ZES often have multiple recurrent or
treatment-resistant peptic ulcers, hypercalcemia, diarrhea, and a family history
of pancreatic, parathyroid, or pituitary tumors. The first study used to evaluate
an individual for ZES is serum gastrin levels. Individuals with ZES typically
have markedly elevated levels of serum gastrin.
The first study used to diagnosis ZES is measuring serum gastrin levels, not
serum secretin levels (choice B).
A CT scan of the abdomen (choice C) or an ultrasound (choice E) may be
helpful in localizing the gastrinoma, after establishing the diagnosis with
elevated serum gastrin levels.
An MRI (choice D) is useful in identifying hepatic metastases of gastrinomas.
However, it is not the most appropriate first step in establishing the diagnosis
of a gastrinoma.
A 69-year-old man comes to the office because of "sticking sensation during
swallowing." He says that about 8 months ago he noticed that he was having
trouble "getting solid food down.” It started to require multiple swallowing
attempts. In the past few weeks, he has begun to have difficulty swallowing
soft foods, such as pasta. He says that he's lost "a bunch of weight" because
"eating is no longer a pleasurable experience. However, there is not much of a
problem with the drink!" He has been a patient of yours for many years, and
you always get a whiff of alcohol and cigarettes on his breath, even though he
tries to hide it with peppermint gum. His temperature is 37.2 C (99.0 F), blood
pressure is 120/80 mm Hg, pulse is 65/min, and respirations are 16/min.
Physical examination is unremarkable. Laboratory studies show:
Hemoglobin
12.9 g/dL
Hematocrit
37%
The most appropriate next step is to
A. prescribe a 1-month course of acyclovir
B. reassure him that he has globus pharyngeus and no further studies are
indicated
C. schedule a barium esophagram
D. schedule a CT scan of the chest and upper abdomen
E. schedule an esophageal manometry
F. schedule 24-hour pH monitoring
Explanation:
The correct answer is C. This patient is presenting with progressive
obstructive dysphagia, (obstruction of the passage of solids more than liquids),
weight loss, anemia. He appears to drink a lot of alcohol, therefore, he most
likely has esophageal cancer. The evaluation of a patient with obstructive
dysphagia that is likely due to cancer, typically includes a barium esophagram,
and an upper endoscopy with biopsy. The barium swallow often shows a lesion
with ragged edges or makes an "apple core" appearance. Esophageal cancer
associated with alcohol abuse is typically squamous cell carcinoma, while
esophageal cancer associated with reflux disease, (Barrett's esophagus) is
typically adenocarcinoma. A CT scan may be performed after the diagnosis is
established to determine the stage of the cancer. The treatment includes
surgery, laser, radiation, and chemotherapy. The prognosis is usually poor.
Acyclovir (choice A), is used to treat dysphagia caused by a herpes infection.
This is usually associated with AIDS, and since this patient does not seem to
be infected with HIV, this management is inappropriate.
Globus pharyngeus (choice B), often presents as a "lump in the throat" with no
associated dysphagia, (difficulty swallowing). It occurs in individuals with
emotion disorders. Diagnostic studies are normal and therefore, reassurance is
usually appropriate. This description is inconsistent with the patient in this
case.
A CT scan of the chest and abdomen (choice D), is used in the staging phases
of esophageal cancer, after the diagnosis is established with a barium swallow,
and an upper endoscopy with biopsy.
Esophageal manometry (choice E), is used to diagnose motility disorders such
as achalasia, which is the reduced or absent peristalsis and high pressure at the
lower esophageal sphincter leading to incomplete relaxation when swallowing.
It is associated with difficulty in swallowing solids and liquids.
24-hour pH monitoring (choice F), is used to diagnose acid reflux, which is
associated with heartburn, dysphagia, odynophagia, hoarseness, wheezing, and
aspiration. This patient's history is suggestive of esophageal cancer, not acid
reflux disease.
A 39-year-old man comes to the office complaining of a 3-day history of
severe abdominal pain and cramps that are relieved with bowel movements. He
also reports loose, watery stools two to five times per day. He has had similar
symptoms in the past and recalls the first incident being nearly 12 years ago.
He tells you that he has been told that he has irritable bowel syndrome. He
states that he has never had any other "tests" and was only prescribed various
medications, some of which seemed to have helped. On examination, he
appears to be in mild distress. His temperature is 38.3 C (101.0 F). He has mild
guarding in his left lower quadrant but no rebound tenderness. He is tender to
direct palpation in his left lower and left middle quadrants. The most
appropriate next step in this patient's care is to
A. prescribe corticosteroids and see the patient in two weeks
B. prescribe loperamide and see the patient in two weeks
C. order a stool Gram stain and culture
D. schedule the patient for an immediate colonoscopy
E. schedule an immediate CT scan of the abdomen
Explanation:
The correct answer is E. Although this patient carries the diagnosis of
irritable bowel syndrome, his presentation on this occasion has some elements
that are concerning. In particular his fever and impressive abdominal
examination coupled with his diarrhea and pain raise a high suspicion for an
acute abdomen of some variety. This process may or may not be related to any
existing abdominal pathology that this patient may have. He requires imaging
of his abdomen to rule out an acute abdominal process such as abscess,
pancreatitis, appendicitis, or even colitis.
Prescribing corticosteroids (choice A) or loperamide (choice B) and seeing the
patient in two weeks presumes that this presentation has a similar etiology to
his past presentations. Again, the impressive abdominal examination and fever
makes the likelihood of this being related simply to irritable bowel syndrome
very unlikely.
A stool Gram stain and culture (choice C) would be useful, but not more so
than abdominal imaging.
Arranging for an immediate colonoscopy (choice D) is not correct for two
reasons. First, the bowel preparation required for a good study doesn't make
this test useful for acute situations. Secondly, the test limits visualization of
possible etiologies to those that affect the large bowel.
A 24-year-old man comes to the office because of a 2-day history of
"excruciating" pain during defecation. He states that the pain often lasts for a
couple of hours after defecation and is accompanied by spasms of what he
assumes is the anal sphincter. There is bright red blood on the toilet paper
during wiping. There is sometimes blood, not associated with stool, on the
toilet paper that he uses to "pat the pain away" for up to a half an hour after
defecation. He says, "there is not a minute of the day that is pain-free." He has
no other medical conditions, does not take any medication, and does not smoke
cigarettes. He is sexually active with "many partners." His grandfather died
from colon cancer at age 64. His father has "benign polyps" removed during
colonoscopies every other year, starting at age 50. Physical examination shows
an external skin tag protruding from the anal margin, an enlarged anal papilla,
and a single 0.7-cm linear ulcer in the posterior commissure of the anal verge.
The most appropriate next step is to
A. advise him to take stool softeners and sitz baths
B. prescribe acyclovir therapy
C. prescribe metronidazole therapy
D. refer him for an internal sphincterotomy
E. schedule a colonoscopy
Explanation:
The correct answer is A. This patient has the classic symptoms of an acute
anal fissure, which is a linear ulceration of the anoderm, often caused by hard
stool, and is therefore initially treated with stool softeners and sitz baths.
Increased dietary fiber is also recommended. It is often said that the pain of an
anal fissure is disproportionate to the size of the lesion.
Acyclovir (choice B) is used to treat herpes infections, which are characterized
by painful vesicles and pustules that ulcerate. A single linear ulcer is not the
typical presentation.
Metronidazole (choice C) is often prescribed for anorectal abscesses associated
with Crohn's disease, which may be larger, purulent, and less "benign" looking.
The pain of an abscess may be constant and not necessarily triggered by
defecation, as it is in this case. This patient does not have any of the symptoms
associated with Crohn's disease.
Internal sphincterotomy (choice D) is often reserved for chronic anal fissures.
It is not generally performed for an ulcer that has been present for 2 days.
Conservative management is generally recommended.
A colonoscopy (choice E) is not indicated in this 24-year-old man with the
classic presentation of an anal fissure. Even though he has a family history of
colon cancer, it is extremely unlikely that this is fissure is associated with
cancer. A colonoscopy may be indicated at an earlier age than usually
recommended (50 years) because of his family history, but certainly not at age
24. This decision is often made based on many factors in each individual case.
A 58-year-old alcoholic with hepatitis C cirrhosis is admitted to the hospital
for management of his ascites. He has been managed as an outpatient with
diuretics and oral lactulose, but over the past few weeks, he reports increasing
abdominal girth, weight gain and lower extremity edema. He has been
noncompliant with his low-sodium diet. His medications include furosemide,
spironolactone, lactulose, ciprofloxacin, and thiamine. On physical
examination, he appears grossly edematous and appropriately responsive. His
lungs are clear and his heart is without extra sounds or murmurs. His abdomen
is tense with a fluid wave and shifting dullness on percussion. He has
numerous non-blanching telangiectasias on his torso and abdomen. His testes
are small for his age and there is no asterixis. Admission laboratory studies
show:
Sodium
121 mEq/L
Potassium
4.3 mEq/L
Bicarbonate
29 mEq/L
BUN
38 mg/dL
Creatinine
1.5 mg/dL
Urinalysis shows some granular casts and a urinary sodium concentration of
<10 mmol/L. The most appropriate therapy is to
A. administer hypertonic saline
B. administer sodium chloride tablets
C. increase the dose of furosemide
D. increase the dose of spironolactone
E. salt restrict
Explanation:
The correct answer is E. True hyponatremia is always hypotonic. There are
then three types of hyponatremia: hypovolemic, euvolemic, or hypervolemic.
The therapy for the hyponatremia depends on both the urinary concentration of
sodium as well as the volume status for each patient. Once the patient's total
body volume status has been estimated, urinary sodium usually allows
focusing of the differential diagnosis. For this patient, he clearly has total body
volume overload. He has cirrhosis and portal hypertension. The standard
therapy for these patients is salt restriction and management of volume status
with diuretics. This patient, at the core of his therapy requires salt restriction.
Although some forms of hyponatremia respond to administering hypertonic
saline (choice A), knowing the pathophysiology of this patient's hyponatremia,
it can be clearly seen that this intervention will only aggravate this patient's
condition.
Giving the patient sodium chloride tablets (choice B) will clearly not be
beneficial to this patient given the above explanations.
Increasing the dose of furosemide (choice C) or the dose of spironolactone
(choice D) will certainly serve to diurese the patient, but in the presence of a
salt load, the kidneys will respond by avid retention of salt and water and thus
aggravate the patient's condition.
A 48-year-old woman comes to the emergency department with right upper
quadrant pain. Except for minor epigastric and right upper quadrant discomfort
in the past few months, she reports being in good health. She never sought
medical evaluation, but did take over-the-counter antacids. Now she complains
of right upper quadrant pain for the past 4 hours that started abruptly during the
night and woke her up from sleep. Since then, the pain has been persistent in
the right upper quadrant and is progressively getting worse. Her temperature is
37.9 C (100.2 F), blood pressure is 140/80 mm Hg, and pulse is 94/min. Chest
auscultation reveals slightly diminished breath sounds in the base of the right
lung. Abdominal examination reveals a soft, distended abdomen with diffuse
discomfort, localized to the right upper quadrant with a positive Murphy's sign.
Laboratory studies show a leukocyte count of 16,000/mm3. Her serum bilirubin
is 1.4 mg/dL. The remainder of the complete blood count, metabolic panel, and
liver function tests are within normal limits. A clinical diagnosis of acute
cholecystitis is made and the patient is referred for sonography.
Ultrasonography of right upper quadrant demonstrated no gallstones, but
gallbladder wall thickening with peripheral cystic fluid. You diagnose her with
acute cholecystitis and admit her to the hospital for treatment with intravenous
antibiotic therapy. Three hours after admission to the hospital, you are called to
the floor as she is complaining of severe abdominal pain, which got worse
since admission. On examination, her vitals are unchanged, but her abdominal
examination reveals voluntary guarding, right upper quadrant pain, and boardlike rigidity of the abdomen. The most appropriate next step is to order
A. an abdominal x-ray, erect and supine
B. a CT scan of the chest
C. a CT scan of the abdomen and pelvis
D. a hepatobiliary nuclear scan
E. an upper gastrointestinal endoscopy
Explanation:
The correct answer is A. This patient presented with sudden onset of right
upper quadrant pain associated with nausea and vomiting. She has had right
upper quadrant and epigastric pain before, associated with her food intake.
These symptoms are classical for a perforated peptic ulcer. In a perforated
peptic ulcer, a patient can still have right upper quadrant localized tenderness,
a thickened gallbladder wall, and pericholecystic fluid from the perforated
ulcer. Hence, with any abdominal pain associated with signs, abdominal x-rays
both erect and supine are very essential in the initial evaluation to rule out any
free air. Worsening of the symptoms, development of board like rigidity, and
voluntary guarding are signs of peritonitis from a perforated peptic ulcer.
A CT scan of the chest (choice B) is not indicated in this patient, as there were
no suspected pulmonary symptoms and no signs.
A CT scan of the abdomen and pelvis (choice C) might reveal free air, free
leakage of the contrast from the perforated peptic ulcer, or infiltration of the
mesentery and omentum in the upper quadrant, suggestive of perforated peptic
ulcer. But, a simple abdominal x-ray is often essential to diagnose free air,
before performing the CT scan of the abdomen and pelvis.
A hepatobiliary scan (choice D) has a high sensitivity for diagnosing acute
cholecystitis, but is not useful in a perforated peptic ulcer.
An upper gastrointestinal endoscopy (choice E) is essential as a primary
investigative tool in elective patients. In patients with peritonitis and a
suspected perforated peptic ulcer, an upper GI endoscopy is not the first
investigational choice.
A 2-year-old girl is brought to the emergency department because of the abrupt
onset of spasms of severe, crampy abdominal pain. The mother says that she
was "completely fine" earlier in the day. She picked her up from a "play date"
at her friend's house, they picked up some fast food for dinner, and before she
even started to eat, she became very irritable, and began complaining of pain.
In the hospital bathroom, she had a bowel movement with mucus and blood.
She is generally healthy and takes no medications. She is lying on the
examination table with her knees drawn into her chest. Her temperature is 36.7
C (98.0 F). Physical examination shows a tender, sausage-shaped mass in the
right lower quadrant. Rebound tenderness is not present. A nasogastric tube is
placed. The most appropriate next step is to
A. arrange for immediate exploratory laparotomy
B. begin therapy with metronidazole
C. order a barium enema
D. order a CT scan of the abdomen
E. reassure that the condition is self-limiting and no further studies are
needed
Explanation:
The correct answer is C. This patient most likely has an intussusception,
which is a segment of bowel telescoping into a distal segment, and typically
presents with the abrupt onset of colicky abdominal pain, lethargy, and
"currant jelly" stools, (stool mixed with blood and mucus). A tender, sausageshaped mass may be palpated. Children may bring their knees to their chest to
relieve the pain. It usually occurs in children less than 2 years old. The exact
cause is unknown. However, it is associated with Meckel's diverticulum, cystic
fibrosis, polyps, and Henoch-Schonlein purpura. A barium enema is both
diagnostic and therapeutic. It will typically show a "coiled-spring" appearance
of the bowel segment and reduce the intussusception. If this is unsuccessful,
surgery may be required.
This patient most likely has an intussusception, which can usually be
diagnosed and treated with a barium enema. If this is not successful in
reducing the intussusception, it may be necessary to arrange for an immediate
exploratory laparotomy (choice A). Surgery is not the initial management.
Metronidazole (choice B) is a medication used to treat anaerobic infections and
infections with trichomonas, Giardia, and bacterial vaginosis. It is not indicated
in a patient with an intussusception, because this is not an infection. It is a
segment of bowel telescoping into a distal segment.
A CT scan (choice D) may be used to diagnose appendicitis, pancreatitis, and
diverticulitis. However, it is not typically used in the diagnosis or treatment of
an intussusception.
It is inappropriate to reassure that the condition is self-limiting, and no further
studies are needed (choice E), because this patient most likely has an
intussusception, which requires reduction with a barium enema to prevent
perforation. An untreated intussusception could be fatal.
A 48-year-old investment banker comes to the office because of a 4-month
history of achy abdominal pain. He says that the pain is exacerbated by meals
and he often feels very nauseous. He is generally very healthy except for some
mild lower back pain for which he takes ibuprofen. He estimates that he has
taken 2 over-the counter ibuprofen pills every 3 days for the past few months.
He smokes a half pack of cigarettes a day and drinks a glass of wine with
dinner every night. He works until 10 p.m. on weekdays and both days of the
weekends. He has to take care of his children in his spare time and says that he
is very "stressed out." Physical examination shows mild epigastric tenderness.
A urea breath test is positive and a barium study shows a 1.5 cm discrete crater
in the antrum of the stomach with radiating mucosal folds originating from the
ulcer margin. The most likely cause of this patient's condition is
A. alcohol consumption
B. a bacterial infection
C. chronic use of ibuprofen
D. cigarette smoking
E. psychological stress
Explanation:
The correct answer is B. This patient has a gastric ulcer that is most likely
due to an infection with H. pylori. H. pylori is a Gram-negative rod that can be
detected by the urea breath test, serology, a rapid urease test, or histological
evaluation of a biopsy specimen. The urea breath test has a greater than 90%
sensitivity and specificity for H. pylori, so a bacterial infection is the most
likely cause of his condition.
Alcohol consumption (choice A) has been thought to play a role in the
pathogenesis of peptic ulcer disease (gastric and duodenal ulcers), but its exact
role is not proven. Since he had a positive urea breath test, H. pylori is the most
likely cause of his condition, not alcohol.
Chronic use of ibuprofen (choice C) is associated with peptic ulcer disease.
However, this patient does not take large amounts of ibuprofen. Also, because
he had a positive urea breath test, he has an H. pylori infection. While it is
possible to have an NSAID-induced gastrointestinal ulceration and a
concomitant infection with H. pylori, in this case his condition is most likely
caused by the bacteria.
Cigarette smoking (choice D) has been associated with peptic ulcer disease,
but the exact mechanism is unknown. This patient's positive urea breath test is
consistent with an H. pylori infection. The cigarette smoking may exacerbate
his condition, but it is unlikely to be the main cause.
Psychological stress (choice E) has long been associated with peptic ulcer
disease, but studies on the subject have produced conflicting results. It is
possible that this patient's stress may exacerbate his symptoms but it is very
unlikely that it is the cause of his condition.
A 53-year-old man is admitted to the hospital from the emergency department
because of worsening confusion. He is brought in by a friend who reports that
the patient has "liver disease", has been drinking lately, and has not been
taking his medications. The friend tells you that he has gotten progressively
more confused over the past few days. She only knows a vague history but
thinks the patient has "cirrhosis". She does not think the patient has had a
recent fall, even though he has not been without alcohol for any appreciable
length of time. His temperature is 37.0 C (98.6 F), blood pressure is 120/70
mm Hg, and pulse is 100/min. He has deep scleral icterus and his skin is
jaundiced. His lungs are clear, cardiac exam is normal, and he has a distended
abdomen with shifting dullness. He is alert to person only and his neurological
exam is normal with the exception of the inability to perform finger to nose
touching and heel to shin maneuvers. He has asterixis. Laboratory studies
show:
The most likely cause of his confusion is
A. acute hyponatremia
B. ascending cholangitis
C. hepatic encephalopathy
D. metabolic acidosis
E. subdural hematoma
Explanation:
The correct answer is C. The most important concept to understand from this
question is what complications affect cirrhotics. The evaluation of altered
mental status in the emergency department is a complex topic, but one of the
most useful and essential components of this evaluation is the history. When
the history is given of a cirrhotic "not taking his medications," an
understanding that encephalopathy is one of the most likely diagnosis should
come immediately to your mind.
Acute hyponatremia (choice A) is not equivalent to the serum Na of
129mEq/L. Acute signs of hyponatremia are seen when the serum sodium falls
more than 12mEq/L in less than 24 hours. The signs of such an illness involve
nausea, vomiting, confusion, and focal neurological findings related to brain
edema.
Ascending cholangitis (choice B) is not supported by your physical exam or by
physical findings such as the classic Charcot triad (fever, right upper quadrant
pain, and jaundice).
Metabolic acidosis (choice D) is not supported by the data. The patient does
have a low bicarbonate level, but his pH is not known. It is likely acidemic but
this is not equivalent to inferring that the patient is suffering confusion as a
result of the acidemia.
Subdural hematoma (choice E) is not the most likely diagnosis in this
noncompliant, cirrhotic patient. The most common cause of SH is tearing of
the bridging veins suffered during a trauma. It is certainly possible, however,
since hepatic encephalopathy occurs in more than half of all cirrhotics with
severe impairment of liver function, and since the friend tells you that he has
not been taking his medications, this patient's confusion is most likely due to
hepatic encephalopathy.
A 73-year-old man with emphysema comes to the clinic with complaints of
food getting stuck when he swallows, which has been getting progressively
worse over the last 8 months. He denies problems swallowing liquids and
thinks he has lost about 5 pounds. He used alcohol heavily for many years but
quit drinking 10 years ago. He still smokes 1 pack of cigarettes per day and has
done so since age 20. He uses albuterol, steroid inhalers and theophylline. His
blood pressure is 123/73 mm Hg, pulse is 87/min, and respirations are 20/min.
Physical examination reveals bilateral scattered wheezes in the lungs. A chest
x-ray shows hyperexpansion and no nodules. The most appropriate next step in
management is to
A. order a barium esophagram
B. order an esophageal manometry
C. order an esophageal pH probe
D. treat with omeprazole and follow up in 3 months
E. treat with ranitidine and follow up in 3 months
Explanation:
The correct answer is A. This patient most likely has an esophageal
squamous cell carcinoma (the most common type of esophageal malignancy).
In any patient with dysphagia that is progressive for only solids, it suggests a
growing and obstructive lesion. The history of tobacco and alcohol use, puts
this person at a much higher risk of carcinoma. The two ways to diagnose this
are a barium swallow study, which will show the mucosal mass, or an upper
endoscopy study to directly visualize and biopsy the lesion.
Esophageal manometry (choice B) is used to evaluate dysphagia caused by
motility disorders. These typically present with dysphagia for solids and
liquids and may or may not be progressive.
A pH probe (choice C) is used to evaluate esophageal reflux disease, which
does not in itself typically cause dysphagia, but over long periods of time will
increase the risk of esophageal adenocarcinoma.
Both omeprazole (choice D) and ranitidine (choice E) are used to treat
symptoms of gastroesophageal reflux disease and would not address his
dysphagia. Furthermore, waiting 3 months to see the patient again would be
inappropriate.
A 73-year-old man with emphysema comes to the clinic with complaints of
food getting stuck when he swallows, which has been getting progressively
worse over the last 8 months. He denies problems swallowing liquids and
thinks he has lost about 5 pounds. He used alcohol heavily for many years but
quit drinking 10 years ago. He still smokes 1 pack of cigarettes per day and has
done so since age 20. He uses albuterol, steroid inhalers and theophylline. His
blood pressure is 123/73 mm Hg, pulse is 87/min, and respirations are 20/min.
Physical examination reveals bilateral scattered wheezes in the lungs. A chest
x-ray shows hyperexpansion and no nodules. The most appropriate next step in
management is to
A. order a barium esophagram
B. order an esophageal manometry
C. order an esophageal pH probe
D. treat with omeprazole and follow up in 3 months
E. treat with ranitidine and follow up in 3 months
Explanation:
The correct answer is A. This patient most likely has an esophageal
squamous cell carcinoma (the most common type of esophageal malignancy).
In any patient with dysphagia that is progressive for only solids, it suggests a
growing and obstructive lesion. The history of tobacco and alcohol use, puts
this person at a much higher risk of carcinoma. The two ways to diagnose this
are a barium swallow study, which will show the mucosal mass, or an upper
endoscopy study to directly visualize and biopsy the lesion.
Esophageal manometry (choice B) is used to evaluate dysphagia caused by
motility disorders. These typically present with dysphagia for solids and
liquids and may or may not be progressive.
A pH probe (choice C) is used to evaluate esophageal reflux disease, which
does not in itself typically cause dysphagia, but over long periods of time will
increase the risk of esophageal adenocarcinoma.
Both omeprazole (choice D) and ranitidine (choice E) are used to treat
symptoms of gastroesophageal reflux disease and would not address his
dysphagia. Furthermore, waiting 3 months to see the patient again would be
inappropriate.
A 54-year-old man with end-stage liver disease secondary to hepatitis C comes
to the emergency department with fevers and mental status changes over the
last 4 days. His wife reports that he has been compliant with his medications,
which include furosemide, spironolactone, and lactulose up until today when
he refused to take them. His temperature is 38.0 C (100.7 F), blood pressure is
100/70 mmHg, pulse is 103/min, and respirations are 19/min. Physical
examination reveals a confused and slightly combative male with scleral
icterus. His abdomen is distended with bulging flanks, shifting dullness, and a
fluid wave. He has asterixis. There is no nuchal rigidity or photophobia. He is
oriented to person but not place or time. The most appropriate next step in this
patient's management is to
A. determine his ammonia level
B. order a CT scan of the head
C. perform a lumbar puncture
D. perform paracentesis
E. send a urine culture and sensitivity
Explanation:
The correct answer is D. This patient likely has spontaneous bacterial
peritonitis. This diagnosis should be first on your list in any patient with ascites
who presents with fevers, abdominal pain, change in mental status, or with
other non-specific complaints. These patients need to have a paracentesis. This
fluid is then sent to the lab for a cell count, culture, and Gram stain. The
diagnosis of SBP can be made by seeing bacteria on a Gram stain, having more
than 500 WBC or 250 PMNs in the cell count, or a positive peritoneal fluid
culture. Patients with SBP need to be started on a third-generation
cephalosporin.
Ammonia levels (choice A) can be elevated in patients that are
encephalopathic. This patient is encephalopathic as evidenced by his asterixis
and mental status changes. Ammonia levels can be followed if you want
additional evidence that this patient's medications are effectively causing a
decrease in those levels, but ammonia should not be used to make the initial
diagnosis of encephalopathy.
CT scan (choice B) is part of the work-up of mental status changes in the
elderly, but acute stroke resulting in mental status changes in a 54-year-old
would be less likely. CAT scan along with LP might be necessary if the
paracentesis does suggest peritonitis.
Lumbar puncture (choice C) is also part of the work-up for mental status
changes but this patient doesn't have headache, high fevers, nuchal rigidity, or
photophobia. Meningitis would be lower on the list of differential diagnoses in
this patient and LP is not indicated as an initial diagnostic procedure.
Urine culture and sensitivity (choice E) is a test that we should all have a very
low threshold to since urosepsis is a very common cause of fevers and mental
status changes in the elderly and in the immunocompromised. This patient
should have his urine evaluated, but the best answer is paracentesis since a
positive urine culture would not rule out confounding peritonitis.
A 27-year-old woman comes to the emergency department because of severe
abdominal pain and diarrhea. She has not been feeling "great" for about a
month and over the past few days the pain has localized in the right lower
quadrant. She is generally very healthy, exercises regularly, does not drink or
smoke, and eats a "fairly" low-fat diet. Her temperature is 37.2 C (99.0 F),
blood pressure is 120/80 mm Hg, and pulse is 65/min. Physical examination
shows right lower quadrant tenderness, but no guarding or rebound tenderness.
You have a funny feeling that this is not the "run of the mill appendicitis," and
you decide to order a small bowel series, which shows a cobblestone
appearance of the small intestinal mucosa. A colonoscopy is performed and
multiple biopsies are taken throughout the colon. All of the biopsies show
transmural inflammation, granulomas, and fibrosis. She is admitted to the
hospital and treated with mesalamine, metronidazole, and corticosteroids. At
discharge, she asks you to be "brutally honest" about the long-term effects of
her condition. The most appropriate response is:
A. "It is likely that you will have a chronic, intermittent course requiring
surgery, and may have relapses and recurrences even after surgery."
B. "Unfortunately, a colectomy is the only definitive treatment for your
disease"
C. "Unfortunately, you will need to have surveillance colonoscopies every
year beginning in 6 months to detect dysplastic changes"
D. "You will most likely recover completely and not require additional
therapy"
E. "You will recover from this episode but will probably need a colectomy
in the future"
Explanation:
The correct answer is A. This patient has Crohn disease (CD), which is often
a chronic and intermittent disease. Approximately 2/3 of all patients with CD
require surgery at some point in their disease, and unfortunately, relapses and
recurrences occur even after surgery. The classic presentation of CD is right
lower quadrant tenderness and diarrhea. It can affect all parts of the
gastrointestinal tract from the mouth to the anus.
It is inappropriate to tell her that "Unfortunately, a colectomy is the only
definitive treatment for your disease" (choice B) because this is true for
ulcerative colitis (UC), not Crohn disease, which can affect the entire
gastrointestinal tract from the mouth to the anus. UC typically presents with
bloody diarrhea and tenesmus or urgency.
"Unfortunately, you will need to have surveillance colonoscopies every year
beginning in 6 months to detect dysplastic changes" (choice C), is incorrect.
The risk of colon cancer is increased in CD, but not as much as it is in UC. The
risk seems to increase after 10 years duration of the disease at approximately 12% a year for patients with UC. Surveillance colonoscopies are generally
recommended every 6 months for 2 years beginning after 8 to 10 years
duration of ulcerative colitis. Since it is not exactly known what the risk of
cancer is in patients with CD, there is no standard recommendation. However,
it is incorrect to perform so many colonoscopies this early in the disease to
screen for dysplasia and carcinoma in this patient with CD.
"You will most likely recover completely and not require additional therapy"
(choice D) is incorrect because most patients with CD have a chronic and
intermittent course, with approximately 2/3 of all patients with CD requiring
surgery at some point in their disease.
"You will recover from this episode but will probably need a colectomy in the
future" (choice E) is incorrect because even though approximately 2/3 of all
patients with CD require surgery at some point in their disease, it is not
necessarily a colectomy that they need. Remember that CD can occur
anywhere in the gastrointestinal tract, from the mouth to the anus, and
therefore, surgery is not limited to the colon. Surgery may be required for
narrowings or obstructions, persistent fistulas or abscesses, perforations or
toxic megacolon.
A 61-year-old man with hypertension and cirrhosis comes to the emergency
department because of increasing abdominal distention. He was diagnosed
with cirrhosis 3 years ago after presenting to the hospital with fever and
abdominal pain. At that time he was discharged after therapy for spontaneous
bacterial peritonitis. He has not received medical care since that time. Over the
past 3 months, he says that he has noticed increasing abdominal girth,
progressive lower extremity edema, and some mild shortness of breath. He
smokes 2 packs of cigarettes per day and has a long-standing history of alcohol
abuse. He continues to consume 3-6 beers per day. His blood pressure is
130/80 mm Hg and pulse is 90/min. He is anicteric, has clear lungs, mild
gynecomastia, bulging flanks with shifting dullness, and small testes. It is most
important to tell him that:
A. Continuation of alcohol consumption will worsen his condition
B. He is not at an increased risk of developing hepatocellular carcinoma
C. His condition is most likely caused by an infection with the hepatitis A
virus
D. The presence of concurrent pulmonary disease will worsen his condition
E. The presence of concurrent renal disease will worsen his condition
Explanation:
The correct answer is A. The majority of cirrhosis in the world is due to
schistosomiasis and hepatitis. In the United States, the epidemiology is
somewhat different with the most common causes being alcohol and viral
hepatitis B/C. Given this, the most important prognostic factor in determining
future disease progression is whether the patient continues to consume ethanol
after their diagnosis. Ethanol is hepatotoxic under normal circumstances and in
the setting of active hepatitis or cirrhosis, alcohol accelerates hepatocyte death
and disease progression.
It is incorrect that he is not at increased risk of developing hepatocellular
carcinoma (choice B) because he is at risk for this malignancy irrespective of
the cause of his cirrhosis.
It is unlikely that his condition is due to an infection with the hepatitis A virus
(choice C) since the majority (>95%) of these patients have a self-limited,
benign disease.
Whether or not he has concurrent pulmonary disease (choice D) and whether
or not he has concurrent renal disease (choice E) are important for
transplantation evaluation but will not affect the course of his cirrhosis.
Cirrhotics are at risk for developing both the hepatorenal and hepatopulmonary
syndromes but these are not related to any underlying renal or pulmonary
disease the patient may already have.
A 44-year-old previously healthy man is admitted to the hospital with a 3-day
history of abdominal pain that initially started as a diffuse pain, and later
became constant and severe, localized to the left lower quadrant. He also had
nausea, vomiting, and a fever of 38.3 C (101.0 F). His past medical history is
remarkable for an appendectomy at the age 9, and a hernia repair at the age of
20. Both surgical procedures were uneventful, and he was discharged home on
the next postoperative day. On admission to the hospital, his temperature is
38.1 C (100.6 F), blood pressure is 140/80 mm Hg, and pulse is 90/min.
Abdominal examination shows a distended, soft abdomen with tenderness and
fullness in the left lower quadrant. Rectal examination does not reveal any
abnormalities. A nasogastric tube is inserted and drains 400 ml bilious
material. Laboratory studies show a leukocyte count of 18,000/mm3.
Abdominal x-ray shows 6-7 air fluid levels with no free air. A CT scan of the
abdomen and pelvis, performed on the day of admission to the hospital,
confirms diverticulitis localized to the sigmoid colon with no abscess or free
air. He is started on intravenous antibiotics and kept nil per os. Over the next
few days, his temperature normalizes and his pain resolves with intravenous
antibiotics. The most appropriate management of this patient is to
A. order a barium enema
B. perform a colonoscopy
C. recommend an elective sigmoid colon resection
D. repeat a CT scan of the abdomen and pelvis in 1-week
E. switch him to oral antibiotics
Explanation:
The correct answer is C. When diverticulitis occurs in young patients, below
the age of 50, it is usually in its severe form. In elderly patients, surgery is
indicated after 3 attacks of diverticulitis or for complications of diverticulitis
like fistula formation or an abscess or free perforation with diffuse peritoneal
signs. In younger patients, the first attack of diverticulitis is usually severe.
When treated non-operatively with intravenous antibiotics, the chances of
recurrent attacks of diverticulitis are very high. The recurrent attacks of
diverticulitis in these patients are usually in its severe form and associated with
complications. Hence, in young people, (below the age of 50) the first attack of
diverticulitis should be treated initially with intravenous antibiotics, and then a
definitive surgical procedure should be performed to resect the involved part of
the colon. In older patients, elective resection is usually recommended after 3
attacks.
A barium enema (choice A) is not performed during the first few days of acute
diverticulitis.
A colonoscopy (choice B) is contraindicated during an acute attack of
diverticulitis.
A repeat CT scan of the abdomen and pelvis (choice D) is not indicated unless
a complication of diverticulitis develops during the course of the disease.
A continuation of intravenous antibiotics is essential for patients with the
severe form of diverticulitis that has been proven by a CT scan of the abdomen
and pelvis. This treatment should continue until the symptoms settle down. In
mild forms of diverticulitis, the patient can be treated with oral antibiotics
(choice E) at home. But in younger patients, a surgical procedure is indicated
even with the first attack of diverticulitis.
A 65-year-old Asian American man comes to the clinic for a follow-up
appointment for symptoms of dysphagia. He has had difficulty swallowing
solid food off and on for the past year. He has no difficulty swallowing liquids
or pills. He has no significant medical problems and his only medication is an
occasional aspirin for arthritis pain in his knees. He denies cigarette smoking,
but he does drink 1-2 glasses of wine each week. An outpatient esophagram
was performed 3 days ago and the x-ray shown is one of the films obtained
during the study. Based on the findings of the esophagram, this patient is at
increased risk for developing
A. Barrett's esophagus
B. esophageal cancer
C. gastric ulcers
D. gastroesophageal reflux
E. Zencker's diverticulum
Explanation:
The correct answer is E. The esophagram demonstrates an indentation on the
posterior cervical esophagus, which is due to a hypertrophied cricopharyngeus
muscle. A Zencker's diverticulum is a pharyngoesophageal pulsion
diverticulum that occurs at the point of transition between the oblique fibers of
the thyropharyngeus muscle and the horizontal fibers of the cricopharyngeus
muscle. A hypertrophied cricopharyngeus muscle is thought to predispose the
development of the diverticulum at this transition point. This can be an
incidental finding or in some patients, as in this case, it can cause symptoms of
dysphagia.
Barrett's esophagus (choice A) is a complication of gastroesophageal reflux
disease. There is metaplasia of the normal cells in the esophagus to what is
called "specialized columnar epithelium". Chronic damage from the acidic
gastric contents is believed to promote the replacement of the normal
esophageal epithelium with the metaplastic columnar cells. This is a
predisposing condition for adenocarcinoma of the esophagus. There is no
association with hypertrophy of the cricopharyngeus muscle.
There is no relationship between a hypertrophied cricopharyngeus muscle and
esophageal cancer (choice B).
There is no relationship between a hypertrophied cricopharyngeus muscle and
gastric ulcers (choice C).
There is no relationship between a hypertrophied cricopharyngeus muscle and
gastroesophageal reflux (choice D). Reflux can occur due to a hiatal hernia or
a dysfunctional lower esophageal sphincter.
A 64-year-old woman comes to the emergency department with a 36-hour
history of diffuse abdominal pain, abdominal fullness, nausea, and vomiting.
She has no appetite and is unable to eat or drink secondary to nausea and
vomiting, which is bilious in color. She passed loose brown stool earlier today.
She denies any bright red blood per rectum or bloody vomitus. Her past
medical history is notable for endometrial cancer 4 years ago treated with
surgery and radiation. The patient denies ever experiencing similar symptoms
in the past. Her temperature is 37.0 C (98.6 F), blood pressure is 110/70 mm
Hg, pulse is 100/min, and respirations are 16/min. She has a moderately
distended abdomen with diffuse tenderness on palpation. There is no rebound
tenderness or guarding. Bowel sounds are high-pitched. There is no occult
blood on rectum examination. Initial laboratory studies show:
The next most appropriate step to confirm the diagnosis is to obtain
A. a CT scan of the abdomen
B. plain films (supine abdomen x-ray and upright chest and abdomen x-ray)
C. a right upper quadrant ultrasound
D. serum amylase and lipase
E. serum lactate
Explanation:
The correct answer is B. This patient most likely has a small bowel
obstruction as evidenced by prior abdominal surgery, a diffusely tender
abdomen, high-pitched bowel sounds, and nausea and vomiting. Small bowel
obstruction usually results from either mechanical blockage or paralytic ileus.
Postoperative adhesions or an incarcerated inguinal hernia are the most
common causes of a mechanical small bowel obstruction. This patient is at risk
for bowel obstruction given her prior abdominal surgery and radiation, thus
putting her at increased risk for adhesion and stricture formation. Patients with
small bowel obstruction typically present with a distended, tender abdomen,
nausea and vomiting, and high-pitched bowel sounds. The presence of
peritoneal signs suggests infarcted bowel or perforation. Laboratory tests may
reveal a normal or elevated white blood cell count, elevated hematocrit
secondary to hemoconcentration, and low potassium and chloride secondary to
vomiting. Blood urea nitrogen and creatinine may be elevated secondary to
prerenal azotemia. Plain films are very helpful in diagnosing small bowel
obstruction and should be part of the initial management of the patient. X-rays
of the abdomen commonly reveal gas-filled, distended loops of bowel with airfluid levels. An upright chest x-ray is needed to evaluate for possible bowel
perforation which is demonstrated by the presence of free air under the
diaphragm.
A CT scan of the abdomen (choice A) is rarely needed to diagnose a small
bowel obstruction and can worsen a patient's symptoms, especially in the
setting of a complete small bowel obstruction. In complete obstruction,
administering contrast from above will add to the degree of fluid and edema
proximal to the obstructed bowel. A CT scan of the abdomen may be helpful
later in a patient's course. For example, it may be necessary to obtain a CT scan
to exclude the possibility of a tumor compressing bowel that leads to
obstruction. This imaging modality, however, is not part of the immediate
management.
A right upper quadrant ultrasound (choice C) is used to image the liver,
gallbladder, and ducts. The patient does not have any abdominal symptoms or
pain localized to this region to warrant starting with this imaging modality. An
uncommon but important cause of small bowel obstruction in elderly patients
is gallstone ileus. In this setting, a gallstone erodes through the gallbladder
wall and into the small bowel, causing intraluminal obstruction. Typically,
patients will give a history of right-sided upper abdominal pain at the onset of
their symptoms and may demonstrate air in the biliary tree on ultrasound. This
patient, however, does not have any symptoms to suggest the presence of
gallstones.
An elevated serum amylase and lipase (choice D) is useful in diagnosing
pancreatitis. Pancreatitis usually presents with epigastric pain radiating to the
back, as well as nausea and vomiting. The two leading causes of pancreatitis
are alcohol abuse and gallstones. This patient's presentation and exam are not
classic pancreatitis as described above.
An elevated lactate (choice E) is frequently elevated in the setting of
hemorrhage, shock, sepsis, and hypoxia. This patient has an elevated anion gap
of which suggests the presence of a lactic acidosis. An elevated lactate,
however, may suggest many processes as outlined above and thus would not be
helpful in confirming the diagnosis of small bowel obstruction.
A 42-year-old man is brought to the emergency department by his wife
because of "dizziness" and "rectal bleeding". He says that he has had many
episodes of bright red blood on the toilet paper after defecation, peri-anal
"itchiness", and "soiling" of his undergarments for years, but he was too
embarrassed to ever go to a physician before. Tonight, after going to the
bathroom, he experienced his first episode of "real bad bleeding." His blood
pressure is 90/60 mm Hg and his pulse is 120/min. Physical examination
shows 2 small external hemorrhoids that are not bloody. There is bright red
blood in the rectum, but there are no palpable masses. Laboratory studies
show:
You tell him that it appears that he lost a lot of blood and requires a blood
transfusion. He says that he is a Jehovah's Witness, and is not allowed to
receive blood. He does not change his mind even after you inform him of the
consequences and potential risks of his decision. The most appropriate next
step is to
A. do nothing at this time
B. insert 2 large-bore intravenous catheters and infuse saline
C. obtain a CT scan of the abdomen
D. perform a flexible sigmoidoscopy
E. proceed with blood transfusion without any formal consent
Explanation:
The correct answer is B. The key to answering this question is understanding
the concept of patient autonomy. Every patient has the privilege of deciding
their own fate; this regards all medical decisions. This concept applies to all
patients, except those with impaired judgement. A patient's beliefs, including
religious and cultural, must therefore be respected. This patient does not seem
to have impaired mentation, and he is informed of the potential consequences
of his decision. Intravenous fluids are appropriate at this time to prevent
vascular collapse.
It is inappropriate to do nothing at this time (choice A) because even though he
has decided not to receive blood, intravenous fluids are necessary at this time
to prevent vascular collapse. Just because he refuses blood, does not mean that
he does not want you to perform all of the other indicated procedures. He did
come to the hospital and therefore wants help.
A CT scan of the abdomen (choice C) may be appropriate later on in the workup, but it is necessary to first try to stabilize the patient by inserting 2 largebore intravenous catheters and infusing intravenous saline.
A flexible sigmoidoscopy (choice D) is indicated in helping to determine the
source of his bleeding, but this should be done after he is stabilized with
intravenous fluids.
It is completely inappropriate (and illegal) to proceed with blood transfusion
without any formal consent (choice E) because this patient has made an
informed decision and he is of sound mind and does not have impaired
judgement. You must therefore respect his autonomy and do everything
possible to save his life, except give him a blood transfusion.
A 35-year-old man comes to the office because of "heartburn" for 3 months.
He tells you that he has a "burning sensation" in the chest that begins in the
"upper stomach and travels up to the neck." The symptoms worsen when he
lies down in bed. He has no chronic medical conditions and takes no
medications. He typically drinks 2-3 cups a coffee a day, has a glass of wine
after dinner, and has a piece of chocolate-covered peppermint candy before
bedtime. Physical examination, an electrocardiogram, a complete blood count
and metabolic profile, and serologic testing for H. pylori are unremarkable.
You recommend that he elevate the head of bed, avoid eating before bed, and
avoid all alcohol, tobacco, chocolate, and caffeine, and schedule a follow-up
visit. He comes back to the office after 2 months and says that his symptoms
are unchanged. At this visit, his temperature is 37.0 C (98.6 F), blood pressure
is 120/80 mm Hg, pulse is 65/min, and respirations are 14/min. Physical
examination is unchanged. The most appropriate next step is to
A. order ambulatory esophageal pH testing
B. order an upper gastrointestinal barium radiograph
C. prescribe famotidine
D. schedule an upper endoscopy
E. schedule esophageal manometry
Explanation:
The correct answer is C. This patient most likely has gastroesophageal reflux
disease (GERD). Reflux disease is usually worse at night because the
recumbent position allows gastric acid contents to go up into the esophagus.
Since all of the tests ordered during the first visit were normal, you were
correct in recommending non-pharmacologic therapy, such as elevation of the
head of bed, avoiding eating before bed, and avoiding alcohol, tobacco,
chocolate, and caffeine. Alcohol, tobacco, chocolate, and caffeine all lower the
lower esophageal sphincter pressure leading to gastric reflux. If these measures
are ineffective, pharmacologic therapy with H2 blockers such as cimetidine,
famotidine, or ranitidine is indicated. For more severe symptoms, a proton
pump inhibitor, such as omeprazole or lansoprazole, is indicated.
Ambulatory esophageal pH testing (choice A) is usually reserved for patients
who fail nonpharmacologic and pharmacologic management.
An upper gastrointestinal barium radiograph (choice B) is useful in detecting
esophageal rings or strictures, which typically present with dysphagia. This
patient complains of heartburn, not dysphagia.
An upper endoscopy (choice D) is usually indicated only after the failure of
nonpharmacologic and pharmacologic management for GERD and when a
patient has GERD for >5 years, an upper endoscopy is recommended to screen
for Barrett's metaplasia. It is not indicated at this time.
Esophageal manometry (choice E) is typically reserved for cases of GERD
when surgical therapy is being considered.
A 47-year-old man comes to the clinic for follow-up care of his ascites and
cirrhosis. He was diagnosed with cirrhosis due to hepatitis C 4 years ago that
he believes that he contracted from a blood transfusion. He is anxiously
awaiting liver transplantation. His only other medical history is that he has
diabetes mellitus controlled with insulin. He reports to you that he avoids all
alcohol consumption and takes his medications, which include spironolactone,
furosemide, multi-vitamins, nadolol, and insulin. He complains, however, that
his abdomen continues to "get bigger" despite the fact he limits his water
intake to less that 1 liter per day. In the office his blood sugar is 198mg/dL. His
physical examination is unchanged from previous visits except for more
abdominal distention and 2+ lower extremity edema. In discussing his
increasing ascites, he should be advised that
A. this is because his blood sugar is poorly controlled
B. this is due to his poor compliance with medications
C. this is to be expected since his disease is progressive
D. this is to be expected since he is drinking too much water
E. this is to be expected since he is not limiting his salt intake
Explanation:
The correct answer is E. The important concept to understand from this
question is that, although diuretics are effective in controlling ascites, dietary
sodium consumption negates all such benefit. Any patient who you believe to
be compliant with their medications that has a continued increase in their
ascites, is almost certainly having too much dietary sodium (>2 gm).
This is to be expected because your blood sugar is so poorly controlled (choice
A) is not relevant to the care of this patient's ascites although it is relevant from
a volume standpoint since if he is dehydrated from glucosuria, he will consume
more fluids, and likely salt.
This is to be expected since you have very poor compliance (choice B) is not
an appropriate comment to make to a patient and furthermore is not supported
in your interview with the patient.
This is to be expected since your disease is progressive (choice C) is not an
acceptable answer since the progressive nature of disease effects a patient's
mental status, bleeding risk, and infection risk, not necessarily the
refractoriness of their ascites to diuresis.
This is to be expected since you are drinking too much water (choice D) is
propagated throughout medicine as a cause for both edema due to CHF as well
as ascites. In fact, water consumption, unless truly excessive, is irrelevant to
total body fluid balance. The key determinant of sodium excretion and thus
fluid excretion by the kidney is sodium intake.
A 39-year-old woman comes to the office because of "spasms" of abdominal
pain and nausea over the past few hours. The pain, which is located in the right
upper quadrant and radiates to the back, started a little while after she ate
french fries at a fast food restaurant with her son. She has had similar episodes
of pain in the past. Her temperature is 37.0 C (98.6 F), blood pressure is 120/80
mm Hg, pulse is 75/min, and respirations are 16/min. Physical examination
shows mild right upper quadrant tenderness with no rebound or guarding. An
ultrasound of the abdomen shows acoustic shadowing of opacities within the
gallbladder lumen that change with the patient's position. Laboratory studies
show a leukocyte count of 9,000/mm3. The most likely diagnosis is
A. acute acalculous cholecystitis
B. acute calculous cholecystitis
C. acute cholangitis
D. biliary colic
E. primary sclerosing cholangitis
Explanation:
The correct answer is D. This patient has biliary colic, which is characterized
by crampy, right upper quadrant abdominal pain that may radiate to the back
and often follows a meal. Nausea and vomiting may be present. Fever, chills,
and leukocytosis are notably absent. Gallstones are present on an ultrasound.
The treatment of symptomatic gallstones is usually laparoscopic
cholecystectomy.
Acute acalculous cholecystitis (choice A) is characterized by fever, nausea and
vomiting, right upper quadrant abdominal pain, and inspiratory arrest on
palpation of the right upper quadrant (Murphy's sign). An elevated leukocyte
count is usually present. Gallstones are not present and it is usually associated
with trauma, burn, surgery, diabetes mellitus, and bacterial infections of the
gallbladder.
Acute calculous cholecystitis (choice B) is characterized by fever, nausea and
vomiting, right upper quadrant abdominal pain, and inspiratory arrest on
palpation of the right upper quadrant (Murphy's sign). An elevated leukocyte
count is usually present. It is associated with gallstones.
Acute cholangitis (choice C) is characterized by fever, right upper quadrant
pain, and jaundice (Charcot's triad). This patient is afebrile and is not
jaundiced.
Primary sclerosing cholangitis (choice E) is a rare disease characterized by
inflammation, fibrosis, and strictures within the biliary system. It typically
presents with obstructive jaundice, malaise, anorexia, pruritus, and indigestion.
It is sometimes associated with ulcerative colitis. This patient's findings are
more consistent with biliary colic than sclerosing cholangitis.
A 65-year-old woman is admitted to the hospital with severe ascites and fever.
She has a 2-year history of portal hypertension secondary to hepatitis Cinduced cirrhosis. The patient was placed on the liver transplant waiting list 3
months ago. Four months prior to admission she suffered an upper
gastrointestinal bleed secondary to esophageal varices, which was
subsequently banded via endoscopy. Two days ago, the patient developed
abdominal pain, increasing abdominal girth, and fever. She was admitted to the
hospital with the diagnosis of spontaneous bacterial peritonitis. The
appropriate therapy is initiated and over the course of the next 4 days the
patient appeared to be responding well. On the day of discharge you begin to
plan her outpatient management and follow-up care. To prevent further
disability from her current acute condition, you should prescribe
A. hydrochlorothiazide
B. lactulose
C. levofloxacin
D. oral protein supplements
E. propranolol
Explanation:
The correct answer is C. The 1-year risk of recurrent infection in patients
surviving spontaneous bacterial peritonitis (SBP) is as high at 70%.
Prophylaxis has been tested in this group and shown to be efficacious. The rate
of recurrence is less with therapy, although there is no survival benefit. The
agents tested have mostly been fluoroquinolone antibiotics. Levofloxacin is
now the preferred agent in that class.
Hydrochlorothiazide (choice A) is a diuretic useful in controlling ascites. It is,
however, not a more important therapeutic intervention than SBP prophylaxis.
Oral lactulose (choice B) is indicated when the patient begins to suffer from
hepatic encephalopathy.
Oral protein supplements (choice D) are never indicated despite the clear
nutritional and total protein deficiency in these patients. The reasoning is that
the metabolized nitrogen products cannot be effectively cleared and often
precipitate episodes of hepatic encephalopathy.
Propranolol (choice E) is a non-selective beta-blocker used to prevent variceal
bleeding. This patient has no documented varices at this time and even if they
were present, the efficacy of propranolol for preventing bleeds is questionable.
A 47-year-old man comes to the office for follow-up management of his
ascites and cirrhosis. He has been your patient for 3 years. He has known
hepatitis B and alcohol-induced cirrhosis. He takes furosemide and
spironolactone daily. Six weeks ago he underwent an upper endoscopy that
showed grade 3 esophageal varices. You are interested in offering him therapy
to prevent a possible devastating upper gastrointestinal bleed. The most
appropriate management to prevent bleeding is to
A. begin to take a beta-1 selective blocker
B. begin to take a beta-2 selective blocker
C. continue to take his current medications
D. schedule endoscopic banding of the varices
E. schedule a transjugular intrahepatic portosystemic shunt (TIPS)
procedure
Explanation:
The correct answer is D. The options for preventing an initial UGI bleed due
to varices in patients are essentially nonselective beta blockade, endoscopic
sclerotherapy or banding, and shunt surgery. A tremendous number of clinical
trials on these options have been published. The current favored therapy is
endoscopic banding of any visible varices and subsequent endoscopy to band
any additional varices. The rate of initial bleeds is decreased by this
intervention but none of the interventions have yet to show any survival
advantage.
It is incorrect that the most effective prevention of a variceal bleed is a beta-1
selective blocker (choice A) or beta-2 selective blocker (choice B). As alluded
to in the discussion above, these are not superior to banding, and in fact, beta
blockade has not been clearly shown to even decrease risk of initial bleeding.
Continuing to take his current medications (choice C) is not the most effective
management to prevent bleeding. None of his current medications are effective
for prophylaxis. He should still take them though.
A TIPS procedure (choice E) is a form a shunting performed by interventional
radiologists. TIPS involves placing a "shunt" via a transjugular approach into a
deep branch of the hepatic vein coupled to a deep branch of the portal vein.
They are designed to relieve portal hypertension and thus decrease the risk of
bleeding. These procedures are used as a bridge to transplant in most centers,
and are rarely used in primary bleed prevention as the shunts tend to thrombose
and cause problems within one year of placement.
A 62-year-old man comes to the office for a periodic physical examination. He
has no complaints. His past medical history is significant for mild systolic
hypertension, non-insulin dependent diabetes mellitus, and atrial fibrillation.
He is taking enteric-coated aspirin 81 mg daily and warfarin for his atrial
fibrillation. He also reports that he is taking an herbal medicine for "strength
and vitality". Review of the herbal medicine package reveals that the medicine
contains iron and vitamins. Physical examination is unremarkable. Abdominal
examination is benign. Rectal examination reveals guaiac-positive stool. Rectal
examination and a repeat guaiac test in the subsequent 2 days reveals guaiacpositive stools. He denies any recent alteration of bowel habits or recent loss of
weight. The most appropriate next step in the management of this patient is
A. colonoscopy
B. observation
C. repeat guaiac test after stopping aspirin for 1 month
D. repeat guaiac test after stopping the iron-containing herbal medicine for
1 month
E. repeat guaiac test after stopping warfarin for 1 month
Explanation:
The correct answer is A. The likelihood that a guaiac-based test will be
positive is directly proportional to the quantity of fecal heme, which in turn is
related to the size and location of the bleeding lesion. Guaiac-based tests are
generally best at detecting large, more distal neoplasms. The guaiac test can be
affected by dietary factors such as meat and vegetables that contain peroxidase.
Iron by itself does not cause erroneous guaiac-positive stool. Iron in large
quantities can yield black colored stools, which might be difficult to
differentiate from positive guaiac with bluish coloration. Aspirin or warfarin
slightly increases the blood loss in the stool, but by themselves do not cause
guaiac-positive stools. Hence, guaiac-positive stools on 3 different occasions in
a 62-year-old patient, requires colonoscopy to rule out any colonic malignancy.
Observation (choice B) is not an option when guaiac-positive stools are
discovered on three different occasions.
A repeat guaiac test after stopping aspirin (choice C) is not essential; the
guaiac positive stools are not caused by aspirin.
A repeat guaiac test after stopping iron containing herbal medicines (choice D)
is not required, as iron by itself does not cause guaiac-positive stools.
A repeat guaiac test after stopping warfarin (choice E) is not essential, as
warfarin by itself, does not cause guaiac-positive stools.
A 61-year-old man with a history of ulcerative colitis comes to the clinic with
a 1-week history of abdominal distension and occasional nausea. He has also
had intermittent constipation and diarrhea for the past 3 weeks. Physical
examination reveals an obese male with a distended abdomen with normal
bowel sounds. The abdomen is diffusely tender to touch. There is no rebound
or hepatosplenomegaly. Rectal examination shows heme-negative stool. His
hematocrit is 44% and leukocyte count is 7000/mm3. The most appropriate
next step in the management of this patient is to
A. do a flexible sigmoidoscopy
B. insert a rectal tube
C. order an abdominal radiograph
D. prepare him for a total colectomy
E. send him for a colonoscopy
Explanation:
The correct answer is C. Patients with ulcerative colitis are at high risk for
toxic megacolon, which is also associated with Clostridium difficile colitis.
Toxic megacolon presents clinically as abdominal distension and bowel
motility disturbances. The next step in evaluation is an abdominal radiograph
which will demonstrate a distended large bowel. Toxic megacolon is the
leading cause of death in patients with ulcerative colitis and carries a 40%
mortality with each bout of toxic megacolon. Treatment consists of nasogastric
tube insertion, no oral food or drink, rectal tube insertion, antibiotics,
positional maneuvers, and ultimately surgery if there is no resolution in 2-5
days.
Flexible sigmoidoscopy (choice A) is a screening test used by some primary
care physicians to evaluate for left-sided colon masses. The sigmoidoscope is
unable to view the entire colon and specifically the right colon. Patients with
ulcerative colitis are at high risk for toxic megacolon which is also found with
Clostridium difficile colitis. Toxic megacolon presents clinically as abdominal
distension and bowel motility disturbances. The next step in evaluation is the
abdominal radiograph which will demonstrate a distended large bowel.
Rectal tube insertion (choice B) is a means to decompress a distended colon. A
diagnosis must first be established by a plain abdominal radiograph.
Total colectomy (choice D) is necessary in some patients with ulcerative colitis
because of multiple polyps or malignancies. Total colectomy is also necessary
in patients with severe toxic megacolon, as a last resort treatment. Nevertheless
an abdominal plain film is necessary first to evaluate if a distended colon is
present.
Periodic colonoscopy (choice E) is necessary to assess for the development of
colon cancer, which is much more common in patients with ulcerative colitis
than in the population at large. Patients with ulcerative colitis are at a high risk
for toxic megacolon which is also found with Clostridium difficile colitis.
Toxic megacolon presents clinically as abdominal distension and bowel
motility disturbances. The next step in evaluation is the abdominal radiograph
which will demonstrate a distended large bowel.
A 51-year-old woman with end-stage liver disease due to cryptogenic cirrhosis
is being cared for by your medical team. She was recently admitted for
increasing abdominal girth and confusion. She was diagnosed with cirrhosis
and portal hypertension 3 years ago and has long-standing ascites and 2
previous admissions for hepatic encephalopathy. She has no allergies. Her
current medications include oral lactulose, ofloxacin, spironolactone, and
furosemide. Over the past few days, the team has been attempting to reduce her
ascites by both repeated large-volume paracentesis and aggressive diuresis.
The patient has been having four to five bowel movements daily while on
lactulose. On reviewing the morning laboratory data, the following values are
noted:
Day 1: Sodium 126 mEq/l, Potassium 3.2 mEq/l, BUN 20 mg/dl, Creatinine
1.1 mg/dl
Day 2: Sodium 129 mEq/l, Potassium 3.5 mEq/l, BUN 29 mg/dl, Creatinine
1.4 mg/dl
Day 3: Sodium 134 mEq/l, Potassium 4.2 mEq/l, BUN 33 mg/dl, Creatinine
1.7 mg/dl
Day 4: Sodium 142 mEq/l, Potassium 4.8 mEq/l, BUN 41 mg/dl, Creatinine
2.1 mg/dl
Day 5: Sodium 148 mEq/l, Potassium 5.2 mEq/l, BUN 55 mg/dl, Creatinine
2.9 mg/dl
The most important management is to
A. administer sodium polystyrene sulfonate, orally
B. bolus 500cc of normal saline
C. discontinue the furosemide
D. discontinue the spironolactone
E. obtain a 12 lead electrocardiogram
Explanation:
The correct answer is D. Aggressive diuresis and fluid management is the
mainstay of therapy for ascites related to liver failure. In the case of this
patient, the aggressive diuresis has caused progressive renal insufficiency. The
potassium sparing diuretic spironolactone is only worsening the hyperkalemia
and should be promptly discontinued until the patient's renal function returns
to normal.
Administering sodium polystyrene sulfonate orally (choice A) would be
indicated if a 12 lead EKG suggested evidence of hyperkalemia (QRS changes,
T wave elevations). Kayexalate is a potassium binding resin used to lower
serum potassium.
Bolus 500cc of normal saline (choice B) may help with the worsening prerenal failure, but will not reverse the problem in a definitive manner.
Discontinuing the furosemide (choice C) is not the primary diuretic to address,
since by keeping spironolactone, the potassium elevation will likely only
worsen.
Obtaining a 12 lead electrocardiogram (choice E) is not unwise but given that
the potassium is not severely elevated, discontinuing the offending diuretic is
the most important concept to grasp. Blindly checking an EKG for a
marginally elevated potassium misses the point of a rising creatinine and
potassium in the setting of aggressive diuresis.
You are on call for the medical services and a nurse pages you to ask you to
come speak with the family of one of the patient's. The patient is not your own,
but one that you are cross covering for the evening. You know that the patient
carries a new diagnosis of widely metastatic pancreatic cancer. On arrival to
the floor, three members of the patient's family greet you. They are somewhat
confrontational and are very insistent that their father not be told of his
diagnosis. When you attempt to discuss with the family the reasons for this,
they continue to demand that you agree and threaten to sue you and the
hospital if you or any other members of the team inform their father, the
patient, of his diagnosis. The most appropriate course of action at this time is
to
A. defer any discussion on the point until you talk with other members of
the team
B. tell the family that the patient will be told his diagnosis and they need to
be more supportive of their father
C. tell the family that the physician caring for the patient will talk with
them in the morning
D. tell the family that you will agree to their demands
E. tell the family that your obligation is to the patient, not to them
Explanation:
The correct answer is A. Situations such as these are all too common in
modern hospitals. The combination of family protection of loved ones and the
terminal diagnoses often combine into a very volatile mix. Since this is not
your patient and you have no relationship with the patient or the family,
attempting to discuss this situation is a poor idea. The family is frightened and
is attempting to gain some control over their situation. Their confronting the
cross-covering physician is simply a means to take that control. Before you
proceed, you should talk with other members of th team.
Telling the family that the patient will be told his diagnosis and they need to be
more supportive of their father (choice B) will only inflame the situation and
perhaps add an element of guilt or anxiety that will make the family or the
patient want to leave the hospital against medical advice.
Telling the family that the physician caring for the patient will talk with them
in the morning (choice C) will likely make the family feel as if they are being
ignored or brushed aside.
Telling the family that you will agree to their demands (choice D) is ill-advised
since as the patient is not yours, you have no right to speak for the primary
caregivers in this case.
Telling the family that your obligation is to the patient, not to them (choice E)
will certainly inflame this situation tremendously and is ill-advised. Although
the statement may be technically correct, it will do no good to say such a thing
since there is no reason that a cross-covering physician would have to inform
the patient of their diagnosis.
A 72-year-old veteran comes to the emergency department with a 2-day history
of abdominal pain that is worse at night and with eating. He recalls that he has
been more constipated than usual lately and has had some mild nausea with
meals for 6 months but he “didn't really think anything of it". His past medical
history is significant for coronary artery disease including a bypass 3 years
ago, claudication, emphysema, gout, and obstructive sleep apnea. He had a
right inguinal hernia repair, cholecystectomy, and multiple podiatric
procedures over the past 30 years. His temperature is 38.1 C (100.6 F), blood
pressure is 150/100 mmHg, pulse is 100/min, and respirations are 12/min.
Physical examination shows an obese man with moderate epigastric and left
lower quadrant pain. There is a suggestion of a pulsatile left lower quadrant
mass. There is no rebound tenderness. Upright, decubitus, and supine plain
radiographs of the abdomen are normal. An electrocardiogram reveals normal
sinus rhythm and mild left ventricular hypertrophy. The next step in the
emergency department is to
A. initiate ciprofloxacin and ampicillin therapy
B. initiate ciprofloxacin and metronidazole therapy
C. order a CT scan of the abdomen and pelvis
D. prepare him for an immediate laparotomy
E. send him for an ultrasound of the abdomen and appendix
Explanation:
The correct answer is C. The differential diagnosis based on history and
physical exam includes diverticulitis, aortic aneurysm, ruptured aortic
aneurysm, bowel obstruction, or an abdominal mass. Plain films of the
abdomen exclude obstruction. Further evaluation with a CT is needed to
exclude an aneurysm before antibiotic therapy is initiated. Moreover, a CT
scan will evaluate for diverticulitis or a diverticular abscess as well as the less
likely possibility of an abdominal mass such as colon cancer.
Ciprofloxacin and metronidazole treatment or ampicillin, gentamicin, and
metronidazole should be used to conservatively treat diverticulitis.
Ciprofloxacin and ampicillin (choice A) would be inadequate anaerobic
coverage.
Ciprofloxacin and metronidazole (choice B) would be good conservative
treatment for diverticulitis, but other items on the differential diagnosis must
first be discarded before discharging the patient.
A laparotomy (choice D) is indicated only in the case of perforation with
peritonitis. Perforation can be diagnosed on plain films of the abdomen by the
presence of free air under the diaphragm.
An ultrasound (choice E) is inferior to a CT scan for evaluating the bowel and
aorta. Also, since he is febrile, it seems more likely that he has diverticulitis
than an aneurysm, so a CT scan is preferable.
A 21-year-old, otherwise healthy man comes to emergency department with
midabdominal pain for 12 hours. He states that now the pain is localized to the
right lower quadrant. He feels very nauseated and has been unable to eat since
the pain began. His last bowel movement was earlier this morning, before the
pain began. His temperature is 37.0 C (98.6 F), blood pressure is 124/78 mm
Hg, and pulse is 78/min. Abdominal examination shows right lower quadrant
tenderness with involuntary guarding. Rectal examination shows tenderness in
the right lateral wall. Laboratory studies are normal except for a mild
leukocytosis. The most appropriate next step in management is to
A. order abdominal ultrasonography
B. order a CT scan of the abdomen and pelvis
C. observe him in the emergency department
D. prepare him for an immediate operation
E. send him for a barium enema
Explanation:
The correct answer is D. The history and examination findings in this young
male patient are classical for appendicitis. There are very few other causes of
right lower quadrant tenderness in a young male like renal calculi or Crohns
disease. History is helpful to rule out the other causes. It is acceptable to have a
negative exploration rate of 15% while operating for a suspected appendicitis.
In a female patient, other pelvic pathology related to ovaries, fallopian tubes,
or uterus need to be ruled out.
Abdominal ultrasonography (choice A) efficacy is subjective in diagnosing
appendicitis, as they are technician dependent. In the best of the hands, an
ultrasonography is 80-85% sensitive in diagnosing appendicitis. In a male
patient, with classical signs and symptoms of acute appendicitis, further
investigation is not justified.
In the era of the CT scan (choice B), a barium enema (choice E) is rarely
performed. A barium enema is diagnostic when the appendix is not filled up
with contrast. In a male patient with classical signs of acute appendicitis, a CT
scan and a barium enema will only delay the diagnosis.
Observation (choice C) is not necessary, when history and clinical examination
point out appendicitis. Delay in diagnosis and treatment in a patient with
inflammation in the abdomen or pelvis can result in significant mortality and
morbidity by the spread of infection or adhesions.
A 45-year-old man who is admitted to the hospital for alcohol detoxification
develops a bloody emesis on hospital day 3. Over the past hour there has been
approximately 500 cc of bloody emesis. The patient has a history of alcoholism
and hepatitis C. Vital signs are: temperature 37 C (98.6 F), blood pressure
100/80 mm Hg, pulse 122/min, and respirations 9/min. The patient is oriented
and answers questions normally. Physical examination reveals a thin,
jaundiced man in a mildly lethargic state. He has mild ascites, a caput medusa,
and lower extremity edema. The most urgent clinical issue that should be
addressed is
A. cirrhosis
B. delirium tremens
C. esophageal carcinoma
D. gastric carcinoma
E. portal vein thrombosis
F. variceal hemorrhage
Explanation:
The correct answer is F. This patient has a history and physical exam
consistent with a history of alcoholism and hepatitis C. These are both risk
factors for cirrhosis of the liver. Upper gastrointestinal bleeding in the form of
bloody emesis should immediately bring the thought of variceal hemorrhage to
mind. Endoscopy is necessary to evaluate for variceal hemorrhage or peptic
ulcer disease.
This patient is at a high risk for cirrhosis (choice A) and already has physical
exam findings of cirrhosis such as ascites and a caput medusa. Cirrhosis in
itself is insidious, leading to liver failure, an immunocompromised state, and a
higher risk for hepatocellular carcinoma. A consequence of cirrhosis is portal
hypertension and esophageal varices. This is the most urgent issue that needs
to be addressed.
Delirium tremens (choice B) may be life threatening if untreated, but this
patient has no symptoms of alcohol withdrawal or seizures at this time.
Esophageal cancer (choice C) is in the differential for upper gastrointestinal
bleeding, but in this patient with risk factors for cirrhosis, variceal bleeding is
the first concern.
Gastric carcinoma (choice D) is in the differential for upper gastrointestinal
bleeding, but in this patient with risk factors for cirrhosis, variceal bleeding is
the first concern.
Portal vein thrombosis (choice E) may be associated with systemic or local
infection such as cholangitis, adjacent suppurative lymphadenitis, pancreatitis,
and a hepatic abscess. It is also seen in patients in hypercoagulable states.
Portal vein thrombosis occurs in 10% of patients with cirrhosis and frequently
accompanies hepatocellular carcinoma. Portal vein thrombosis is not
symptomatic itself, but it can lead to complications such as esophageal varices.
The urgent issue that needs to be addressed in this patient is the evaluation of
the hematemesis.
A 29-year-old woman comes to the emergency department because of a 3-hour
history of severe left-sided abdominal pain. She says that the pain came on
suddenly when she was watching her favorite television show. She has no
medical problems and takes no regular medications. She is sexually active with
2 different partners and "usually" uses condoms for contraception. Her last
menstrual period began 10 days ago. Her temperature is 37.0 C (98.6 F), blood
pressure is 120/80 mm Hg, pulse is 80/min, and respirations are 20/min.
Physical examination shows moderate left lower quadrant tenderness with no
guarding or rebound tenderness. Rectal examination shows soft, brown guaiac
negative stool. A urine pregnancy test is negative. The most appropriate next
step is to
A. begin antibiotic therapy, intravenously
B. obtain a pelvic ultrasound
C. order a CT scan of the abdomen
D. order quantitative B hCG levels
E. perform a pelvic examination
Explanation:
The correct answer is E. This patient has acute abdominal pain that may be
caused by an abdominal or pelvic process. A pelvic exam is the most
appropriate next step to help to differentiate the cause of her pain. Torsion of
an ovarian cyst will present like this, and pelvic examination may help to
reveal a palpable mass. Pelvic inflammatory disease is another possibility and
a pelvic examination may reveal cervical motion tenderness and a vaginal
discharge. She would most likely be febrile if this was the cause. Other
possible causes are renal calculi and diverticulitis (unlikely given her age).
Antibiotic therapy (choice A) is appropriate if she had pelvic inflammatory
disease, however you would first need to do a pelvic examination.
A pelvic ultrasound (choice B) may be appropriate if an ectopic pregnancy is
suspected, however the pregnancy test was negative.
A CT scan (choice C) is sometimes used to establish the diagnosis of
appendicitis, however this patient has left-lower quadrant pain, which makes
this diagnosis unlikely. A CT scan is also used to evaluate diverticulitis, but
given her young age, chances are that this pain is caused by a gynecologic
process. A pelvic exam is necessary.
Quantitative B hCG levels (choice D) are typically followed in patients with
positive urine pregnancy tests who have a suspected ectopic pregnancy. These
women often present with abdominal pain, vaginal bleeding, and amenorrhea.
This patient does not have these symptoms and her urine pregnancy test is
negative.
A 9-year-old girl is brought to the office by her mother because of "stomach
aches" and constipation. She has been having one painful, hard bowel
movement every 4 to 5 days. She admits that she never goes to the bathroom in
school because she is too embarrassed, so she "holds it in until she is at home."
Many times she is so busy with after school activities such as ballet, piano, and
gymnastics, that the "feeling" often passes by the time she gets home. The
mother tells you that she complains of abdominal pain when this occurs, but it
is too painful to defecate, so she continues to hold it in. She does not take any
medications, has no medical illness, and has had normal bowel habits until 6
months ago. Physical examination shows mild abdominal tenderness, a hard
mass in the lower abdomen, and a dilated rectum filled with a large amount of
hard, guaiac negative, brown stool. The most appropriate next step is to
A. obtain a consult with a pediatric gastroenterologist
B. obtain an x-ray of the abdomen
C. recommend a balanced diet containing whole grains, fruits, and
vegetables
D. recommend a phosphate soda enema
E. schedule anal manometry
Explanation:
The correct answer is D. This patient has fecal impaction that is most likely
caused by school bathroom avoidance. Fecal impaction can be diagnosed by
finding a hard mass in the lower abdomen and a dilated rectum filled with a
large amount of hard stool. This often occurs because of fecal stasis in the
colon leading to an increased absorption of fluids, which in turn leads to the
accumulation of large, hard stools that are painful to pass. This pain often leads
to avoidance of defecation, even if the child is at home. The treatment includes
disimpaction by oral or rectal medications, dietary and behavior modification.
Disimpaction must be done before starting maintenance therapy.
A consult with a pediatric gastroenterologist (choice A) is usually only
necessary after the child fails therapy, when an organic disease is suspected, or
when there is complex management. This case seems like a simple case of
school bathroom avoidance that can be treated with disimpaction, and dietary,
behavior modification. If this fails, a consult may be considered.
An x-ray of the abdomen (choice B) is not necessary to establish the diagnosis
of fecal impaction if the physical examination reveals a hard mass in the lower
abdomen and a dilated rectum filled with a large amount of hard brown stool.
It may be necessary, if the child refuses a rectal examination.
After a thorough history, physical examination, and disimpaction, you should
recommend a balanced diet containing whole grains, fruits, and vegetables
(choice C) and behavior modification to promote healthy bowel habits.
Anal manometry (choice E) is used in the evaluation of Hirschsprung disease,
which is a cause of constipation in infants and rarely in school-age children. It
is caused by a lack of colonic ganglion cells. Physical examination usually
reveals a distended abdomen, a contracted anal sphincter, and a rectum devoid
of stool. The physical exam of the child in this case is inconsistent with
Hirschsprung disease. Also, she admits to school bathroom avoidance, which is
probably the cause of her impaction.
A 58-year-old homeless man is brought to the emergency department with
severe hematemesis. He has a history significant for severe alcohol abuse and
significant esophageal varices with bleeding in the past. You notice in his old
chart that it was recommended that he take a multivitamin, folate, and
thiamine. His blood pressure is 100/50 mmHg, pulse is 105/min, and
respiratory rate is 26/min. Physical examination shows coarse breath sounds
and a protuberant abdomen. Nasogastric lavage yields fresh blood. Given that
you strongly suspect another variceal bleed, the most appropriate next step in
the management of this patient is
A. administration of amiodarone, intravenously
B. administration of nadolol, orally
C. administration of nitroglycerin, intravenously
D. administration of octreotide, intravenously
E. administration of phenylephrine, intravenously
Explanation:
The correct answer is D. Octreotide, a synthetic somatostatin analog, is used
in the acute management of variceal bleeding. It decreases variceal bleeding by
decreasing splanchnic blood flow and decompressing the portal system.
Amiodarone (choice A), an anti-dysrhythmic agent, has no role in the
management of a variceal hemorrhage.
Oral nadolol (choice B) has been shown to be of significant benefit in the
primary and secondary prevention of variceal bleeding, but plays no role in its
acute management. A beta antagonist should also be used in the setting of
acute hemorrhage with extreme caution.
Intravenous nitroglycerin (choice C) plays no role in the acute management of
a variceal bleed. This is despite evidence that nitrates play a role in the primary
and secondary prevention of variceal bleeding in combination with beta
antagonists. Vasodilators such as nitroglycerin are also contraindicated with
hemorrhage.
Phenylephrine (choice E), an alpha agonist vasopressor, has no role in the
management of variceal hemorrhage. This patient's hemodynamics also do not
warrant vasopressor therapy.
A 10-year-old boy is admitted to the pediatrics unit with rectal bleeding and
right lower quadrant abdominal pain. He has no significant past medical
history. Vital signs are: temperature 37.2 C (99 F), blood pressure 90/40 mm
Hg, pulse 80/min, and respirations 11/min. The physical examination is
normal. Rectal examination reveals bright red blood, but no other
abnormalities. A colonoscopy extending to the ileocecal valve is normal except
for a moderate amount of fresh blood. The next step in managing this patient is
to order a(n)
A. abdominal angiography
B. nuclear medicine technetium scan
C. sigmoidoscopy
D. small bowel follow through
E. upper gastrointestinal endoscopy
Explanation:
The correct answer is B. Lower gastrointestinal bleeding in a child with a
negative endoscopy is suspicious for a Meckel's diverticulum (MD). A MD
occurs in 2% of the population and 2% are symptomatic. They occur 2 feet
from the ileocecal valve, are usually 2 inches in length, and contain 2 types of
mucosa (gastric and pancreatic). A MD is a true diverticulum from the
antimesenteric border of the small bowel, and is the most common congenital
abnormality of the gastrointestinal tract. It is usually asymptomatic, but may
develop symptoms, usually before the age of 12. Bleeding may either be
pronounced, as in this case, or present as a subtle anemia. A technetium-99m
pertechnetate scan is about 90% accurate in its diagnosis. Presentation may be
bleeding (50%) or obstruction (25%).
Abdominal angiography (choice A) is only useful in the unstable patient with a
rising pulse and potentially, falling blood pressure. In this case, the patient is
hemodynamically stable, and a nuclear medicine technetium scan is
appropriate.
Sigmoidoscopy (choice C) is not necessary, as it will offer no more
information than the previously performed colonoscopy.
A small bowel follow through (choice D) sometimes reveals a Meckel's
diverticulum in asymptomatic patients. It is not the study of choice in this
symptomatic patient.
Upper gastrointestinal endoscopy (choice E) would not be appropriate as this
patient is having lower gastrointestinal bleeding.
A 52-year-old man is brought to the emergency department by his wife
because he has had "bright red blood pouring from his mouth" for the past 20
minutes. The wife tells you that he has a 4-year history of alcoholic cirrhosis
and he continues to drink 1 or 2 beers per day. He also has hypertension and
hypercholesterolemia. Two days prior to admission, he had an episode of
hematemesis and this morning, had an additional episode. He is diaphoretic
with a blood pressure of 80/50 mm Hg and pulse of 110/min. Physical
examination shows scleral icterus and mild jaundice, a tense abdomen, and
cool, moist extremities. The most appropriate immediate action is to
A. begin large volume intravenous fluids
B. insert a Minnesota tube
C. perform an emergent portal-systemic surgical shunt
D. provide intravenous pressors for blood pressure control
E. send a blood bank sample for type and crossmatching
Explanation:
The correct answer is A. The first priority in the management of any patient
with GI bleeding is hemodynamic resuscitation and stabilization. IV fluids
should be given immediately to maintain hemodynamic stability and adequate
urinary output.
A Minnesota tube (choice B) is a type of orogastric tube with a gastric and
esophageal balloon used for tamponade bleeding varices. They have fallen out
of favor primarily due to the availability of endoscopes as well as the issue of
pressure necrosis of the esophagus when they are utilized.
An emergent portal-systemic surgical shunt (choice C) would be a therapy of
last resort if the bleeding cannot be controlled by less invasive measures.
Intravenous pressors for blood pressure control (choice D) is not indicated
unless the patient proves refractory to therapy for his initial problem that is
volume loss due to GI bleed. Therapy is volume resuscitation with blood and
fluid.
A blood bank sample for type and crossmatching (choice E) is next on the
priority list once the patient is hemodynamically stable.
You are seeing a 41-year-old man with alcoholic cirrhosis in your office for a
follow-up visit after a recent upper endoscopy showed significant lower
esophageal varices. His current medications include a multivitamin, folate, and
thiamine. While he strongly denies any continued alcohol use, you are
suspicious that he is still drinking. His blood pressure is 100/63 mmHg, pulse
is 98/min, and respirations are 21/min. Physical examination shows a slightly
protuberant abdomen. Given his varices, you are concerned about an upper
gastrointestinal bleed, especially in the setting of continued alcohol use. Given
this concern, the most appropriate pharmacotherapy to add to his treatment
regimen is
A. aspirin
B. atorvastatin
C. isosorbide mononitrate
D. nadolol
E. warfarin
Explanation:
The correct answer is D. Non-selective beta antagonists such as nadolol have
been shown to decrease the risk of an initial variceal bleed (through a reduction
in splanchnic blood flow) in someone with esophageal varices (primary
prevention).
Aspirin (choice A), an antiplatelet agent, has no role in the primary prevention
of variceal bleeding.
Atorvastatin (choice B), an HMG Co-A Reductase antagonist used in the
management of hyperlipidemia, has no role in the primary prevention of
variceal bleeding.
Nitrates such as isosorbide mononitrate (choice C) should not be started as
monotherapy for the primary prevention of variceal bleeding in cirrhotics since
it has been associated with increased mortality when used alone. However, it
can be used in combination with beta antagonists.
Warfarin (choice E), an oral anticoagulant against vitamin K dependent
clotting factors, has no role in the primary prevention of variceal bleeding.
An 8-month-old infant, who you have been taking care of since he was born at
the local community hospital, is brought to the office because of constipation.
The father, a "stay-at-home dad," tells you that she has been having 1 bowel
movement every 3-4 days, and that the stool is always very hard. He says that
she is doing very well otherwise; she is a very happy and easy little girl. She is
fed primarily infant formula and he is starting to introduce solid foods. He says
that he is concerned because he remembers always having to change "very
dirty" diapers for both of his other children, at least twice a day. Physical
examination is unremarkable. A rectal examination shows guaiac negative
brown stool. The most appropriate next step is to
A. advise him to give her mineral oil 3 times a day until she is "regular"
B. advise him to give her prune juice or pear juice
C. determine thyroid-stimulating hormone levels
D. order a barium enema
E. order rectal manometry and a rectal biopsy
F. reassure him that all infants have different bowel habits
Explanation:
The correct answer is B. Constipation is a common problem in formula-fed
infants, and it is best treated by increasing the amount of fluids in the diet,
especially with fruit juices that contain sorbitol, such as prune and pear, which
help to relieve constipation. It is often caused by a diet that is too low in fluids
or deficient of bulk.
It is not recommended to give mineral oil (choice A) to infants because of the
risk of aspiration and severe lipoid pneumonia.
Determining TSH levels (choice C) is not appropriate at this time because this
is a common problem in formula-fed infants that is related to diet.
Hypothyroidism is associated with constipation, however, newborns born in
United States hospitals are routinely screened for this disorder, which can also
lead to mental retardation. It is unlikely that she has hypothyroidism.
A barium enema (choice D), rectal manometry and a rectal biopsy (choice E)
are all part of the evaluation of patients with suspected Hirschprung's disease,
which is a lack of ganglion cells in the distal colon. It is very unlikely that this
infant has this relatively rare (1 in 5000) disease that is often suspected if a
newborn does not have stool in the first 24 hours of life. Infants with this
disease often have a distended abdomen and a rectum that is devoid of stool. It
is more likely that this formula-fed infant has diet-related constipation.
Reassurance (choice F) is incorrect in this case because formula-fed infants
this age should be having 1-2 bowel movements a day. You can reassure him
that this is very common and she will most likely be fine once they adjust her
diet.
A 36-year-old woman comes to the office because of a 3-day history of
"yellow skin," fever, and abdominal pain. The pain is mostly present in the
right upper quadrant. However she sometimes feels it in her right shoulder. She
has had several similar episodes in the past, but they were not accompanied by
fever, and skin discoloration. She is married and has 3 children, none of whom
are sick. Her temperature is 39.3 C (102.7 F), blood pressure is 110/70 mm Hg,
pulse is 70/min, and respirations are 20/min. Physical examination shows right
upper quadrant tenderness. She has the "chills", but she continues to breathe
normally during right upper quadrant palpation. Laboratory studies show
The most likely diagnosis is
A. acute cholangitis
B. acute cholecystitis
C. acute hepatitis
D. acute pancreatitis
E. biliary colic
Explanation:
The correct answer is A. This patient has fever, jaundice, and right upper
quadrant abdominal pain, Charcot's triad, which is usually diagnostic of acute
cholangitis. This typically occurs because of a stone impacted within the
common bile duct. Blockage of this duct results in cholestatic jaundice, hence
the elevated bilirubin, and high alkaline phosphatase. Gram-negative bacteria
penetrate into the biliary ducts, and cause inflammation with leukocytosis and
fever. The pain is due to gallbladder distention. The treatment involves
antibiotics and surgery.
Acute cholecystitis (choice B), is inflammation of the gallbladder and the
obstruction of the cystic duct by a gallstone. Symptoms include right upper
quadrant pain, a mild fever, and possibly nausea and vomiting. Physical
examination shows right upper quadrant pain with inspiratory arrest during
palpation, (Murphy's sign). Mild jaundice may occur. The leukocyte count is
elevated. Treatment includes antibiotics and surgery.
Acute hepatitis (choice C), manifests with mild right upper quadrant
abdominal pain, nausea, anorexia, and a low-grade fever. Serum alanine
aminotransferase and aspartate aminotransferase are markedly elevated,
helping to distinguish this from cholangitis.
Acute pancreatitis (choice D), typically presents with intense midepigastric
pain, usually radiating to the back, fever, nausea, and vomiting. Very high
levels of amylase and lipase will support the diagnosis. The treatment usually
involves insertion of a nasogastric tube, intravenous fluids, and electrolyte
replacement.
Biliary colic (choice E), is episodic right upper quadrant abdominal pain that
radiates to the back, and may be associated with nausea and vomiting. It often
occurs after a meal. It is caused by a transient blockage of the cystic duct with
a gallstone. It is not usually associated with fever or jaundice.
A 78-year-old woman comes to the geriatric clinic for a follow-up
appointment. She was seen 3 weeks ago in the clinic for a routine appointment
and was found to have a hematocrit of 28%. A rectal examination was positive
for heme in the stool. Her only complaint is a long history of constipation. She
has multiple medical problems including diabetes, hypertension, osteoarthritis,
and a history of a myocardial infarction many years ago. To further evaluate
her anemia, additional laboratory testing was initiated at that time. Since her
last appointment, she had an outpatient barium enema and is now returning for
the results of all her tests. An x-ray of the recto-sigmoid colon from the barium
enema examination is shown. Serum laboratory tests are as follows:
At this time, the most appropriate next step is to
A. admit the patient to the hospital for further evaluation
B. do a colonoscopy
C. encourage the patient to eat a high fiber diet
D. order serum carcinoembryonic antigen (CEA-125)
E. prescribe iron supplements
Explanation:
The correct answer is B. A colonoscopy is the most appropriate next step in
the management of this patient. Her laboratory tests reveal a microcytic anemia
with a low ferritin, which is consistent with an iron deficiency anemia. This is
a common presentation of colon cancer. The barium enema demonstrates an
irregular lesion in the sigmoid colon that is highly suspicious for an
adenocarcinoma of the colon. A colonoscopy can be performed to evaluate the
entire length of the colon for polyps and masses. It can better characterize the
extent of the lesion detected on the barium enema, as well as allow for a biopsy
of the lesion for definitive tissue diagnosis.
Admitting the patient to the hospital for further evaluation (choice A) is not
necessary in this situation. Patients with colon cancer who present with
symptoms of bowel obstruction or perforation will need to be hospitalized to
treat their acute disease. This patient is presenting with more indolent
symptoms of colon cancer such as iron deficiency anemia and chronic
constipation. She can be safely evaluated as an outpatient.
Encouraging the patient to eat a high fiber diet (choice C) is not appropriate
management. She has a lesion on the barium enema that is highly suspicious
for malignancy and requires further evaluation with colonoscopy. A high fiber
diet is recommended for patients with constipation and for the prevention of
diverticulosis.
Ordering a serum carcinoembryonic antigen (CEA-125) (choice D) is not the
best step in the management of this patient. CEA-125 is a serum glycoprotein
frequently used in the management of patients once they are diagnosed with
colon cancer. CEA-125 is not a useful screening test for colorectal cancer due
to the large numbers of false-positive and false-negative reports. CEA-125
testing in postoperative patients should be restricted to patients who will be
candidates for resection, in cases of liver or lung metastases. Routine use of
CEA-125 alone for monitoring responses to treatment is not recommended.
Prescribing iron supplements (choice E) is not the appropriate management of
this patient. Her iron deficiency anemia is a secondary sign of her primary
diagnosis of colon cancer. The lesion on the barium enema is highly suspicious
for malignancy and needs further evaluation with colonoscopy. Iron
supplementation is good adjuvant care for this patient, but it does not address
the more immediate need to make a diagnosis.
A 78-year-old nursing home resident is admitted to the hospital because of
increasing left-sided abdominal pain for the past 48 hours. She has had several
episodes of bloody diarrhea according to the nursing attendant at the nursing
home. There was no associated fever or nausea or vomiting. On admission, her
temperature is 37.3 C (99.1 F), blood pressure is 90/64 mm Hg, and pulse is
100/min. Her abdomen is soft and mildly distended without masses or
organomegaly. There is moderate tenderness to palpation in the left lower
quadrant, but no associated peritoneal signs. Rectal examination reveals
guaiac-positive stool and no masses. A flexible sigmoidoscopic examination
reveals patchy, depigmented mucosa. The most appropriate initial management
of this patient is
A. angiographic embolization of the inferior mesenteric artery
B. intravenous fluid and bowel rest
C. mesenteric angiogram
D. sigmoid resection and colostomy
E. subtotal colectomy
Explanation:
The correct answer is B. Abdominal pain in an elderly patient associated with
bloody diarrhea and hypotension should arouse the suspicion of ischemic
bowel. In this patient, ischemic bowel is precipitated by dehydration and
hypotension. A classical appearance on the flexible sigmoidoscopy of green
mucosa and isolated depigmented patches are suggestive of ischemic colitis.
These patients should initially be adequately hydrated and put on bowel rest.
Mucosal ischemia sometimes will improve with these measures and further
therapeutic measures may not be necessary.
Mesenteric angiogram and embolization (choice A) is not essential in this
patient. The bloody diarrhea is from mucosal slough injury from the ischemia
and will not be prevented by embolization.
An angiogram (choice C) is not necessary in the management of ischemic
mucosal colitis. Ischemic colitis is diagnosed by colonoscopy. Patchy
depigmented areas confirm mucosal ischemia, which is managed by
intravenous fluids and bowel rest. Transmural ischemia is confirmed by green
sloughing mucosa. Resection of the colon is determined by the extent of
ischemia. A mesenteric angiogram may be useful in diagnosing mesenteric
occlusion, but is not essential, as it is invasive, and diagnosis and guidelines
for resection can be obtained by colonoscopy.
A sigmoid resection and colostomy are indicated (choice D) provided that
ischemic colitis is not controlled and the patient is developing systemic signs
of sepsis, which she does not currently seem to have.
Isolated pigmented ischemic colitis in the sigmoid colon (choice E) would not
require a subtotal abdominal colectomy in an elderly patient.
A 78-year-old nursing home resident is admitted to the hospital because of
increasing left-sided abdominal pain for the past 48 hours. She has had several
episodes of bloody diarrhea according to the nursing attendant at the nursing
home. There was no associated fever or nausea or vomiting. On admission, her
temperature is 37.3 C (99.1 F), blood pressure is 90/64 mm Hg, and pulse is
100/min. Her abdomen is soft and mildly distended without masses or
organomegaly. There is moderate tenderness to palpation in the left lower
quadrant, but no associated peritoneal signs. Rectal examination reveals
guaiac-positive stool and no masses. A flexible sigmoidoscopic examination
reveals patchy, depigmented mucosa. The most appropriate initial management
of this patient is
A. angiographic embolization of the inferior mesenteric artery
B. intravenous fluid and bowel rest
C. mesenteric angiogram
D. sigmoid resection and colostomy
E. subtotal colectomy
Explanation:
The correct answer is B. Abdominal pain in an elderly patient associated with
bloody diarrhea and hypotension should arouse the suspicion of ischemic
bowel. In this patient, ischemic bowel is precipitated by dehydration and
hypotension. A classical appearance on the flexible sigmoidoscopy of green
mucosa and isolated depigmented patches are suggestive of ischemic colitis.
These patients should initially be adequately hydrated and put on bowel rest.
Mucosal ischemia sometimes will improve with these measures and further
therapeutic measures may not be necessary.
Mesenteric angiogram and embolization (choice A) is not essential in this
patient. The bloody diarrhea is from mucosal slough injury from the ischemia
and will not be prevented by embolization.
An angiogram (choice C) is not necessary in the management of ischemic
mucosal colitis. Ischemic colitis is diagnosed by colonoscopy. Patchy
depigmented areas confirm mucosal ischemia, which is managed by
intravenous fluids and bowel rest. Transmural ischemia is confirmed by green
sloughing mucosa. Resection of the colon is determined by the extent of
ischemia. A mesenteric angiogram may be useful in diagnosing mesenteric
occlusion, but is not essential, as it is invasive, and diagnosis and guidelines
for resection can be obtained by colonoscopy.
A sigmoid resection and colostomy are indicated (choice D) provided that
ischemic colitis is not controlled and the patient is developing systemic signs
of sepsis, which she does not currently seem to have.
Isolated pigmented ischemic colitis in the sigmoid colon (choice E) would not
require a subtotal abdominal colectomy in an elderly patient.
A 64-year-old man comes to the emergency department because of a 24-hour
history of severe left-sided lower abdominal pain and shaking chills. He denies
any diarrhea or blood per rectum. He has no significant past medical history
and takes no medications. His temperature is 38.3 C (101.0 F), blood pressure
is 120/80 mm Hg, pulse is 75/min, and respirations are 15/min. Physical
examination shows a tender left lower quadrant with guarding. There is no
rebound tenderness present. A rectal examination shows brown, guaiacnegative stool. Laboratory studies show:
The most appropriate next step is to
A. admit him to the hospital and prepare him for an immediate
hemicolectomy
B. order a barium enema
C. order a CT scan of the abdomen
D. perform a colonoscopy
E. recommend a high-fiber diet
Explanation:
The correct answer is C. This patient most likely has diverticulitis, which is
caused by a microperforation of a diverticula (or outpouching) in the colon.
The disease is thought to be due to a low-fiber diet. A CT scan will show
thickening of the bowel wall and possibly an abscess or a fistula. Treatment
includes an antibiotic, such as cefoxitin, which has good coverage of enteric
colonic flora. This patient should be made nil per os (NPO) and given
intravenous hydration.
Admitting him to the hospital, and preparing him for an immediate
hemicolectomy (choice A) is inappropriate at this time. This patient most
likely has diverticulitis and should have a CT scan. If there are findings
consistent with diverticulitis, he should be made NPO, given intravenous
hydration, and started on antibiotics. Surgery is usually only indicated for
repeated episodes of diverticulitis, disease unresponsive to medical therapy, or
if a complication, such as an obstruction, abscess or formation, or peritonitis
occurs.
A barium enema (choice B) and a colonoscopy (choice D) should not be
ordered during the acute stage of diverticulitis because they may cause a free
perforation.
Recommending a high-fiber diet (choice E) is appropriate long-term
management after the patient recovers from the acute phase of diverticulitis. It
is not appropriate management at this time. This patient requires acute
treatment.
A 16-year-old boy comes to the office because he recently noticed that his eyes
and skin are yellow. He denies any abdominal pain, nausea, vomiting, fevers,
chills, diarrhea, or sick contacts. He does admit to a large amount of stress
because his SAT examination is in 2 weeks and he is concerned about getting
into college. He denies IV drug use and has never been sexually active. He
reports that he "became yellow" 3-4 times before during periods when he had
viral illnesses. His temperature is 37.0 C (98.6 F), blood pressure is 120/70 mm
Hg, pulse is 73/min, and respirations are 13/min. Physical examination shows
scleral icterus. His abdomen is soft, non-tender, non-distended, with normal
bowel sounds. There is no hepatomegaly. Laboratory studies show:
The most appropriate next diagnostic step is to
A. do no further studies at this time
B. order a HIDA scan
C. perform an abdominal ultrasound
D. send hepatitis serology
E. schedule a liver biopsy
Explanation:
The correct answer is A. This patient has Gilbert syndrome, which is a
genetic defect seen in about 7% of the general population. Patients have a
decreased amount of the enzyme glucuronyl transferase, which leads to slightly
elevated levels of unconjugated bilirubin. Patients tend to notice jaundice when
they are under emotional stress, during illness, or during fasting. Patients live a
normal life with this disease and only patient education is necessary.
A HIDA scan (choice B) is a nuclear medicine scan that is useful in the workup of cholecystitis. This patient has no pain, fever, white count, or risk factors
for cholecystitis. Therefore a HIDA scan isn't indicated
An abdominal ultrasound (choice C) is useful if you expect a structural
problem such as biliary tree obstruction as an etiology of his jaundice. With a
normal alkaline phosphatase and normal transaminases, intrahepatic or
extrahepatic obstruction is unlikely. Ultrasound is also useful for identifying
solid or cystic lesions.
This patient has no evidence of hepatitis and no risk factors for hepatitis. With
normal transaminases and no RUQ pain, acute hepatitis is unlikely. A hepatitis
profile (choice D) would likely be low-yield in this patient.
Liver biopsy (choice E) is often the gold standard for diagnosis of many liver
diseases. It is also useful in examining the progression of chronic liver disease.
It has no role in the work up of Gilbert disease.
A 16-year-old boy comes to the office because he recently noticed that his eyes
and skin are yellow. He denies any abdominal pain, nausea, vomiting, fevers,
chills, diarrhea, or sick contacts. He does admit to a large amount of stress
because his SAT examination is in 2 weeks and he is concerned about getting
into college. He denies IV drug use and has never been sexually active. He
reports that he "became yellow" 3-4 times before during periods when he had
viral illnesses. His temperature is 37.0 C (98.6 F), blood pressure is 120/70 mm
Hg, pulse is 73/min, and respirations are 13/min. Physical examination shows
scleral icterus. His abdomen is soft, non-tender, non-distended, with normal
bowel sounds. There is no hepatomegaly. Laboratory studies show:
The most appropriate next diagnostic step is to
A. do no further studies at this time
B. order a HIDA scan
C. perform an abdominal ultrasound
D. send hepatitis serology
E. schedule a liver biopsy
Explanation:
The correct answer is A. This patient has Gilbert syndrome, which is a
genetic defect seen in about 7% of the general population. Patients have a
decreased amount of the enzyme glucuronyl transferase, which leads to slightly
elevated levels of unconjugated bilirubin. Patients tend to notice jaundice when
they are under emotional stress, during illness, or during fasting. Patients live a
normal life with this disease and only patient education is necessary.
A HIDA scan (choice B) is a nuclear medicine scan that is useful in the work-
up of cholecystitis. This patient has no pain, fever, white count, or risk factors
for cholecystitis. Therefore a HIDA scan isn't indicated
An abdominal ultrasound (choice C) is useful if you expect a structural
problem such as biliary tree obstruction as an etiology of his jaundice. With a
normal alkaline phosphatase and normal transaminases, intrahepatic or
extrahepatic obstruction is unlikely. Ultrasound is also useful for identifying
solid or cystic lesions.
This patient has no evidence of hepatitis and no risk factors for hepatitis. With
normal transaminases and no RUQ pain, acute hepatitis is unlikely. A hepatitis
profile (choice D) would likely be low-yield in this patient.
Liver biopsy (choice E) is often the gold standard for diagnosis of many liver
diseases. It is also useful in examining the progression of chronic liver disease.
It has no role in the work up of Gilbert disease.
A 1-year-old boy is brought to the office by his mother because of a swelling
in his left grointhat was initially noticed while giving him a bath 3 months ago.
She feels that this swelling is completely asymptomatic and has grown
minimally in size. The child was born without any difficulties, but developed a
hydrocephalus, for which he underwent a ventricular peritoneal shunt. Since
then he has had no other significant difficulties, besides some mild upper
respiratory tract infections, which were well controlled. Physical examination
reveals a left-sided easily reducible inguinal hernia and no other abnormalities.
The most appropriate advice to this child's mother is that
A. a bilateral inguinal hernia repair is indicated
B. a left inguinal hernia repair is indicated
C. observation is all that is indicated
D. repair should be delayed until the child is 2 years of age
E. reversal of the ventricular peritoneal shunt is indicated
Explanation:
The correct answer is A. Inguinal hernias in children are usually indirect,
resulting from failure of the obliteration of the processus vaginalis. Treatment
requires high ligation and transection of the sac with or without excision of the
distal component. Repair need not be delayed, unless the infant has associated
medical problems. The complications from the hernia are most likely to occur
during the first 6 months of life hence, a repair should be performed soon after
the diagnosis. Contralateral exploration of the hernial sac should be performed
routinely in the subset of children who are most likely to have a clinically
occult hernia, children less than 2 years old, females less than 3 years old,
patients with ventricular peritoneal shunts, and patients less than 2 years old
with a left sided hernia.
Left sided repair of hernia (choice B) alone is not the correct option because in
a child under the age of 2 with a ventricular peritoneal shunt, contralateral
occurrence of the hernia is highly likely and should be repaired at the same
time.
Observation (choice C) may lead to complications of hernia like irreducibility,
incarcerations, strangulation and hence, the hernia should be repaired as soon
as possible.
There is no indication to wait until the age of 2 years (choice D) to treat a
hernia in a child.
Reversal of the ventricular peritoneal shunt (choice E) cannot be done as the
patient has a hydrocephalus and this should not be considered as an option for
a hernia in an infant.
A 37-year-old very healthy and functional woman is scheduled to undergo a
right total hip replacement after she suffered a femoral head fracture 2 days
ago during a fall while climbing a flight of stairs. She has no significant past
medical history and takes no routine medications. She is scheduled to have her
operation in the morning and you are called as a medicine consult to make any
appropriate recommendations. Postoperatively, this patient is at greatest risk
for developing
A. atrial fibrillation
B. confusion
C. a deep venous thrombosis
D. a myocardial infarction
E. a pulmonary embolism
Explanation:
The correct answer is C. Risk factors for deep venous thromboses (DVT) are
both heritable as well as environmental. Among the latter, certain situations
such as pregnancy, trauma, prolonged immobilization, orthopedic and certain
neurosurgical procedures are associated with the greatest short-term risk. For
this patient, with the risks of the fracture and the surgery being additive, some
estimates place her risk of DVT at greater than 80%.
Atrial fibrillation (choice A) can be present after certain surgical procedures,
usually thoracic or cardiac. This is not a significant risk for this patient.
Confusion (choice B) is always a concern when elderly patients are
hospitalized. However, while she may develop it, she is not considered elderly
at 37 years old, so the chances are not that great. If she does infact suffer from
perioperative confusion, it poses minimal actual risk to her, only discomfort.
The risk of myocardial infarction (choice D) is always a concern and numerous
preoperative risk assessment protocols have been developed. However,
generally speaking, a patient with no past medical history who has good
functional status almost certainly has minimal risk for severe coronary disease
and thus a myocardial infarction is not a great risk.
The risk of pulmonary embolism (choice E) is high and significant, but not
nearly as much as DVT. These estimates range from 2-27%.
A 16-year-old high school student is brought to the emergency department
because of severe right lower quadrant abdominal pain. She was at volleyball
practice when she suddenly doubled over in pain. She has no significant past
medical history and had a few episodes of right-sided abdominal pain in the
last week. She says that the earlier episodes of pain were much less severe. She
is sexually active with one partner, her boyfriend, and they use condoms for
birth control. Her temperature 38.8 C (101.8 F), pulse is 90/min, blood
pressure is 120/85 mm Hg, and respirations are 14/min. On physical
examination she appears very uncomfortable and is in obvious pain.
Abdominal examination is significant for focal exquisite tenderness and
guarding in the right lower quadrant. Leukocyte count is 11,200/mm3 with
69% segmented neutrophils and 2% band forms. The most appropriate test or
study at this time is
A. lower gastrointestinal barium enema
B. ultrasonography of the appendix
C. ultrasonography of the pelvis
D. upper gastrointestinal barium study with small bowel follow through
E. urinalysis
Explanation:
The correct answer is B. Severe pain localized to the right lower quadrant,
fever, and leukocytosis with bandemia are classic signs and symptoms of acute
appendicitis. Acute appendicitis is the most common cause of acute abdomen.
The initial study of choice to make this diagnosis is ultrasonography of the
appendix. Ultrasound signs of acute appendicitis are a dilated (greater than 6
mm), non-compressible appendix, and visualization of an appendicolith. An
appendicolith is calcified fecal material that gets trapped within the appendix
and is usually the cause of appendicitis. The sonographic finding of an
appendicolith is very specific for appendicitis. If ultrasonography does not
reveal such findings, the next appropriate step is a CT of the abdomen and
pelvis. A surgery consult should also be called.
Lower gastrointestinal barium enema (choice A) is not the best study in the
work up of acute appendicitis. Complete filling of the appendix to the bulbous
tip with barium is strong evidence against appendicitis. Nonfilling of the
appendix due to an obstructing appendicolith is what you would expect to see
in acute appendicitis. However, even in normal barium enema studies the
appendix does not always fill. Furthermore, this is a fairly invasive and time
consuming diagnostic study which is not appropriate in this case of an acute
abdomen.
Ultrasonography of the pelvis (choice C) is not indicated in this patient at this
time. Ultrasonography of the pelvis is useful for evaluating the uterus and
ovaries. Although she is sexually active, her symptoms do not suggest a
disease process involving these pelvic organs. The differential diagnosis for
women of child bearing age who present with right lower quadrant pain
includes acute appendicitis, ruptured ovarian cyst, and pelvic inflammatory
disease. The latter two disease processes, however, are not associated with
fever and leukocytosis.
Upper gastrointestinal barium study with small bowel follow through (choice
D) is not indicated in this patient. This examination is used to evaluate the
esophagus, stomach, and small bowel. Common indications for this diagnostic
study include dysphagia, symptoms of peptic ulcer disease, and
gastroesophageal reflux.
Urinalysis (choice E) will not provide useful information in the evaluation of
this patient. She has classic signs and symptoms of acute appendicitis and a
urinalysis will not be significant. Urinalysis is indicated when the diagnosis of
a urinary tract infection is suspected. Symptoms of cystitis include urinary
frequency and urgency. These symptoms associated with flank pain and fever
are suspicious for pyelonephritis.
A 53-year-old woman comes to the office because of the acute onset of severe
"anal pain." The pain is constant, but it is worse during defecation, walking,
and sitting. She says that it feels as if there is a "bulge" around her rectum and
that her anal sphincter is in a constant "spasm". She tells you that she is very
embarrassed and has never experienced anything like this before. She has spent
the last 7 months helping her best friend deal with rectal cancer, and now she
cannot believe she has a "pain down there" too. She tries to laugh as she says,
"this may be sympathy pain." She is a bus driver for the local elementary
school and she has 3 children of her own. She admits to a diet consisting
mainly of fast food, french fries and hamburgers. She rarely exercises, smokes
a pack of cigarettes a day, and drinks alcohol "socially". Her blood pressure is
130/80 mm Hg, pulse is 70/min, and respirations are 18/min. Physical
examination and anoscopy show a tender, blue swelling below the dentate line.
The most appropriate next step is to
A. admit her to the hospital for an evaluation for anal carcinoma
B. advise her to take sitz baths, apply a topical steroid cream, and increase
her fiber intake
C. incise the mass, evacuate the thrombus, and apply compression to the
incised area
D. perform an immediate rubber band ligation and tell her that it will
undergo degeneration
E. prescribe a bowel preparation and schedule a colonoscopy for next week
Explanation:
The correct answer is C. This patient has a thrombosed external hemorrhoid
that requires immediate incision and evacuation of the clot to provide
symptomatic relief. Pressure by compression is usually all that is needed to
control the bleeding. The typical presentation of a thrombosed external
hemorrhoid is an acute onset of very severe perianal pain, particularly when
walking and sitting. You know that it is an external hemorrhoid because it is
below the dentate line.
Since this patient has a thrombosed external hemorrhoid, it is unnecessary to
admit her to the hospital for an evaluation for anal carcinoma (choice A). Just
because her friend has it, it is unlikely that she too has anal carcinoma. She is
not complaining of rectal bleeding. She is complaining of acute, severe pain
and you can see the thrombosed hemorrhoid, therefore this case is consistent
with a thrombosed external hemorrhoid. In a patient with chronic hypochromic
anemia and rectal bleeding, since hemorrhoids are so common, a thorough
investigation should be done to rule out an additional source. However, since
this patient has an obvious thrombosed hemorrhoid, this should be treated right
now.
Sitz baths, applying a topical steroid cream, and increasing her fiber intake
(choice B) are the usual treatment for symptomatic external hemorrhoids that
are not thrombosed. Increased fiber intake and possibly stool softeners are part
of the long-term management of hemorrhoids. This patient requires an incision
and evacuation of the thrombus at this time.
Rubber band ligation (choice D) is the treatment for refractory symptomatic
internal hemorrhoids. Local anesthesia is given, an anoscope is inserted, and a
rubber band is placed around the hemorrhoid. The hemorrhoid will eventually
become necrotic and slough over time. Internal hemorrhoids arise above the
dentate line.
Bowel preparation and colonoscopy (choice E) are not the next step in the
management of a thrombosed external hemorrhoid. This patient needs an
incision and the evacuation of the clot to provide symptomatic relief. If this
patient has never had a colonoscopy, you may consider this after this acute
episode of hemorrhoids is effectively treated.
A 72-year-old man presents to your outpatient clinic center with diffuse
pruritus and “yellow skin” for the last several weeks. He denies any rash. He
has not started any new medications and denies any unusual exposures. His
past medical history is significant for benign prostatic hypertrophy,
emphysema, and osteoarthritis. His temperature is 37.2 C (99 F), blood
pressure is 130/90 mm Hg, pulse is 78/min, and respirations are 14/min. His
lungs are clear and heart examination is remarkable for a 2/6 systolic ejection
murmur. There is suggestion of a right upper quadrant palpable mass on
abdominal examination and bowel sounds are normal. Laboratory studies show
a leukocyte count 7,300mm3, platelets 335,000mm3, hematocrit 48%, BUN 20
mEq/L, creatinine 1.1 mEq/L, alanine aminotransferase (ALT) 60 U/L,
aspartate aminotransferase (AST) 80 U/L, alkaline phosphatase 430 U/L,
bilirubin (total) 15.2 mg/dL, bilirubin (direct) 14.6 mg/dL, and amylase 100
U/L. Management of this patient will likely entail
A. endoscopic retrograde cholangiopancreatogram (ERCP) and
sphincterotomy for an obstructing gallstone
B. intravenous fluids and meperidine for pancreatitis
C. laparoscopic cholecystectomy for cholecystitis
D. systemic intravenous antibiotics for ascending cholangitis
E. Whipple procedure for pancreatic adenocarcinoma
Explanation:
The correct answer is E. The patient is presenting with painless jaundice, a
syndrome that is commonly associated with diseases such as pancreatic
carcinomas or duodenal ampullary carcinomas. These tend to obstruct the
distal common bile duct over a more subacute to chronic timeframe, rather
than an acute obstruction from a passed gallstone (choice A) that gets lodged
in the common bile duct (CBD). In the latter, patients may present with similar
lab values consistent with CBD obstruction, such as elevated total/direct
bilirubin and elevated alkaline phosphatase. However, they will typically have
right upper quadrant pain and fever if there is cholangitis present (choice D).
The palpable mass in this case is actually a dilated gallbladder and not the
pancreatic mass itself, (which actually may be very small and still cause these
types of symptoms). This classic, but rare finding, is termed Courvoisier's sign.
Although the amylase is in the upper range of normal, the clinical scenario
provided is not consistent with pancreatitis (choice B) in which the patient will
typically have epigastric pain radiating to the back, nausea, vomiting, and
anorexia. More often than not, neither amylase nor lipase will be elevated in
pancreatic malignancy.
The clinical syndrome here is not consistent with acute cholecystitis (choice
C), which would present with right upper quadrant pain, fever, and elevated
leukocyte count. In simple cholecystitis there will not be lab values suggestive
of CBD obstruction.
A 55-year-old banker comes to the office for a routine initial visit. He says that
he has no significant past medical or surgical history. He takes no medications
except for a daily multivitamin. His family history reveals that his mother died
of breast cancer and his father committed suicide when he was 5 years old. On
further questioning, he reports that he has been working 10 to 12 hours a day
and is feeling very stressed at work. He has been drinking 2 to 3 beers a night
to help relieve the stress and help him get some sleep. He vehemently denies
ever experimenting with intravenous drugs, but currently uses marijuana about
once a month. He has had 3 different sexual partners over the past month and
does not routinely use condoms. Physical examination reveals a nodular liver
edge of 9-cm in diameter and a tender abdomen in the right upper quadrant but
no rebound or guarding. His Murphy's sign is negative. His liver function panel
shows:
The factor in this patient's history most closely correlated with his condition is
A. alcohol intake
B. drug use
C. family history of cancer
D. His history correlates with biliary tract disease.
E. unsafe sexual practices
Explanation:
The correct answer is E. This patient has multiple different reasons to have
liver damage and cirrhosis. The liver function panel indicates that this patient
probably has cirrhosis, as indicated by synthetic liver disease (coagulopathy
and decreased albumin). His transaminases are more consistent with a viral
etiology than alcoholic hepatitis (ALT>AST), probably viral hepatitis from a
sexual encounter.
Alcoholic hepatitis (choice A) commonly causes liver disease characterized
with an elevated AST > AST. Additionally, cirrhosis secondary to alcoholic
hepatitis routinely causes an enlarged liver edge rather than a normal or small-
sized liver.
The most common cause of viral hepatitis/cirrhosis in the United States is
hepatitis C as a result of intravenous drug use (choice B). However, this patient
denies ever using intravenous drugs and given his candor during the interview,
there is no reason to suspect that he is lying. Marijuana has not been linked to
hepatitis.
The patient's family history of cancer (choice C) doesn't correlate to an
increased risk of liver disease.
This patient has a negative Murphy's sign and a normal alkaline phosphatase. It
is incorrect to say that nothing in his history suggests biliary disease (choice
D), because his unsafe sexual practices increase his risk of hepatitis.
A 52-year-old woman with hypertension comes to the office for a follow-up
visit to discuss her recent diagnosis of colonic carcinoma. Her mother has
breast and ovarian carcinomas, her grandmother had breast carcinoma, and her
mother's sister had stomach and ovarian carcinomas. Because of the significant
family history, she underwent screening for colonic, ovarian, and breast
carcinomas starting at an early age. In the last month, she noticed some change
in her bowel habits and black colored stools. She was noted to have guaiacpositive stools and was referred for a colonoscopy. The colonoscopy
examination confirmed a lesion in the right side of the colon, a biopsy of which
identified it as an adenocarcinoma. No other lesions were noted in the rest of
the colon on colonoscopic examination. She underwent an appendectomy at
the age of 12 and a hysterectomy at the age of 31. Both surgical procedures
were uneventful. The patient has 3 healthy children. At this time the most
appropriate management is to advise the patient to consider
A. biannual CT scans of the abdomen and pelvis
B. gene therapy
C. a right hemicolectomy
D. a segmental colectomy
E. a subtotal colectomy and bilateral salpingooophorectomy
Explanation:
The correct answer is E. This patient's family history is strongly suggestive
of hereditary non-polyposis colorectal carcinoma (HNPCC or Lynch
syndrome). Lynch syndrome is due to mutations in mismatch repair genes, and
are inherited as autosomal dominant. This predisposition runs in families to
develop into endometrial, ovarian, breast, and gastric carcinomas. Affected
individuals show predominance for right-sided cancers or multiple cancers and
tend to be young when these cancers develop. They often develop
metachronous colorectal cancer. Because of this predisposition, family
members at risk should undergo biannual colonoscopy beginning at the age of
25, women should have an annual pelvic examinations with endometrial
biopsies every 3 years and mammograms at an early age. In a woman who has
completed child bearing, a total colectomy, and a hysterectomy, with a
bilateral salpingooophorectomy should be considered.
A CT scan of the abdomen and pelvis (choice A) is indicated as a metastatic
workup, but not as a therapy for right colonic carcinoma. Biannual CT scans
are not a useful recommendation, as this patient is already diagnosed with
colon cancer for which she requires therapy.
Gene therapy (choice B), when available, may prevent multiple carcinomas,
but won't treat an already established cancer.
Right hemicolectomy (choice C) in a patient with known Lynch syndrome
may not prevent further cancers. Physicians should thoroughly discuss the
possibilities of the development of multiple cancers and metachronous cancers
in the future with these patients.
Segmental colonic resection (choice D) is not an option for the treatment of
colonic carcinoma.
A 52-year-old woman with hypertension comes to the office for a follow-up
visit to discuss her recent diagnosis of colonic carcinoma. Her mother has
breast and ovarian carcinomas, her grandmother had breast carcinoma, and her
mother's sister had stomach and ovarian carcinomas. Because of the significant
family history, she underwent screening for colonic, ovarian, and breast
carcinomas starting at an early age. In the last month, she noticed some change
in her bowel habits and black colored stools. She was noted to have guaiacpositive stools and was referred for a colonoscopy. The colonoscopy
examination confirmed a lesion in the right side of the colon, a biopsy of which
identified it as an adenocarcinoma. No other lesions were noted in the rest of
the colon on colonoscopic examination. She underwent an appendectomy at
the age of 12 and a hysterectomy at the age of 31. Both surgical procedures
were uneventful. The patient has 3 healthy children. At this time the most
appropriate management is to advise the patient to consider
A. biannual CT scans of the abdomen and pelvis
B. gene therapy
C. a right hemicolectomy
D. a segmental colectomy
E. a subtotal colectomy and bilateral salpingooophorectomy
Explanation:
The correct answer is E. This patient's family history is strongly suggestive
of hereditary non-polyposis colorectal carcinoma (HNPCC or Lynch
syndrome). Lynch syndrome is due to mutations in mismatch repair genes, and
are inherited as autosomal dominant. This predisposition runs in families to
develop into endometrial, ovarian, breast, and gastric carcinomas. Affected
individuals show predominance for right-sided cancers or multiple cancers and
tend to be young when these cancers develop. They often develop
metachronous colorectal cancer. Because of this predisposition, family
members at risk should undergo biannual colonoscopy beginning at the age of
25, women should have an annual pelvic examinations with endometrial
biopsies every 3 years and mammograms at an early age. In a woman who has
completed child bearing, a total colectomy, and a hysterectomy, with a
bilateral salpingooophorectomy should be considered.
A CT scan of the abdomen and pelvis (choice A) is indicated as a metastatic
workup, but not as a therapy for right colonic carcinoma. Biannual CT scans
are not a useful recommendation, as this patient is already diagnosed with
colon cancer for which she requires therapy.
Gene therapy (choice B), when available, may prevent multiple carcinomas,
but won't treat an already established cancer.
Right hemicolectomy (choice C) in a patient with known Lynch syndrome
may not prevent further cancers. Physicians should thoroughly discuss the
possibilities of the development of multiple cancers and metachronous cancers
in the future with these patients.
Segmental colonic resection (choice D) is not an option for the treatment of
colonic carcinoma.
A 44-year-old man comes to the emergency department complaining of severe
abdominal pain and coffee ground vomitus. The patient is a busy financial
executive who reports that over the past few months he has had increasing
abdominal pain associated with eating. The patient reports some mild reflux of
acid in between meals but has no prior episodes of emesis. This morning, on
his way to the office he developed the acute onset of mid-epigastric pain
associated with nausea. Ten minutes later, he vomited coffee ground-like
material. An upper endoscopy is performed and the patient is found to have a
large ulcer in the first portion of the duodenum. There is no visible vessel or
active bleeding seen. He is admitted to the hospital. You go to examine him
and he is awake and alert and wants to know about his disease. At this time the
most correct statement about this patient's condition is:
A. The lesion is benign and is not the cause of the bleed
B. The lesion is premalignant and likely caused the bleeding
C. The lesion must be treated or the bleeding will likely recur
D. The lesion must be treated but the bleeding will still recur
E. There is no effective treatment for this lesion
Explanation:
The correct answer is C. The cause of more than 90% of duodenal ulcers is
infection with Helicobacter pylori. The treatment for these ulcers, when nonbleeding, is antibiotic therapy. When they have bled, in addition to all of the
appropriate therapy for a bleeding patient, they still need to be tested and
treated for infection. Without the proper treatment for the infection, their ulcers
will worsen or they will develop new ones and they will be at continued risk
for bleeding.
The lesion is benign and is not the cause of your bleed (choice A) is incorrect.
The lesion identified in the vignette is the classical lesion for such episodes of
bleeding.
The lesion is premalignant and likely caused your bleeding (choice B) is
incorrect. Premalignant lesions that are associated with upper GI bleeding are
almost solely confined to the stomach.
The lesion must be treated but bleeding will still recur (choice D) is incorrect
because therapy for duodenal ulcers is curative and therefore will attenuate, if
not abrogate any future risk of rebleeding.
There is no effective treatment for this lesion (choice E) is incorrect. Very
effective treatment for this infection exists and it includes metronidazole,
tetracycline, and bismuth orally for two weeks. In many centers, an oral
macrolide antibiotic such as clarithromycin and a proton pump inhibitor such
as omeprazole are becoming standard therapy.
A 53-year-old man comes to the emergency department because of
intermittent, crampy abdominal pain that has progressively worsened over the
past 6 days. He had called you 4 days ago complaining of nausea and diarrhea,
and you diagnosed him with gastroenteritis over the phone and told him to
drink plenty of fluids. Now he says that he has not passed stool in 2 days,
which is more "his style" because he is usually very constipated. He has no
prior history of surgeries, and has no chronic medical conditions. He denies
any weight loss or fatigue. His temperature is 37.8 C (100.0 F), blood pressure
is 130/85 mm Hg, pulse is 110/min, and respirations are 25/min. Physical
examination shows an erythematous scrotum with a firm, tender mass on the
right side. Gentle manual attempts at reduction of the mass are unsuccessful.
The most appropriate next step is to
A. do a trans-scrotal biopsy
B. obtain a CT scan of the abdomen
C. order an excretory urography
D. prepare him for an immediate operation
E. recommend bed rest, scrotal elevation and support, and ice packs
Explanation:
The correct answer is D. This patient most likely has an incarcerated inguinal
hernia that cannot be reduced and therefore requires immediate surgery. The
crampy abdominal pain, tachycardia, and fever suggests a small bowel
obstruction (SBO) with strangulation. A hernia is one of the most common
causes of SBO. A firm, tender, red mass in the groin is an incarcerated hernia
until proven otherwise. His history of constipation increases his risk of
developing a hernia.
A trans-scrotal biopsy (choice A) is not indicated in a suspected incarcerated
hernia. You may think that it is used to diagnosis testicular cancer in a patient
with a painless testicular mass, however, it is not recommended in that case
either because of the potential risk of "contamination" of the scrotal lymphatics
with tumor cells.
A CT scan of the abdomen (choice B) is useful in acute diverticulitis, which
often presents with left-lower quadrant abdominal pain. A scrotal mass will not
be present.
An excretory urography (choice C) is used in suspected cases of renal calculi,
which present with nausea, vomiting, hematuria, flank pain, and possibly
inguinal and scrotal pain. A scrotal mass would not be present.
Bed rest, scrotal elevation and support, and ice packs (choice E) are part of the
treatment for epididymitis, which is an infectious process. Antibiotics must be
added too. This usually presents with a tender scrotum and a swollen
epididymis. Elevation of the scrotum often relieves the pain. Signs of a bowel
obstruction (crampy abdominal pain, nausea, etc.), are not present in
epididymitis.
A 34-year-old intravenous drug abuser who is HIV positive is admitted to the
hospital because of gastrointestinal bleeding. He was admitted to the hospital 2
months ago for HIV treatment. At the time of discharge, he was in good health,
able to tolerate regular diet, and take minimal medications. He went back to
work and was feeling well. Two days before presenting to the hospital, he
developed nonspecific abdominal discomfort, which he attributed to food
poisoning and treated himself with lots of hydration. The abdominal
discomfort persisted and he noticed bleeding per rectum, the night before
coming to the hospital. The next morning, he noticed more blood per rectum,
and alarmed by that, decided to come to the hospital. His temperature is 37. C
(99.1 F), blood pressure is 110/70 mm Hg, and pulse is 96/min. His hematocrit
is 28% compared with 34% on discharge a couple of months earlier. There are
no signs of hemodynamic instability. Blood is sent for cross match and stool is
sent for ova and parasites. A nasogastric tube is inserted and returns clear fluid.
The next step in the investigation of this patient's gastrointestinal bleeding is
a(n)
A. barium enema
B. colonoscopy
C. CT scan of the abdomen and pelvis
D. small bowel series
E. upper gastrointestinal endoscopy
Explanation:
The correct answer is B. A colonoscopy is the initial investigation of choice
in gastrointestinal (GI) bleeding in HIV-positive patients. Gastrointestinal
bleeding is an unusual occurrence in HIV infected individuals, but when it
does occur, it is usually related to a complication of an HIV infection. Lower
GI bleeding is twice as common as upper GI bleeding. Upper GI bleeding,
when it occurs, is related to Kaposi's sarcoma or lymphoma 50% of the time.
CMV ulcers do occur in the upper GI tract, but more frequently in lower GI
tract. Lower GI tract bleeding is usually caused by localized colitis of
infectious origin from Cytomegalovirus, herpes simplex, or bacteria. In a stable
patient, colonoscopy is the procedure of choice for localizing the bleeding and
obtaining biopsies to look for specific infections and antibiotic sensitivities.
A barium enema (choice A) is not as useful as the first investigation, unless a
colonic carcinoma is suspected.
A CT san of the abdomen and pelvis (choice C) may show thickening of the
colon with the infiltration of fat in the surrounding mesentery, but may not be
diagnostic.
A small bowel series (choice D) is rarely indicated as the initial investigation
of choice in investigating gastrointestinal bleeding. If upper gastrointestinal
endoscopy and colonoscopy do not reveal any lesions and the patient continues
to bleed, then one should look for small bowel sources by means of small
bowel series or enteroscopy.
Since upper gastrointestinal tract bleeding is less common than lower
gastrointestinal tract bleeding, a upper gastrointestinal endoscopy (choice E) is
not indicated in this patient. Also, a nasogastric tube return does not show
blood in this patient. Although it is difficult to rule out upper gastrointestinal
bleeding, there is no bile in the nasogastric tube, so a colonoscopy should still
be the first investigation of choice.
A 17-year-old woman comes to the office complaining of a 3-month history of
"crampy" abdominal pain along with alternating episodes of constipation and
diarrhea. She reports that the pain has been worse recently since starting a new
job, which she describes as "high stress." Pain also seems worse with eating
fatty meals. Her temperature is 37.0 C (98.6F), blood pressure is 120/72 mm
Hg, pulse is 63/min, and respirations are 10/min. Physical examination reveals
a soft, non-tender, non-distended abdomen with normal bowel sounds and
without organomegally. Her rectal exam reveals normal tone. The correct
diagnosis would be supported by finding
A. a biopsy with transmural intestinal inflammation
B. fistula formation within the abdomen
C. a history of bloody diarrhea
D. a normal colonoscopy
E. positive H. pylori antibody titers
Explanation:
The correct answer is D. This patient has irritable bowel syndrome (IBS).
This is the most common functional GI disorder. It is characterized by
abdominal pain with alternating diarrhea and constipation. It is often related to
stress or exercise. Diagnosis is made by history and thoughtful exclusion of
other organic diseases. Colonoscopy, if performed, should be normal.
Indication for colonoscopy would be to differentiate irritable bowel syndrome
from inflammatory bowel disease.
Transmural inflammation (choice A) and fistula formation (choice B) is
typical of Crohn disease. Crohn disease is characterized by inflammation of
any part of the gastrointestinal tract (mouth to anus). Patients often have
lesions, which are not continuous, described as skip lesions. The inflammation
is transmural, which means that it involves all layers of the mucosal wall
contrary to UC. Patients typically present with non-bloody diarrhea, weight
loss, and abdominal pain.
Bloody diarrhea (choice C) is a finding in ulcerative colitis (UC). UC is
characterized by inflammation limited to the colon and rectum and is typically
described as continuous (as opposed to "skip" lesions). Bloody diarrhea is the
typical presenting symptom.
H. pylori (choice E) may be a cause of abdominal pain and its eradication is
recommended if it is discovered incidentally, but routine serology checks in
the work-up of abdominal pain is of unproved benefit.
A 37-year-old man comes to the emergency department because of the abrupt
onset of crampy abdominal pain and "bright red blood oozing" from his mouth.
There were no episodes of emesis preceding the hematemesis. The patient has
a past medial history significant for alcoholic cirrhosis documented by liver
biopsy 3 years ago. He has been poorly compliant with medications and has
not been seen by a physician for over 2 years. He continues to drink 6-12 beers
per day. His blood pressure is 90/40 mm Hg and pulse is 90/min. Physical
examination shows scleral icterus, clear lung fields, a distended and tense
abdomen with a fluid wave, and diffuse spider angiomata on his chest and
abdomen. There is no asterixis. You send him for upper endoscopy, which
reveals grade three esophageal varices with no active bleeding. These varices
are sclerosed. He is admitted to the hospital. The most appropriate next step in
management to prevent morbidity is to
A. admit the patient to the ICU for a re-endoscopy in 48 hours
B. begin intravenous octreotide therapy
C. observe the patient for 48 hours and then discharge to home
D. perform an immediate portal-systemic shunt operation
E. transfuse the patient to a hematocrit greater that 30%
Explanation:
The correct answer is B. About 30-60% of variceal bleeding episodes stop
spontaneously. Therefore, in the setting of a presumed upper gastrointestinal
(UGI) bleed when an endoscopy shows varices but no active lesions, banding
and intravenous somatostatin or its analogue, octreotide, are indicated. These
agents are vasodilators that cause a reflex increase in splanchnic vessel tone
and thus serve to decrease bleeding. This is the standard of care for GI bleeds
from varices.
Admitting the patient to the ICU for a re-endoscopy in 48 hours (choice A) is
not indicated as the patient appears hemodynamically stable and does not
require ICU monitoring.
Observe the patient for 48 hours and then discharge to home (choice C) is not
acceptable since most centers choose to re-endoscope patients prior to
discharge.
Performing an elective portal-systemic shunt operation (choice D) is certainly
an option in the secondary prevention of UGI bleeds due to varices. It is a
highly effective option. The drawback is that the procedures are associated
with a significant incidence of hepatic encephalopathy. As such, there is no
indication to refer all patients with variceal bleeds for elective shunt therapy,
but the option should be offered to them.
Transfuse the patient to a hematocrit greater that 30% (choice E) is a "trigger"
often taught but the decision to transfuse a patient should be based upon the
patient's clinical condition and not a number. If the patient is stable and the
bleeding has been controlled, there is no absolute reason why a hematocrit
greater than 30 must be attained. There is in fact recent literature that suggests
some critically ill patients do worse with a more aggressive transfusion
strategy (Hct >30) compared to a more permissive goal (Hct >24).
A 21-year-old woman is admitted to the hospital because of a 3-day history of
diarrhea and abdominal pain. Her bowel movements were loose, mucous-like
and occasionally bloody. The patient reports no recent ingestions or any
change in her diet. On further questioning, she reports that over the past few
years she has occasionally had episodes such as these but that these have
passed with a home remedy. She came to the hospital this time because her
boyfriend was worried that she might be ill. On physical examination, the
patient is thin and in no distress. Her abdominal examination shows diffuse
tenderness to palpation but no rebound tenderness or guarding. The laboratory
or diagnostic findings most supportive of the diagnosis of inflammatory bowel
disease is
A. elevated fecal fat content
B. fecal Gram stain positive for organisms
C. fecal gross blood
D. fecal leukocytes
E. positive fecal reducing substance test
Explanation:
The correct answer is D. Although the bowel is populated with a variety of
organisms, white cells generally indicate active inflammation. The presence of
leukocytes in a stool sample would be highly suggestive of IBD.
An elevated fecal fat content (choice A) is a result that is expected in
conditions that result in malabsorption of fat. Examples include pancreatic
insufficiency, certain forms of sprue, and gallbladder disease. It is not a
specific finding for IBD.
A fecal Gram stain positive for organisms (choice B) would be a test that is
largely without any benefit. The colon, as mentioned above, is normally
populated with a tremendous number and variety of bacteria. A positive Gram
stain is therefore expected even under normal conditions.
Fecal gross blood (choice C) is not specific for IBD, and in fact, is more
generally indicative of hemorrhoids or a low sigmoid or rectal lesion.
A positive fecal reducing substance test (choice E) would indicate the presence
of reducing substances in the stool, such as sugars. This is a test commonly
used in pediatric populations to test for certain digestive deficiencies that may
be causing diarrhea.
A 51-year-old man is post-procedure day number 2 from an upper endoscopy
and banding for bleeding esophageal varices. The patient has a 7-year history
of chronic active hepatitis and over the past few years has developed stigmata
associated with cirrhosis and worsening portal hypertension. Three days ago,
he presented to the emergency department with bright red blood per mouth and
rectum and a nasogastric tube evacuated bright red blood and coffee grounds
from the patient's stomach. He was admitted to the hospital, transfused with 2
units of red blood cells and underwent an endoscopy. On preparation for the
patient's discharge, you have a long discussion with your patient about the
course of events. In counseling this patient on his future risks and course of
therapy, you should advise him that:
A. The risk of rebleeding is between 50% and 80% and medical therapy is
indicated
B. The risk of rebleeding is between 50% and 80% and surgical therapy is
indicated
C. The risk of rebleeding is between 50% and 80%, but no therapy is
indicated given the nature of the problem
D. There is no concern for rebleeding
E. There is no concern for rebleeding, he is an imminent candidate for
transplant
Explanation:
The correct answer is A. There is overwhelming data to support the treatment
strategy of some sort of medical therapy (beta blockade, sclerotherapy,
banding) for all patients with an episode of variceal bleeding. The risk of
rebleeding is high and approaches 80% in most series at 1-year. There is
excellent data showing the efficacy of portal-systemic surgical shunts (side-toside caval, end-to-side caval, distal splenorenal, TIPS) in secondary
prevention. Medical therapy is indicated in all patients and all patients should
be offered the option of a surgical shunt. The largest drawback to these
procedures is their resultant incidence of encephalopathy.
The risk or rebleeding is between 50% and 80% and surgical therapy is
indicated (choice B) is incorrect because there are nonsurgical options
available. These options include beta blockade, sclerotherapy, and banding.
It is incorrect to say that the risk of rebleeding is between 50% and 80% but no
therapy is indicated given the nature of the problem (choice C). As stated
above, therapy options include beta blockade, sclerotherapy, banding, and
hopefully a liver transplant.
There is a 50% and 80% chance of rebleeding, therefore, there is no concern
for rebleeding (choice D) is incorrect.
Liver transplant (choice E) is of course curative and its 5-year survival rate of
approximately 70% is superior to cirrhotics with Child class C disease who are
treated by other modalities. This answer is incorrect because it starts with
"there is no concern for rebleeding," which is incorrect.
A 69-year-old man is brought to the clinic from his convalescent home,
because of decreased mental status. He has a history of Alzheimer's disease,
depression, hypertension, coronary artery disease, and glaucoma. There is a
“do not resuscitate” (DNR) order on the chart signed by the patient's wife. His
temperature is 37.0 C (98. 6 F), blood pressure is 110/70 mm Hg, and
respirations are 16/min. Physical examination shows a distended abdomen
without focal tenderness or peritoneal signs and hard stool in the rectal vault.
The patient is alert and oriented only to person. An electrocardiogram reveals
normal sinus rhythm with a few premature ventricular contractions (PVC).
Laboratory studies are normal. A plain x-ray of the chest reveals multiple
pulmonary nodules. A plain x-ray of the abdomen demonstrates a distended
ascending and transverse colon measuring 20 cm with copious stool present.
The next step in the evaluation of this patient is to
A. consult with the family to consider comfort care only
B. discharge him to the convalescent home with comfort care only
C. order a barium enema after a preparatory enema
D. order a CT scan of the thorax
E. prepare him for a laparotomy
Explanation:
The correct answer is C. This patient has symptoms of fecal impaction which
is common in the elderly and in debilitated patients. A preparatory enema to
clean out impacted stool is necessary prior to a barium enema and is likely to
be therapeutic in this patient. Subsequent barium enema could assess the cause
of large bowel dilatation which is unusual in the setting of simple fecal
impaction. In this case, colon cancer is a definite possibility.
A “do not resuscitate” order is not a mandate to forego appropriate minimally
invasive diagnostic tests such as an enema. Hence, comfort care only (choice
A) would not be appropriate as a diagnosis of a potentially easily treatable
condition such as stool impaction.
A “do not resuscitate” order is not a mandate to forego appropriate minimally
invasive diagnostic tests such as an enema. Discharge (choice B) is not yet
warranted.
A CT of the thorax (choice D) is not necessary now as there are clearly
multiple nodules on the chest radiograph.
A laparotomy (choice E) is not indicated at this point in the diagnostic workup.
Moreover, surgery would only be performed in a patient with a do not
resuscitate order after consult with the family or durable power of attorney.
A 22-year-old woman comes to the emergency department with 18 hours of
abdominal pain that began late last night and has progressively worsened. She has
been unable to eat due to loss of appetite and has had severe nausea with vomiting
for the past 6 hours. On physical examination she has rebound tenderness over her
right lower quadrant with abdominal guarding. Her temperature is 37.8 C (100 F),
blood pressure is 130/70 mm Hg, and pulse is 100/min. A 12-lead
electrocardiogram shows sinus tachycardia. The diagnosis of appendicitis is
confirmed by diagnostic imaging. Her laboratory data are as follows:
The most appropriate next step in the management of this patient is to
A. initiate broad spectrum antibiotic coverage and observe in the emergency
department
B. perform a lumbar puncture for possible meningeal spread
C. prepare the patient for urgent appendectomy
D. send blood and urine cultures for possible bacteremia
E. wait until the patient defervesces and then plan for elective appendectomy
Explanation:
The correct answer is C. The physical examination and the laboratory data indicate
that this patient has a moderate to severe systemic inflammatory response. The
treatment is definitive removal of the source of infection before perforation or septic
physiology occurs.
Initiating broad spectrum antibiotic coverage (choice A) is certainly appropriate, but
fails to DEFINITIVELY address the underlying infection. The antibiotic regimen
will improve this patient's chances of event-free survival, but without surgical
removal of the appendix, the patient faces extreme morbidity and high mortality
risk.
Performing a lumbar puncture for possible meningeal spread (choice B) is not
necessary in the absence of any clinical signs of meningeal infection.
Sending blood and urine cultures for possible bacteremia (choice D) is a diagnostic
intervention. In addition, the results of the test will in no way influence therapy
since as long as the bacteremic focus remains, it is likely that the patient has highgrade bacteremia. The patient will however still go to the operating room
irrespective of the results of the test.
Waiting until the patient defervesces and then planning for elective appendectomy
(choice E) is almost akin to a natural history experiment in that the patient is
essentially receiving no treatment for a very inflamed appendix. This is completely
inappropriate.
A 41-year-old man with asthma comes to the clinic for an annual employee
physical examination. The patient uses an albuterol metered dose inhaler. Vital
signs and physical examination are normal. The patient, however, is mildly
drowsy and confused. The patient brought with him a CT scan obtained at
another hospital. He is not sure why it was obtained. A report with the CT
indicates a nodular liver without malignant appearing masses. The most
important next step is to
A. obtain old chest x-rays
B. obtain a review of systems
C. obtain a social history
D. order liver function tests
E. perform colonoscopy
Explanation:
The correct answer is C. The patient is confused, so alcohol intoxication must
be suspected especially in light of a CT that is suspicious for cirrhosis. Alcohol
and hepatitis are by far the most common causes of cirrhosis in the United
States.
Obtaining old chest x-rays (choice A) is not essential in the management of
this patient, since he has no pulmonary symptoms.
A thorough review of symptoms (choice B) is important in all patients, but it is
not of primary concern in this setting. A social history is valuable especially
since the CT demonstrates a cirrhotic liver.
Ordering liver function tests (choice D) is necessary, but of secondary
importance. This patient is very likely to have a mild to moderate elevation of
liver function tests in light of his cirrhosis. Determining the alcohol history is
more critical.
A colonoscopy (choice E) would be a consideration if there were lesions seen
in the liver that were suspicious for metastatic disease. Colon cancer frequently
metastasizes to the liver and a colonoscopy would be part of a routine
metastatic work-up. It is not necessary in this situation because there are no
metastatic lesions of the liver seen on the CT. Routine screening such as
flexible sigmoidoscopy is recommended for health maintenance in this case.
A 22-year-old college student comes to the clinic because of abdominal pain
and diarrhea. He has had diffuse crampy abdominal pain intermittently for the
last 4 months. He also has occasional episodes of watery diarrhea and has seen
bright red blood in his stool several times. He denies fever, nausea, vomiting,
or recent travel. He has no significant medical problems and does not take any
medications. Vital signs are: 38.1 C (100.6 F), blood pressure 120/80 mm Hg,
and pulse 70/min. Abdominal examination is significant for diffuse guarding
and tenderness. He went to the emergency department 1 week ago with similar
symptoms and had a CT of the abdomen and pelvis performed. The CT
demonstrated a thickening of the terminal ileum and the remaining bowel
appeared unremarkable. The most appropriate next step in management is to
A. advise him to eat a gluten free diet
B. order a colonoscopy
C. order an upper gastrointestinal (UGI) barium study
D. prescribe broad-spectrum antibiotics
E. prescribe steroids
Explanation:
The correct answer is C. This patient's symptoms of abdominal pain and
watery bloody diarrhea are suspicious for inflammatory bowel disease. The CT
finding of terminal ileum thickening is suggestive of Crohn disease. The
diagnosis of Crohn's disease is often made by an upper gastrointestinal (UGI)
barium study. Classic findings include deep ulcers and discontinuous or "skip"
inflammatory lesions in the small bowel. Complications include stricture and
fistula formation. Colonoscopy is also useful if colon involvement is
suspected.
A gluten free diet (choice A) is not necessary for this patient. Celiac or
nontropical sprue is a malabsorption syndrome that is treated with a gluten free
diet. This is a rare syndrome of gluten allergy that presents with diarrhea and
weight loss. This is not a likely diagnosis in this patient, because celiac sprue
usually presents in childhood as the patient starts to eat gluten in wheat-based
foods.
A colonoscopy (choice B) may be useful in the work-up of this patient, but it is
not the most appropriate next step. The CT demonstrated involvement of the
terminal ileum, which is highly suggestive of Crohn's disease. An upper
gastrointestinal (UGI) barium study will evaluate the small bowel for
inflammatory lesions. A colonoscopy can also be performed later to look for
involvement of the colon.
Prescribing broad-spectrum antibiotics (choice D) is premature at this time.
The patient's symptoms and CT findings are highly suggestive of an
inflammatory bowel disease and in particular, Crohn's disease. The supportive
treatment of Crohn's disease includes antibiotics, bowel rest, and parenteral
nutrition. Prior to starting antibiotics, it is important to confirm the diagnosis
first with an upper gastrointestinal (UGI) barium study.
Prescribing steroids (choice E) is premature at this time. Although steroid
therapy is a part of the treatment of Crohn's disease, it is important to confirm
the diagnosis first with an upper gastrointestinal (UGI) barium study.
A 22-year-old college student comes to the clinic because of abdominal pain
and diarrhea. He has had diffuse crampy abdominal pain intermittently for the
last 4 months. He also has occasional episodes of watery diarrhea and has seen
bright red blood in his stool several times. He denies fever, nausea, vomiting,
or recent travel. He has no significant medical problems and does not take any
medications. Vital signs are: 38.1 C (100.6 F), blood pressure 120/80 mm Hg,
and pulse 70/min. Abdominal examination is significant for diffuse guarding
and tenderness. He went to the emergency department 1 week ago with similar
symptoms and had a CT of the abdomen and pelvis performed. The CT
demonstrated a thickening of the terminal ileum and the remaining bowel
appeared unremarkable. The most appropriate next step in management is to
A. advise him to eat a gluten free diet
B. order a colonoscopy
C. order an upper gastrointestinal (UGI) barium study
D. prescribe broad-spectrum antibiotics
E. prescribe steroids
Explanation:
The correct answer is C. This patient's symptoms of abdominal pain and
watery bloody diarrhea are suspicious for inflammatory bowel disease. The CT
finding of terminal ileum thickening is suggestive of Crohn disease. The
diagnosis of Crohn's disease is often made by an upper gastrointestinal (UGI)
barium study. Classic findings include deep ulcers and discontinuous or "skip"
inflammatory lesions in the small bowel. Complications include stricture and
fistula formation. Colonoscopy is also useful if colon involvement is
suspected.
A gluten free diet (choice A) is not necessary for this patient. Celiac or
nontropical sprue is a malabsorption syndrome that is treated with a gluten free
diet. This is a rare syndrome of gluten allergy that presents with diarrhea and
weight loss. This is not a likely diagnosis in this patient, because celiac sprue
usually presents in childhood as the patient starts to eat gluten in wheat-based
foods.
A colonoscopy (choice B) may be useful in the work-up of this patient, but it is
not the most appropriate next step. The CT demonstrated involvement of the
terminal ileum, which is highly suggestive of Crohn's disease. An upper
gastrointestinal (UGI) barium study will evaluate the small bowel for
inflammatory lesions. A colonoscopy can also be performed later to look for
involvement of the colon.
Prescribing broad-spectrum antibiotics (choice D) is premature at this time.
The patient's symptoms and CT findings are highly suggestive of an
inflammatory bowel disease and in particular, Crohn's disease. The supportive
treatment of Crohn's disease includes antibiotics, bowel rest, and parenteral
nutrition. Prior to starting antibiotics, it is important to confirm the diagnosis
first with an upper gastrointestinal (UGI) barium study.
Prescribing steroids (choice E) is premature at this time. Although steroid
therapy is a part of the treatment of Crohn's disease, it is important to confirm
the diagnosis first with an upper gastrointestinal (UGI) barium study.
You are notified that one of your patients, a previously asymptomatic 67-yearold woman, is being discharged from the hospital where she was being treated
for the past 4 days for severe left-sided lower abdominal pain, fever, and
shaking chills. Her temperature on admission was 38.3 C (101.0 F) and
laboratory studies showed an elevated leukocyte count with increased
neutrophils. She improved with intravenous antibiotic therapy, intravenous
hydration, and bowel rest. She is now concerned about her prognosis. You
should tell her that:
A. A barium enema should be scheduled at the follow-up visit in 3-4 days
B. Massive, painful hemorrhage along with abdominal pain and fever is a
common complication
C. Pneumaturia is a sign that immediate surgery is indicated
D. She will need to have a screening colonoscopy each year because of her
increased risk of colon cancer
E. Surgical management will be necessary in the future
Explanation:
The correct answer is C. This patient was admitted to the hospital for
diverticulitis and was treated successfully with intravenous fluids, antibiotics,
and bowel rest (NPO). Pneumaturia (air in the urine) is a sign of a colovesical
fistula, (a fistula between the colon and bladder), which is the most common
type of fistula formed in diverticulitis. This type of fistula requires surgery to
close the hole in the bladder and usually remove a segment of colon. The
diagnosis of a colovesical fistula involves a barium enema, cystography, and
intravenous pyelogram.
At discharge, a barium enema can be scheduled for 2-3 weeks after the acute
episode, not 3-4 days (choice A) to confirm the clinical impression. It should
not be performed during the acute phase of the illness because of the risk of
free perforation.
Massive, painful hemorrhage along with abdominal pain and fever is a
common complication (choice B) is incorrect. A massive hemorrhage may
occur when a patient has diverticulosis, but it is usually painless.
Diverticulitis is not typically associated with an increased risk of colon cancer
(choice D).
You should not tell her that surgical management will be necessary in the
future (choice E). Surgery is usually only necessary for approximately 25-30%
of patients with diverticulitis.
A 67-year-old man comes to the office for a follow-up visit to review the
findings from a colonoscopy that was performed 2 weeks earlier. A 0.9 cm
tubular adenoma was removed from his sigmoid colon. No other lesions were
visualized in the colon. He has no family history of colon cancer and is very
concerned when you tell him that the polyp was adenomatous. All previous
colonoscopies were normal. In explaining the findings to him, you should tell
him that:
A. A chest x-ray should be performed to ensure that there are no
abnormalities associated with the adenoma
B. A colectomy should be performed to avoid the risk of developing colon
cancer
C. Colonoscopy will be required every 6 months to determine if any new
polyps have formed
D. His children should have screening colonoscopies beginning at age 25
E. Tubular adenomas such as his have a low risk of malignant potential
Explanation:
The correct answer is E. Colonic polyps are very common in older patients,
with approximately 40% of all patients at age 60 having at least 1 adenomatous
polyp, (and 50% at age 70). Since there is a link between polyps and the
development of malignancy, it is recommended that polyps be removed and
evaluated. There are 3 types of adenomas—tubular, tubulovillous, and villous.
Tubular adenomas have a low risk for malignant foci (approximately 5% risk),
tubulovillous have an intermediate risk (approximately 20%), and villous have
a high risk (approximately 40%).
A chest x-ray should be performed to ensure that there are no abnormalities
associated with the adenoma (choice A) is incorrect. Since the case does not
say that the tubular adenoma found has any malignant foci, there will not be
any associated changes found in the lung.
A colectomy should be performed to avoid the risk of developing colon cancer
(choice B) is inappropriate. This patient does not have a malignancy and
therefore any additional treatment for this polyp is unnecessary. Prophylactic
colectomies are typically recommended for patients with autosomal dominant
polyposis syndrome because it is associated with an almost 100% risk of colon
cancer by age 40.
Patients with a history of colonic polyps need to be followed closely with a
colonoscopy every 1-3 years, not every 6 months (choice C).
Since about 40-50% of all patients in this age range will have at least 1 polyp,
it is not that unusual that this 67-year-old patient has a polyp. His children
should NOT have screening colonoscopies beginning at age 25 (choice D)
because this is too young and the chance of them having polyps at that age is
small. According to the United States Preventive Services Task Force, the
recommended screening for colon cancer is an annual fecal occult blood test
and/or a sigmoidoscopy every 3-5 years beginning at age 50.